Você está na página 1de 353

2013 OSCE

Booklet
Lia Stepan
Fe Menz
Lachy Stranks
Table of Contents
1

3. HISTORIES

189.
193.
197.
200.
204.
207.
210.
213.
216.
220.
224.
228.
230.
233.

- General Medicine & Geriatrics:


8. MMSE/Cognitive assessment
13. Fall in the elderly
19. Headache 1
21. Headache 2
26. Febrile illness (HIV)
29. Obesity
32. Painful rash
36. Polycythaemia
65. SOB 1
74. SOB 2
- Surgery:
10. Difficult swallowing
40. Prostate disease
46. Peripheral vascular disease
53. Back pain (renal colic)
58. Retinal detachment
- Orthopaedics:
4. Joint Pain
61. Scaphoid fracture + exam
- Obstetrics & Gynaecology:
81. Abdo pain (ectopic pregnancy)
85. Menopause
93. Pre-term labour
100. Pregnancy dating
104. Salpingitis
- Paediatrics:
111. Respiratory (Croup)
114. Febrile seizure
119. Anaphylaxis
- Psychiatry:
124. Anxiety 1
129. Dementia vs. Depression
134. Anxiety 2
137. Schizophrenic patient
140. Psychosis
145. PTSD
150. Suicidal ideation
158. Suicide attempt

236. COUNSELLING
237.
238.
240.
243.
246.
250.
255.
258.
261.
265.
266.
266.
267.
270.

SPIKES
ACEi commencement
Angry patient
Breaking bad news (CRC)
Childhood obesity
Genetic counseling
Cardiovascular risk factors
Pre-operative counseling
Skin lesion (melanoma)
Dementia
Oesophageal cancer
Breast cancer
Pap smears & HPV
Cervical cancer

271. DATA STATIONS


272. ABG interpretation
274. Acute renal failure
280. Assess foetal growth
284. Assess early child growth
288. Asthma management plan
291. Prescribing Asthma
294. Prescribing Statin
298. Prescribing Diabetes
302. Prescribing Analgesia
305. Prescribing Incontinence
307. Blood dipstick
311-18. ECGs
319. Pathology

164. PHYSICAL EXAMS


165.
168.
172.
174.
177.
182.
187.

Diabetic lower limb


Groin lump
Scrotal exam
Hearing assessment
Knee exam
Parkinsons exam
Peripheral vascular exam
Respiratory
Resuscitation (BLS, ALS)
Rheumatological hand
Shoulder 1 (impingement)
Shoulder 2 (dislocation)
Thyroid/neck
Trauma & Primary survey

Acute abdo 1 (cholecystitis)


Acute abdo 2 (appendicitis)
Driving assessment
Breast
Cardiovascular
Carpal Tunnel Exam
Cranial nerve

HISTORIES

JOINT PAIN
Ben Smith is a 68-year-old retired man who lives with his wife. Ben has
noticed pain in his left knee for the past day and has come to the ED as
he is concerned about this.
3

Description of task:
Take a focused history to formulate differential diagnoses and
explain them to the patient
Explain to the patient what investigations are needed

Patient Profile:
- Ben Smith
- 68 year old
- Lives with wife, retired lawyer
PC: Pain in left knee for past day
HPC:
4

- Knee pain;
Onset earlier this morning about 5 hours ago, gradually worsening
Severity 8/10
Burning deep pain in left knee
Nil radiation
Worse on movement, cannot move leg anymore
- Additional;
Knee is red and swollen, feels hot
Associated fever and malaise
Had been to ED 4 days previously for pain in same leg was found to have
Staph aureus cellulitis; was given antibiotics but didnt take them
Nil injury to the joint
Nil pain in any other joint
Nil family history, PMH, social history
DIFFERENTIALS FOR ACUTE ARTHRALGIA:
- Septic arthritis
- Crystal arthropathy gout, pseudogout
- Osteomyelitis
- Trauma fracture, haemarthrosis, dislocation
INVESTIGATIONS & MANAGEMENT:
- Urgent joint aspiration for synovial fluid microscopy and culture
- Empirical antibiotics flucloxacillin 2g IV to cover staph
- Blood cultures
- Bloods CBE, ESR, CRP
- Joint X-ray
- Further mgmt dependent on aspirate
- Consult with orthopaedics and microbiology

SEPTIC ARTHRITIS

- Infection of one or more joints by pathogenic inoculation of microbes


- 10x more common in people with underlying joint disease or with prosthetic joints
Aetiology & Pathophysiology:
- Most common organisms are Staph and Strep
Together account for 91% of cases
- Other organisms;
Gonorrhoea consider in sexually active patients
Gram ve elderly, immunocompromised
Anaerobes rare except in penetrating trauma
- Risk factors;
Underlying joint disease OA, RA
Joint prostheses
IVDU
DM, immunosuppression, alcoholism
Previous intra-articular corticosteroid injection
Cutaneous ulcers
- Inoculation either by direct inoculation or haematogenous spread
- Septic arthritis can destroy a joint in <24 hours, therefore urgent mgmt essential
Clinical features:
- Hallmark is hot, swollen, tender, restricted joint
- 60% have fever
- May have clinical features of primary focus of infection (if haematogenous spread)
e.g. pneumonia
6

Investigations:
- Urgent joint aspiration for synovial fluid MCS;
Perform gram stain and culture prior to commencing ABs - +ve in 70%
A ve result does not exclude septic arthritis
Exclude crystal arthropathies
- Blood culture prior to commencing ABs, +ve in 25%
- Bloods;
CBE raised WCC
ESR, CRP elevated
- Plain X-ray no diagnostic benefit but may reveal underlying joint disease
Management:
- All patients suspected for septic arthritis should be commenced on empirical ABs,
but only after joint aspiration and blood cultures (unless major delays)
- Usual ABs to cover common organisms;
Flucloxacillin (or vancomycin if MRSA)
Cefotaxime (from gram ve organisms)
- Consultations;
Microbiology for AB duration
Orthopaedics
Complications:
- Osteomyelitis
- Joint destruction
Prognosis:
- Irreversible joint destruction if Tx delayed
- 11% mortality

MMSE/COGNITIVE ASSESSMENT
7

STEM
Perform an MMSE and interpret the results

EXAM FINDINGS (FOR SP TO FAKE)


Orientation season correct (1/5)
8

Orientation (location) correct state, country, town (3/5)


Registration 3/3
Attention + calculation 1/5
Recall 0/3
Language 1/3 for 3 stage command but otherwise good (7/9)

WHAT STUDENT MUST DO


Introduce self
Explain task to patient
Do MMSE
Identify cognitive impairment
o Score 15/30
o Dementia cut off (according to QEH geris):
15-21 = early
9-15 = moderate
<9 = severe
Differentials
o Delirium UTI, pneumonia
o Dementia Alzheimers, Vascular, Lewy Body, B12/folate deficiency,
hypothryoid
o Depression
o Medications e.g. sedated from benzos
Investigations
o Take full history first, with collateral from family
o Physical exam fluctuating consciousness, signs of infection, CVS,
neurological etc
o Urinalysis
o Bloods
CBE, ESR, CRP - anaemia, infection (e.g. UTI)
Electrolytes (hypercalacaemia), blood glucose, LFTs (hepatitis,
alcoholism), renal function (renal failure)
Vitamin B12 + folate
Thyroid function (hyper and hypothyroidism)
Syphilis serology - rare, only present in those > 55
HIV antibodies - HIV is the most common cause of dementia in
young people
o Neuroimaging
All should at least get a CT head
CT atrophy, SOL, normal pressure hydrocephalus, subdural
haematoma, vascular disease
MRI Generalised atrophy with temporoparietal predominance (AD),
infarction + SOLs
Other PET, SPECT
o Further geriatric assessments nutrition, continence, falls screen,
depression screen, ADLs, iADLS

10

Interteaching OSCE

Generated by TY Kang MBBS V

OSCE Station History: (6 minutes)


You are a GP working at a practice you just joined yesterday, and so far its been
pleasant. A 72 year old fe/ male presents with difficulty swallowing for the last few
months.
Task:
1. Please take a history from him and
2. In the last minute, discuss differentials and investigations.

11

12

13

FALLS IN THE ELDERLY

You have 6 minutes at this station Description of task:


Fred Johnson is a 70-year old man who has been admitted to the ward for
investigation of falls.
He has fallen several times over the past two weeks and has severe
bruising over his buttocks and shoulders.
Take an appropriate focused history, with the intention of clarifying the
possible causes of his falls. Tell the examiner which diagnosis you think is
more likely.

14

Patient profile:
- Fred Johnson
- 70 years old
- Retired carpenter
- Widower, lives alone
PC: Ive fallen a few times over the past couple of weeks, and now Im pretty battered
and bruised
HPC:
- Fallen 4 times in the past 2 weeks;
All occurred at home, none witnessed
3 falls occurred inside after getting out of chair;
o Feeling fine beforehand
o Felt dizzy when getting out of chair and subsequently fell over
o Never lost consciousness
o Was on ground for no more than 20 minutes each time
o Did not hit head, but was quite painful where he landed (buttocks,
shoulder)
o Able to mobilise comfortable afterwards
1 fall occurred outside whilst gardening;
o Hot day, had not been drinking much water
o Got up from weeding, felt faint and then fell
o No LOC, no head injury, no fractures
o Wasnt able to get up straight away neighbor heard him calling for help,
spent an hour on the ground
o Subsequently admitted to hospital
- Able to move shoulders arms freely but significant pain, also in buttocks
- No palpitations
- No LOC
- No weakness
- No visual/hearing problems
- Did not trip, no obvious environmental cause
- Recently commenced on new antihypertensive medication;
Beta-blocker commenced 3 weeks ago
Already on ACEi and diuretic
Has been taking it as prescribed
- Has never fallen before, no previous fractures
PMH:
- HT, previous MI, CABG 10 years ago
- Stiff joints OA in knees but mobilises independently
- No memory problems, no depression
- Meds;
Antihypertensives ACEi, diruteic, B-blocker (started 3 weeks ago)
Aspirin
Sleeping tablet
- Smoker 10 cigs per day
- Alcohol only drinks on Thursdays at the bowls club
Social:
- Lives alone at home
15

- Wife died of breast cancer 15 years ago


- 3 adult children, sees them regularly
- Plays bowls once a week, otherwise not much physical activity
LIKELY DIAGNOSIS:
- Multifactorial, but likely cause of fall is orthostatic hypotension, occurring
secondary to;
Recent commencement of new, 3rd antihypertensive medication (beta-blocker)
PLUS Dehydration, hypovolaemia when he fell outside
- Contributing factors;
Osteoarthritis, impaired mobility
Polypharmacy
Deconditioning (little physical activity)
- Antiplatelet therapy likely contributing to significant bruising
- Given his PMHx, must also consider;
Arrhythmias
TIA
(although history is not suggestive of these causes)

Falls in the Elderly


- Common occurrence in elderly population, and are major factors threatening the
independence of older individuals
- Usually occur when impairments in multiple domains compromise compensatory
ability of the individual
A number of environmental and physical conditions that predispose to falls are
modifiable, and can be prevented to some extent
- Related to significant morbidity and mortality
16

Epidemiology:
- Increases with increasing age, and varies according to living status
- Community-dwelling;
30-40% of people >65 years fall each year
50% of people >80 years fall each year
- Long-term care setting;
50% of individuals in care fall each year
60% with Hx of previous fall will fall again
- Equal prevalence in men and women
- Account for majority of nonfatal injuries in adults >65 years, 5% will result in
hospitalization
Risk factors:
- Past history of a fall
- Lower extremity weakness
- Increasing age
- Cognitive impairment
- Balance problems
- Psychotropic drug use

Fear of falling
Arthritis
History of stroke
Orthostatic hypotension
Dizziness
Anaemia

Aetiology & Pathophysiology:


- Usually multifactorial in origin and reflect failure/demise of compensatory
mechanisms
- Can be broadly divided into intrinsic/medical and environmental causes
- Sensory impairment;
Poor vision
Poor hearing
Peripheral neuropathy
- Neuropsychiatric;
Vestibular dysfunction
Gait and balance disturbance Parkinsons, previous stroke
Cognitive/mood impairment dementia, depression, delirium
Seizure disorder
Subdural haematoma
CVA or TIA
- Cardiovascular;
Syncope arrhythmias, bradycardia, vasovagal syncope
Orthostatic hypotension
Carotid sinus syndrome
Post-prandial hypotension
Anaemia
- Musculoskeletal;
Joint bucking/instability
Mechanical mobility/gait abnormality previous fracture, arthritis
De-conditioning
- Medications;
Benzodiazepines
Antihypertensives
Antidepressants
Diuretics
Antipsychotics
NSAIDs
Sleeping medications
Marijuana
17

- Others;
Polypharmacy use of 5 or more medications increases risk by 30%
Substance abuse
Environmental poor lighting, uneven surfaces, loose rugs, ill-fitting shoes,
slippery floors

Clinical assessment:
- History of most recent fall;
Conscious or unconscious collapse/fall syncope, amnesia etc.
Was the fall witnessed?
Intrinsic or environmental?
o Palpitations, syncope, visual disturbance, neuropathy, weakness etc.
o Poor lighting, uneven surface etc.
Were they able to get up after the fall? How long were they on the ground?
Complications fractures, bruising, bleeding, head injury etc.
- History of previous falls/PMHx
Number of falls (>2 falls within a 12 month period is considered significant)
Previous fragility fractures? If yes, patient must receive bone-strengthening
meds
Other complications
PMHx previous stroke/TIA, CVD, osteoporosis, DM etc.
- Medications;
Polypharmacy (>5 meds) is a risk factor for falls
Significant meds for falls;
o Benzodiazepines
o Anticholinergics
o Psychotropics (antipsychotics, antidepressants, sedatives)
o Diuretics
o Antihypertensives
o Statins
Anticoagulants, antiplatelets bleeding risk
- Physical exam;
General wellbeing level of self care, nutritional status, body habitus
CV exam arrhythmias, postural BP, bradycardia
Gait assessment
Balance assessment;
o Timed up and go test;
Time taken to get out of chair, walk at a comfortable speed for 3
metres to a line on the floor, turn, return to chair and sit down
Time taken >15 seconds suggests a high risk of falls
o Single leg stance test observe patient standing on one leg with eyes
open on firm surface for 10 seconds; repeat 2 more times
Lower limb strength
Cognitive assessment
Visual acuity and hearing assessment
Assessment of feet and footwear

Investigations:
18

- May be indicated based on Hx and exam, but no standard diagnostic


evaluation exists for a person with a history of or at high risk for falls
- Investigations are to rule out potential causes
- Serology;
CBE anaemia
Biochemistry elevated urea and creatinine (dehydration)
BSL, HbA1c
Vitamin D
- ECG, echo
- Brain CT/MRI
- EEG (if seizures suspected)
- Spine x-ray, MRI
- DEXA scan

Falls prevention:
- Exercise gait and balance training, muscle strengthening, flexibility,
endurance
- Medication assessment and modification especially benzos/psychoactive
drugs
- Vitamin D supplementation;
High dose cholecalciferol especially if vitamin D low
- OT assessment;
Hazard reduction in the home
Proper footwear
Training and education
Mobility devices
- Vision glasses, cataract surgery etc.
- Podiatry review
- Indications for referral to Geriatrician/falls clinic;
>2 falls in 12 month period
Unexplained falls with syncope, dizziness or poor recall
Falls as part of downward physical, social or psychological spiral
Falls occurring at low threshold (e.g. with basic ADLs)
Falls with head injury, low trauma fracture on floor >1 hour
Gait disturbance or unsteadiness

Complications:
- Falls often result in injury of some type, usually minor soft tissue injuries
(bruises etc.)
41% of falls result in minor injury
6% result in major injury (fractures, head trauma, lacerations etc.)
- Fractures NOF, pelvis, upper limb (fall on outstretched hand)
- Head injury
- Bleeding, bruising, lacerations
- Downward physical/social spiral, reduced QOL
- Loss of independence/ability to perform ADLs
- Rhabdomyolosis, acute renal failure if immobile on floor for prolonged period
- Hospitalisation (5% of falls result in hospitalization)
19

- Death

20

HEADACHE 1

Mary Smith is a 55 year old lady who has been experiencing newonset headaches for the last 5-6 weeks.
Please:
Take a history from Mrs Smith (4 minutes)
Ask the examiner for relevant results of the physical examination
that you would perform (1 min)
Discuss differential diagnoses and further investigations (1 min)

HISTORY
Profile
o 55 years old
o Accountant
o Lives at home with her husband and teenaged children
PC
o 5-6 weeks of headache
Headache
o Non localised
o Worse in the morning, worse on lying down
o Non responsive to analgesia
o Hasnt noticed coughing making it worse
o Never had this before
o Has not been unwell recently
Last week or so has had some double vision and been unable to look with her
left eye to the left side
Nil seizures noticed, husband has been telling her that she has had short
moments of unresponsiveness
Weird feelings of dj vu lately
Slightly less propensity for concentration
Otherwise well
No hx cancer
No melanomas previously diagnosed, gets skin checked regularly
No recent trauma

Physical Findings
Papilloedema
6th nerve palsy
Right sided homonymous hemianopia
Upbeat nystagmus

Diagnosis with differentials


Space occupying lesion GBM
o Other tumour (30% are metastatic from breast, lung, melanoma)
o Idiopathic intracranial hypertension
o Trauma

Investigations
CT head

HEADACHE 2

Charles / Charlotte Young (Fe)Male, age 3050, presents with a


history of headaches over the past month.
Tasks for the station:
1. Please take a history from this patient in relation to the
presentation. (4 minutes)
2. Tell the examiner what you consider is the relevant examination
and ask for the results of that clinical examination. (1 minute)
3. Inform the examiner of your likely diagnosis or diagnoses and
what would be the important investigations if any required. (1
minute)

Patient profile:
Charles Young
45 year old man
Works as lawyer
PC: Headache for the past few weeks
HPC:
- Headache;
Present for the past month
Worst at the back of head but hard to localise, not feeling pain anywhere else
Worse when lying down, coughing, leaning forward
Worse in the mornings when getting out of bed
Never experienced anything like this before, never had migraines
Visual changes blurred vision a couple of times in R eye in the past week
blacking/blurring of vision
Sometimes feels nauseous when waking up in the morning, has vomited twice in
the past couple of days on waking
No dizziness
No weakness
Slightly more tired than usual, but is doing long hours at work
No migraine features no photophobia, no phonophobia, no aura etc.
No tinnitus
No obvious trigger has not bumped head, no caffeine, drugs etc.
PMH:
Nil significant
Asthma as child
No operations
Not on any medications, occasional Panadol for headache but with little effect
No history of malignancy
Nil allergies
SMOKING/ALCOHOL:
Smokes half a pack/day, has been smoking since age 12
Drinks socially on weekends
FAM/SOCIAL:
Nil significant family history
Happily married, 3 children, all well

DIFFERENTIALS:
Raised ICP of any cause
Brain tumour
Intracranial haemorrhage
Migraine unlikely
Other headache causes unlikely tension, cluster

EXAMINATION:
Vitals pulse rate and rhythm, BP
Clinical features of raised ICP;
Papillodoema bilateral

Fluctuating conscious level


Cushings triad HT, abnormal breathing, bradycardia
Visual function testing peripheral vision, ocular movements (may have palsy of
abducens nerve)
Full neurological examination any features of UMNL

INVESTIGATIONS:
Bloods CBE, biochemistry
CT/MRI head
If SOL identified, further investigation and referral required

HEADACHE HISTORY

Introduction
History of presenting complaint;
Headache;
o Location of pain
o Onset of pain, duration
o Character
o Radiation
o Chronicity
o Severity
o Aggravating/relieving factors inc. time of day
o Ever had anything like this before?
Associated symptoms;
o Aura, flashing lights
o Nausea, vomiting
o Photo/phonophobia
o Lacrimation, rhinorrhoea (cluster headache)
o Neck stiffness
o Weakness, paraesthesia
o Fever
Possible causative factors;
o Head injury
o Lack of sleep
o Stress, exertion
o Refractive errors
o Caffeine, alcohol
Past medical history;
History of headaches? Migraines?
Medications OCP
Alcohol, smoking, caffeine
Family history cancer, headaches

Raised intracranial pressure:


- Headache worse in mornings and after lying flat
- Associated with nausea and vomiting

- Often have transient unilateral or bilateral visual disturbances blurring,


blackening
- Signs;
Altered conscious level
Papilloedema
Abnormal ocular movement, sluggish light reflexes
Bradycardia
Systemic HT
- Causes;
Tumours
Subdural/extradural haematoma
Brain abscess
Hydrocephalus
Benign intracranial hypertension
Aneurysms, AV malformations
- Investigations CT/MRI head

Migraine:
- Defined as recurrent headache that occurs with or without aura and lasts 2-48
hours
- Characteristic features;
May have preceding aura flashing lights etc.
Usually unilateral
Gradual onset (15-30 mins)
Pulsating/throbbing in quality
Moderate-severe intensity
Aggravated by routine physical activity
Commonly associated with nausea, vomiting, photo/phonophobia
- Investigations clinical diagnosis
- Management;
Identify any precipitating factors and remove them (e.g. dark chocolate, caffeine
etc.)
Withdraw OCP if recurrent episodes
Acute attack;
o First line = simple analgesia i.e. paracetamol, NSAID (2x aspirin + 1
maxalon +/- 3 panadol)
o If these fail, codeine can be used
o Others antiemetics if vomiting, ergotamine
Prophylaxis for recurrent episodes;
o Propranolol
o TCAs e.g. amitryptilline
o Flunarizine (Ca2+ channel blocker)
Relaxation techniques

Tension headache:
- Most common cause of headache
- Characteristic features;
Steady dull ache
Bandlike sensation around head

Usually bilateral
Episodic
- Can occur chronically

Cluster headache:
- Occur in clusters i.e. periodically
- Characteristic features;
Severe headache, typically steading boring sensation often behind one eye
Tearing (lacrimation)
Red eye
Runny nose/nasal congestion (rhinorrhoea)
Horners syndrome

Subarachnoid haemorrhage:
- Medical emergency
- Due to rupture of intracranial Berry aneurysm
- Characteristic features;
Severe, acute headache thunderclap, worst pain ever felt
Meningism
Decreased consciousness/LOC
Focal neurological deficits
Features of raised ICP +/- herniation

Temporal arteritis:
- Unilateral throbbing headache
- May have associated visual disturbances and jaw claudication
- Tender to palpation over temporal artery
- Associated with polymyalgia rheumatic
- Requires temporal artery biopsy for diagnosis with subsequent corticosteroid
treatment

Station Name: HISTORY


1 minute reading time
6 minute question time

Instruction to Students
You have 6 minutes at this station. You will be warned at 5 minutes. If you finish early, you will be asked
some questions for bonus marks.
Description of task:
You are the student in a local GP practice and have been asked to see Miss Carmen Sandiago, a 22 year
old woman. She has had a fever, and been feeling unwell.
Take a history from this woman with a view towards a diagnosis

OBESITY

An obese patient (BMI 35) is concerned about their weight and its
effect on their health.
Please take a history focussed on the following two aspects of their
obesity:
1) factors leading to their obesity (4 minutes) AND
2) obesity-related complications (2 minutes)
This is a history station. DO NOT give management advice.

Patient profile:
Barry Chubb
42 years old
Works in IT, lives alone
PC: concerned about his weight and what it might be doing to my health
HPC:
- Has always been big since he was a teenager, but is now at his heaviest (BMI
35)
- Has just gradually been putting on weight up to this point
- Previous attempts at weight loss;
Tried a few diets he saw advertised on TV and bought an AbSwing Pro none of
these worked
Wasnt exercising on the diet and would still snack on junk food, became
frustrated with lack of results so gave up
Has never tried drug therapy
Has never had gastric stapling
- Reasons for gaining weight;
Does not exercise sits behind desk for work, watches a couple of hours of TV
per night
Diet lots of meat, some vegetables, gets McDonalds for dinner a couple of
times per week because he cant be bothered cooking, lots of soft drink,
icecream
Drinks socially on weekends
- Family history;
Mother was obese and had high blood pressure died from MI when she was 63
Father has DM
Brothers and sisters all on the bigger side
- Obesity related complications;
Occasionally gets reflux uses antacid, not a major problem
Has some back pain and knee pain was told he probably has osteoarthritis
Can walk 500m before getting puffed
Has never been told he has diabetes, but sugars were borderline on his last
test
Hypertensive 150/100
High cholesterol
His weight does get him down feels socially isolated and constantly judged,
has no hope of finding a partner when he is this big I eat becomes Im
unhappy, but Im unhappy because I eat
Has never had chest pain, SOB

PAINFUL RASH

You have 6 minutes at this station


Description of task:
The patient had a painful rash which has cleared up 6 weeks ago.
She still has pain.

- Take a focussed history (4 minutes)


- In the last 2 minutes describe to the examiner the patients
symptoms, give the most likely diagnosis and its natural history and
suggest a management plan for this patient.
- You will receive a warning when 4 minutes have passed.

Profile
Joan Colins
72 years old
Widowed

PC
Painful area over chest

HxPc
Had a rash over current area of pain about 6 weeks ago
o Very painful burning, stinging
o Red, bumpy, weepy
o Lasted for about a week
o Located unilaterally over chest
o Went away by itself
o Pain started a few days before the rash and is still there now
o Skin where rash was is currently more sensitive than usual

Nil cardiac symptoms


Nil SOB

MHx
No allergies
Had chickenpox when she was a child
Hypertension metoprolol
Osteoporosis vitamin D and calcium
Has been taking panadol for pain but no relief
Otherwise well

FHx
Grandchild has just caught the chicken pox
o Sees him regularly
Father died of MI at 60
Mother died naturally
No hx other diseases

SHx
Smoker
o 10/day for 20 years
Non-drinker
No illicit drugs

MARKING SCHEME
Introduction
Empathy

Lives alone
Received pension

Nil recent trauma


Otherwise well

History
o Pain history finds out onset and relationship to rash
o Excludes cardiac/resp/traumatic causes
o Asks about history of chicken pox
HERPES ZOSTER
This is a sporadic disease that results from reactivation of latent VZV in the
dorsal root ganglion.
It can occur at all ages, but has the highest incidence in individuals in their sixth
decade of life and beyond.
Presentation
Characterised by a unilateral vesicular eruption within a dermatome
Often there is severe pain
Dermatomes T3-L2 are most commonly involved
Distribution:
o 50% thoracic
o 10-20% trigeminal
o 10-20% cervical
o Disseminated in HIV
The factors leading to reactivation are unknown. In children the disease can be
relatively benign, but in adults it can be debilitating.
The pain is called zoster-associated pain:
Disease onset is marked by pain within a dermatome
o This may precede rash by 24-72 hours
An erythematous maculopapular rash evolves rapidly into vesicular lesions (and
bullae, pustules
In the normal host, these lesions tend to continue to form for 3-5 days and the
disease generally resolves within 7-10 days, however it may take as long as 2-4
weeks for the skin to return to normal.

If located over the trigeminal nerve distribution, this is called Ramsay Hunt
Syndrome
Taste lost at anterior 2/3 of tongue
EAC pain
Ipsilateral facial palsy

Patients with herpes zoster can transmit infection to seronegative individuals


chickenpox
Risk Factors
Immunosuppression
Old age
Occasionally associated with haematological malignancy

Complications
The most debilitating outcomes are:
Acute neuritis
Postherpetic neuralgia

Postherpetic neuralgia is uncommon in a young population, but at least 50% of


patients aged >50 report at least some pain for months after resolution of the
cutaneous disease. This pain must have been present for AT LEAST 3 MONTHS.
Changes in sensation of the dermatome (hyper or hypoalgesia) are common.
Differentials
Consider other causes of chest pain if there is no rash
Rash:
o Contact dermatitis
o Localised bacterial infection

Management

Gabapentin 300-600mg PO TDS for post herpetic neuralgia, or TCA, or


anticonvulsants
If active rash:
o Compress with normal saline or betadine solution
o NSAIDs, amytriptiline
o If over 50 years with severe acute pain or ophthalmic involvment, or
immunocompromised, vancyclovir
Non pharmacological
o Acupuncture
o Relaxation techniques
o Heat/cold packs
o TENS
Outcome
In the majority of cases, there is gradual resolution of the pain over around 5
years. Some patients may have this for life
Manage with TCAs or gabapentin

POLYCYTHAEMIA

Description of the task:

A 65-year-old man has presented to the GP to receive his results


from the blood test that you performed a week ago due to his 3
month history of shortness of breath and cough. The CBE results are
shown below.

1. Take an appropriate history to determine the cause of this problem


2. Tell the examiner your differentials and what investigations are
required to distinguish them
3. Tell the examiner how you would manage this patient

Hb:
196
(135-175)
RBCs:
9.0 1012/L (4.5-6)
PCV:
0.49 L/L (0.4-0.5)

MCV:
WBCs:
Platelets:
Haematocrit:

Patient profile:
John Smith
65 year old man
Retired carpenter
PC: Told to return for blood test results that were taken as he has been SOB and
coughing
HPC:
Has been feeling increasingly SOB over the past few months which is getting
worse
Used to be able to walk for 20+ mins without feeling SOB, now struggling to
walk 5 minutes on the flat
Persistent cough for few months to year
o Worse in mornings
o Usually clear, occasionally discoloured (brown/green)
o No blood
Frequently gets chest infections 1-2 a year, requiring antibiotics
Not feeling unwell at the moment, has not lost weight, a little tired

90 fL
(80-98)
8.6
(4-11)
260
(150-400)
Increased

This blood profile indicates polycythaemia.

Currently smoking;
o Pack a day
o Started age 17
PMH:
Generally well, rarely sees doctors
Has never been told about lung problems
Surgical cholecystectomy at age 50
Meds friend suggested Ventolin but hasnt been helping
Nil allergies
Smokes
Drinks alcohol on weekends
Nil illicit drugs
SOC:
Retired carpenter
Lives with wife well
FAM:
Father died of lung cancer at age 65 heavy smoker
Mother died of stroke age 80
Diagnosis: Likely COPD causing secondary polycythaemia (due to chronic
hypoxia)

Polycythaemia

- A haematological abnormality characterized by increased number of


circulating RBCs (erythrocytosis) i.e. increased RBC mass
Also commonly associated with thrombocytosis and leukocytosis
- Classification;
Relative (raised RBCs as a result of reduced plasma volume, normal RBC mass)
Absolute (raised RBC mass)
o Primary polycythaemia rubra vera
o Secondary caused by hypoxia or inappropriate increase in EPO
- CBE findings;
Raised RBCs
Raised Hb
Raised haematocrit
Raised PCV
May also have elevated WCC and platelets (depending on cause)

Relative polycythaemia:
- An apparent rise in circulating RBCs secondary to a decrease in plasma volume
- RBC mass is normal, however Hb, haematocrit and RBC count are elevated
- Typically an acute change secondary to;
Dehydration diuretics, reduced fluid intake, alcohol
Stress
Burns
- A less common, chronic form exists Gaisbocks disease, associated with;

Male gender more common


Hypertension (which produces reduced plasma vol)
Obesity
High alcohol and tobacco intake

Absolute polycythaemia:
- The overproduction of RBCs with either a primary or secondary cause;
Primary myeloproliferative disorder caused by an acquired or inherited
mutation abnormality with RBC precursors (Polycythaemia rubra vera)
Secondary as a result of chronic hypoxia or increased EPO

Polycythaemia rubra vera:


- Myeloproliferative disorder characterized by neoplastic proliferation of cells
derived from the pluripotent marrow stem cell
- Results in;
Erythrocytosis
Leucocytosis
Thrombocytosis
- >90% of cases are related to a mutation in JAK2 (JAK2 V617F)
- Associated with thrombotic complications as a result of blood hyperviscosity
- Clinical features;
May be asymptomatic and detected on routine CBE
Alternatively may present with vague S+S related to hyperviscosity;
o Headaches
o Dizziness
o Tinnitus
o Visual disturbance
o Itch after hot bath/shower
o Erythromelalgia sudden, severe burning pain in hands and feet
Signs;
o Facial plethora
o Splenomegaly (60%)
o Gout (increased urate due to increased RBC turnover)
o Arterial or venous thrombosis
- Investigations;
CBE as above
Marrow aspirate hypercellularity with erythroid hyperplasia
Reduced serum EPO
- Treatment;
Young patients venesection
Old patients hydroxycarbamide
- Complications;
Thrombosis venous or arterial
Transition to;
o Myelofibrosis (30%)
o Acute leukaemia (5%)

Secondary polycythaemia:
- Increased RBC production as a result of;

Chronic hypoxia (increased O2 carrying capacity required erythrocytosis)


Increased EPO production
- Chronic hypoxia;
COPD
Cyanotic congenital heart disease
High altitudes
Heavy smoking
- Increased EPO production; (AKA tumour polycythaemia)
Renal carcinoma
Hepatocellular carcinoma
Phaeochromocytoma
haemangioblastoma
- May also be secondary to EPO doping in athletes

PROSTATE DISEASE

Mr McClean is a 68 year old man who has been having trouble with
his waterworks and has been having some back pain.
Instructions for students:
Take a history and give most likely differentials (4 minutes)
List further investigations and justify why you are ordering them
Answer the examiners questions (2 minutes)

EXAMINER QUESTIONS
Four minutes has passed
o List differentials and what is the most likely
Which investigations would you like to do and why?
What are your management options if this patient was shown to have prostate
cancer?
How would you manage this patient?
HISTORY
Profile

Don McClean
She is well
68 years old
Pensioner
Lives at home with wife
PC
Trouble with waterworks and back pain
HxPc
Difficulty passing urine for the last year or so
Takes 20 minutes for him to be able to pass urine
o Stopping/starting stream
o Passes small volumes
o Feels as though the void was incomplete
o Dribbling
Has the sudden urge to empty his bladder
Happens nearly all the time
Gradual onset, getting worse
Has to get up to use the bathroom during the night (3-4 times)
Nothing seems to make it better or worse, but has tried running the tap and
using warm water to get things going
No blood in urine
No loin pain
No offensive smell of urine
No discharge
No pain
No change in bowels
No overflow incontinence
Still able to get an erection
No fever
Back pain
o Started a few weeks ago
o Constant pain
o Lower back
o Nothing makes it better
o 7/10
o Worse at night difficulty
o No radiation to legs
sleeping

Has never had his prostate checked


MHx
Nil allergies
UTIs
o A few over the last few years
Kidney stones
o Once, 10 years ago
Otherwise well
No medications
Doesnt like going to the dr
FHx
Grandfather died of prostate cancer at 75
No other Hx
SHx
Non smoker
Drinks 1 beer/night
2 children both well
DIFFERENTIALS
Prostate cancer
o Most likely
BPH
Prostatitis
o No blood, no pain, less likely
Renal colic
UTI/pyelonephritis
Urethral stricture/tumour
INVESTIGATIONS
PSA
Ultrasound
o Size, shape of prostate
PR
o Smooth BPH
o Rough, stony hard
malignancy
Trans-rectal biopsy
Vertebral XRAY/bone scan
o Mets
Cystoscopy
Urinalysis and culture
o Rule out UTI
Renal function
If suspecting calculi
o Abdo XRAY

MANAGEMENT AND TREATMENT


Radical Prostatectomy:

The patient most likely to benefit from radical prostatectomy is one with a
relatively long life expectancy (>10 years)
Who have a low PSA
Moderately differentiated tumour
Interstitial Radiotherapy:
Patient group most likely to benefit are those with high life expectancy and low
volume, low grade disease
No Initial Treatment:
Those who have a preference for no intervention, long life expectancy, low
volume disease, moderately differentiated disease
TURP:
A vasectoscope is inserted through the urethra and sections of the prostate are
removed in order to increase the opening of the urethra
Complications can include:
o Incontinence
Permanent dilation of the urethra
Effects on external sphincter
o Infection
o Bleeding
o Retrograde ejaculation

BRACHYTHERAPY
In patients with METASTATIC DISEASE, the aim of treatment is to control
symptoms and to retard the disease progression. Most cancers are androgendependent, at least initially, and hormonal manipulation is the mainstay of
treatment of advanced disease. Local radiotherapy is frequently effective for
treating painful metastases.

HORMONAL THERAPY
There are three main treatment options:
REMOVAL OF BOTH TESTES BY SUBCAPSULAR ORCHIDECTOMY
o Quick and simple procedure that removes around 95% of the testosterone
synthesised (the rest is from the adrenals), producing an immediate fall in
plasma testosterone
MONTHLY INJECTIONS OF DEPOT LHRH AGONISTS
o Need to be administered at intervals ranging from 4-12 weeks
o Therapy causes initial stimulation of LH from the pituitary, which turns up
testosterone secretion for up to 2 weeks, which is followed by inhibition of
LH release by competitively blocking the receptors
o Many patients experience a flare in symptoms in the first 2 weeks,
aggravating bone pain or spinal compression
o For this reason, the first dose is usually covered by anti-androgen therapy
ANTI-ANDROGEN DRUGS SUCH AS CYPROTERONE ACETATE OR FLUTAMIDE
o These block the binding of dihydrotestoerone to its receptor at a cellular
level

5-ALPHA-REDUCTASE INHIBITORS
Examples Include:
Dutasteride
Finasteride
Mode of Action

They inhibit 5-alpha-reductase, which is the enzyme that converts testosterone


into dihydrotestosterone, which is an androgen that stimulates prostatic growth
decreased prostatic enlargement
Indications
BPH
Contraindications
Adverse Effects
Common
o Impotence, decreased libido, ejaculation disorder
Uncommon
o Breast tenderness or enlargement
Rare
o Allergic reaction
ALPHA-ADRENDERGIC BLOCKERS
Examples:
Alfusozin
Mode of Action
These block alpha1 receptors smooth muscle relaxation in the bladder neck
and prostate decreased resistance to urinary flow symptom relief in
BPH/obstruction
Indications
Symptom relief in BPH
Contraindications
Hepatic impairment
Adverse Effects
This is a new drug so adverse events are relatively unknown

GNRH ANALOGUES
Examples Include:
Goserelin
Leuopolide
Mode of Action
Stimulates production of testosterone in a continuous manner (non-pulsatile)
increased LH production feedback to pituitary gland down-regulation of
GnRH receptor down regulation of testosterone production eventual
cessation of hypothalamus-pituitatry-gonad axis
Indications
Prostate cancer (specifically Goserelin)
Endometriosis
Uterine fibroids
Breast cancer
Contraindications
Pregnancy
Breastfeeding
Unexplained vaginal bleeding
Polycystic ovarian disease
Pituitary Adenoma
Adverse Effects
Common

o Transient changes in BP, hot flushes, sweats, sexual dysfunction, reduced


libido
Uncommon
o Bronchospasm, rash
Rare
o Depression, hypersensitivity reactions
ANDROGEN-AGONIST
Examples
Bicalutamide
Mode of Action
Competitively inhibits the binding of androgens (eg, testosterone) to androgenreceptors
Indications
Metastatic prostate cancer with GnRH agonist
o This prevents the initial surge of testosterone from the GnRH agonist
prostatic growth prior to receptor downregulation
Prevention of GnRH-agonist associated initial tumour flare
Locally advanced prostate cancer
Contraindications
Consider dose reduction in hepatic impairment
Adverse Effects
Common
o Dizziness, dyspnoea, constipation, dry skin, rash, weakness
Rare
o Thrombocytopaenia, CVS disorders (angina, heart failure, arrhythmias,
ECG changes), pneumonitis, pulmonary fibrosis
MANAGEMENT OF THIS PATIENT
Await results and gleason score, staging, PSA
Low risk:
o T1 or T2a
o Gleason score </= 6
o PSA </= 10ng/mL
High Risk
o T3
o Gleason score 8-10
o PSA >20
Intermediate in between these values
Low risk:
o Active surveillance
o Radical prostatectomy if young and healthy
If high risk:
o Radical prostatectomy if young and healthy
o RadiotherapY

PERIPHERAL VASCULAR DISEASE

Description of task:

Mr Brian Watson, a 73-year-old retired builder, has come to you (his


GP) as he has been experiencing pain in both legs when walking.

He has a history of coronary artery disease and MI, which occurred


10 years ago.

Tasks for the student:


Take a focused history from the patient
Explain the likely diagnosis
Describe what you would look for on examination of this patient and
what investigations you would order.

ID: Brian Watson, 73-year-old retired builder, lives at home with wife
PC: Ive been having pain in my legs when I walk doctor
HPC:
- Pain in legs;
Present for 2-3 months
Occurs on walking, relieved with rest
Pain located in calves, both sides but worse on the left
Can now walk about 50m on flat before pain develops, could previously walk a
few kms
Gradually getting worse
- No pain at rest
- Has not noticed any ulcers, lesions on legs/feet
- No acute critical ischemia
- Associated symptoms;
Erectile dysfunction present for the past year or so, getting worse
Has been having some minor chest pain when walking in past couple of weeks,
relieved with GTN and rest
PMH:
- MI 10 years ago doctors told me it was a small heart attack
- Hypertension for quite a few years
- High cholesterol
- Not diabetic
- Meds a few have been prescribed but I dont always take them
Aspirin
GTN
ACEi
Statin
- No operations
- No allergies
Smok/Alc:

- Smokes 10 cigs/day has been smoking since his 30s


- Drinks beer socially on weekends
- No illicit drugs

DIAGNOSIS:
- Intermittent claudication transient, exercise-induced muscular pain that is
relieved with rest
Suggests that there is a build up of plaques/fatty tissue in the large arteries of
your legs just like in your heart when you had your MI
- Only differential is spinal claudication - pain in the buttocks and legs brought
on by exercise with the back in an extended position

PERIPHERAL VASCULAR EXAM

- Performed with patient supine


- Always compare both sides

General inspection:
- Patient able to comfortably move onto bed?
- Body habitus overweight/obese
- Any obvious respiratory distress etc.

Inspection of legs:
- Colour of skin pale/purple/black?
- Trophic changes;
Distal hair loss
Temperature change
Shiny, dry, scaly skin
- Muscle atrophy, scars
- Ulcers, gangrene check whole foot and ankle (inc. between toes, heel, sole)
Arterial punched out, painful lesions over pressure areas/bony prominences
Venous irregular, moderately painful lesions over gaiter area
Neuropathic (diabetic) deeply penetrating, painless lesions over pressure
areas
- Venous changes;
Varicose veins (best seen with patient standing)
Venous ulcers, thrombophlebitis
Haemosiderin deposition, venous eczema

Palpation of legs:
- Temperature change;
Start distally and move up leg
Compare both sides simultaneously

- Capillary refill of nail bed normal <2 seconds


- Palpate aorta and peripheral pulses start distally and move up
- +/- check for radio-femoral delay to exclude coarctation of aorta

Buergers test:
- Test for arterial insufficiency
- Raise leg allowing to 45 degrees, allowing blood to drain from leg
In a leg with normal circulation, the foot and toes will remain pink, even with leg
rasied to 90 degrees
In a leg with arterial insufficiency, the foot/toes will become pale
- Then ask patient to dangle leg over side of the bed
In presence of PVD, foot will become pink-red and painful (reactive hyperaemia)

Auscultation:
- Auscultate for renal and femoral bruits

Other tests:
- Trendelenburgs test to assess level of venous insufficiency in patients with
varicose veins
Patient lies supine and leg is flexed at hip to allow emptying of veins
Tourniquet/hand pressure is then applied at the sapheno-femoral junction (2cm
below and 2cm lateral to pubic tubercle)
Patient then asked to stand rapid filling of varicosities with tourniquet still on
suggests incompetence is below SFJ
Depending on result, tourniquet can be raised or lowered and test repeated to
determine level of incompetence
- Sensory examination

Peripheral Vascular Disease

- PVD refers to a wide range of arterial syndromes, but particularly obstructive


disease (i.e. atherosclerosis) of major lower limb arteries ischaemia
Also occurs in upper limbs, but much less common
- Defined as ABI <0.9

Epidemiology:
- Prevalence increases with age, from age 40
50-59 years = 3-5%
60-69 years = 5%
80 years = >20% (>25% in men)
- More common in men

Aetiology & Pathophysiology:


- Most commonly caused by atherosclerotic disease
- Risk factors thus the same as for other causes of atherosclerosis (DM, HT,
hyperlipidaemia, obesity, increasing age)
- Atherosclerosis causes obstruction and narrowing of arterial lumen distal
ischaemia

- Thus ischaemia produces the 3 cardinal symptoms of PVD (represents


progression of disease);
Intermittent claudication exercise-induced leg pain relieved with rest
Pain at rest
Gangrene
- Over time, the disease also affects the neurologic and metabolic function of
the lower limb
Muscle atrophy, functional decline
Neuronal ischaemia sensory loss
- Rarer causes of claudication inc.;
Aortic coarctation
Vasospasm
Arterial dissection
Arterial tumour
Thrombosis, embolism
Spinal claudication
Trauma
Clinical features:
- Majority of patients are asymptomatic and diagnosis is based on risk factors
- Intermittent claudication;
Exercise-induced muscular pain in lower limb, relieved with rest
Occurs with ABI between 0.5-0.9
Must determine how far patient can walk (on flat) before pain occurs, and how
far they could previously walk
Often more obvious in one limb, producing a limp
Pain almost always in calf, but may extend up to thing and buttocks (depending
on level of disease)
- Ischaemic rest pain (chronic severe ischaemia);
Occurs with more severe arterial occlusion may be a sign of critical limb
ischaemia
Occurs with ABI <0.5
Very severe burning pain that occurs at rest occurs mostly at night (reduced
effect of gravity, reduced CO, reactive dilation of skin vessels to warmth)
Characteristically relieved to a degree by dangling leg over side of bed, and
completely relieved with analgesics
Often associated with trophic changes in the limb hair distribution, skin
changes, temperature
NB: in patients with diabetic neuropathy, severe ischaemia may be painless
- May present with arterial ulcers/gangrene
- Acute critical ischaemia (or acute-on-chronic ischaemia);
Severe pain
Paralysis
Pallor
Paraesthesias
Pulselessness
Perishingly cold
- Erectile dysfunction is a common early symptom of PVD that should be asked
about in men may suggest narrowing of internal iliac arteries
- Physical examination as above
- Other features to determine on Hx;
Erectile dysfunction
CVD CAD, MI
Cerebrovascular disease
FHx

Investigations:
- Doppler US of lower limb;
Assess blood flow in lower limb, as well as location and degree of stenosis
Normal pressure is slightly above brachial systolic
Patients with claudication range from 50-120 mmHg
Peak systolic velocity >2 = stenosis >50%
- Ankle brachial index (ABI);
Ratio of BP in lower limbs compared to BP in upper limbs i.e. ABI = systolic BP of
dorsalis pedis (or posterior tibial)/systolic BP of brachial artery
Assessed via Doppler US
Results;
o 0.9-1.2 = normal
o 0.5-0.9 = moderate arterial disease intermittent claudication
o <0.5 = severe arterial disease ischaemic rest pain, gangrene
o >1.2 = abnormal/artificially elevated result due to arterial calcification in
PVD causing artificial patency of vessels
- Toe pressure test;
Performed if ABI normal but patient is clinically abnormal, or if ABI >1.2
If toe pressure is <30mmHg in a non-diabetic with an ulcer, it suggests this will
not heal
- Arteriography (CT or MRI);
Reserved for patients thought to require intervention in the form of angioplasty
or reconstructive surgery
Provides map or arterial system, showing sites and severity of stenosis
Does not measure rate of blood flow
- CBE to exclude polycythaemia or thrmocythaemia (hypercoagulability
disorders)

Management:
- Intermittent claudication (mild-moderate disease);
Lifestyle modification;
o Walking/exercise plan
o Weight loss
o Smoking cessation
o Foot care and appropriate
foot wear
Medical;
o Antihypertensives
o Statins (even if cholesterol levels normal)
o Aspirin
Most patients do not require revascularisation
- Disabling claudication;
Lifestyle modification as above (first line)
Medical therapy as above
If there is no improvement, patient should be referred to vascular surgeon for
assessment for revascularisation
Methods of revascularisation;
o Balloon angioplasty most widely used method
o Arterial reconstructive surgery with bypass grafting
Indications for arteriography and assessment for revascularisation;

o Lifestyle-limiting claudication without improvement with conservative


therapies
o Critical limb ischaemia symptoms (rest pain, gangrene, non-healing
wounds)
o Acute limb ischaemia
- Acute limb ischaemia;
Emergency vascular study with ABI or Doppler US
If PVD found, should be immediately treated with antiplatelet (aspirin,
clopidogrel) and assessed for aetiology of ischaemia (embolism, progressive
PVD with thrombus, popliteal cyst, trauma etc.)
Non-viable limb;
o Tissue loss, nerve damage, sensory loss
o Requires amputation
Viable limb;
o None of the above features
o Urgent arteriography and revascularisation
o Localized intra-arterial infusion of thrombolytics (urokinase)
- Chronic severe limb ischaemia (critical ischaemia);
IV drug therapies e.g. prostacyclin
Lumbar sympathectomy
Revascularisation
Amputation if revascularisation not viable
Terminal pain relief

Complications:
- Gangrene
- Functional deficit, sensory loss
- Permanent limb pain
- Amputation
- Complications of arterial surgery;
Haemorrhage
Embolism to limb or renal vessels
Thrombosis of reconstructed
Infection
vessels/graft acute limb
False aneurysm formation
ischaemia

Prognosis:
- Asymptomatic;
All patients with PVD have increased risk of CV ischaemic events
20-60% increased risk of MI, 2-6x increased risk of death due to CAD, 40%
increased risk of stroke
- Intermittent claudication;
Usually remains stable and does not rapidly worsen
Increased risk of chronic limb ischaemia with DM or significantly reduced ABI
- Critical limb ischaemia;
1-year mortality = 25-45%
Poor prognosis unless revascularisation is performed
- Acute limb ischaemia prognosis depends on speed and completeness of
revascularisation

RENAL COLIC

Ross Bob is a 22 year old male who presents with a 4 hour history of
excruciating lower back pain. You are the intern at the RAH ED.
Take a history from Ross
Provide differentials
Suggest management for the most likely differential

Profile
Ross Bob
22 years old
Student
Lives at home with parents
PC
Severe back pain starting a few hours ago
HxPC
Was eating dinner after football training and had sudden onset of lower back
pain

o Loin pain
o Dull ache
o Constant pain that becomes more severe in waves
o Radiates to groin
o Rates it at 8/10
o Worse on movement, better when lying still
o Never had this before
o Hasnt taken any pain relief
No haematuria
Has been feeling nauseous since pain onset and vomiting just before coming to
the RAH
No fever
No difficulty urinating, no urgency, frequency, nocturia, dysuria
No offensive smell of urine
No urethral discharge
No recent sore throat/flu like illness
Probably doesnt drink enough water, diet is good, exercises often
MHx
No allergies
No medications
Previously well
No hospitalisations
FHx
No renal disease
No significant family history
Mum: 54, healthy
Dad 60, healthy
SHx
Social smoker
Binge drinks on weekends and after footy matches
Uni student, studying human movement
Doesnt work, gets centrelink

RENAL STONE DISEASE


Renal stones consist of crystal aggregates. Stones form in collecting ducts and
may be deposited anywhere from the urethra to the pelvis.
The four narrowest points of the urinary tract, and hence the likely places for
stones to lodge are:
Pelviureteric Junction
Pelvic Brim
Under the vas deferens/broad ligament
Vesicoureteric junction
Incidence
2-3% prevalence (Toronto), oxford says 15% lifetime incidence
Peak age 20-40 years
Male: female = 3:`1
Type of Stones
Calcium oxalate (75-85%)

Magnesium ammonium phosphate (10-20%)


Urate (5-10%)
Hydroxyapatite (5%)
Pathogenesis
Supersaturation of stone constituents
Stasis, low urine flow (dehydration)
These lead to crystal formation/stone nidus
Risk Factors
Herediatry
o G6PD, Cystinuria
Dietary Excess
o Vitmain C, oxalate, purine, hypercalcaemia/hypercalciuiria
Renal tubular acidosis
Dehydration
Sedentary lifestyle
UTI
o With urea splitting organism
o Recurrent UTI
Urinary tract abnormalities
o Eg hydronephrosis, PUJ obstruction, horseshoe kidney, VUR, ureteral
stricture
Myeloproliferative disorders
IBD
Hypercalcaemia disroders
o Hyperparathyroidism
o Sarcoidosis
Clinical Features
Can be asymptomatic
Urinary obstruction upstream distension pain
o Flank pain from renal capsular distension (non-colicky)
o Severe waxing and waning pain radiating from flank to groin, testes, or tip
of penis (colicky)
o loin to groin pain
o Obstruction of mid ureter may mimic appendicitis/diverticulitis
o Obstruction in urethra pelvic pain, dysuria, desire but inability to void
Other
o Writhing on the bed during
o Vomiting
history/examinaitoin
o Haematuria (90%
o Never comfortable
microscopic)
o Better on lying still, worse
o Diaphoresis
on moving
o Tachycardia
o Nausea
o Tachypnoea
There are occasionally symptoms of bladder irritation (frequency, urgency)
Can have co-existing UTI or pyelonephritis
o Fevers, rigors, loin pain, nausea, vomiting
Haematuria
Proteinuria
Sterile pyuria
Anuria
Investigations

CBE
o Elevated WCC in presence of fever suggests infection and requires urgent
management
EUC
BGL
Bicarbonate
Urate
Urinalysis/dipstick
o Usually Positive for blood
o pH
o MCS
Imaging
o Abdo xray
80% of stones are visible (calcium, struvite)
20% are not (urate, indinavir)
o CT if stones not visible
99% are visible on CT
Calcium, struvite, cystine, uric acid
Spiral non-contrast CT is superior, particularly in differentiating
differentials of acute abdomen
o Abdo ultrasound
May demonstrate stone in ureter and hydronephrosis
o Intravenous pyelogram
Not usually done
For anatomical defects
Cystoscopy for suspected bladder stone
If recurrent stone formers, conduct metabolic studies:
o Electrolytes, Ca, PO4, uric acid, creatinine, urea
o PTH if hypercalcaemic
Indications for Admission to Hospital
Intractable pain
Intractable vomiting
Fever
o If septic, urgent ureteric stent or percutaneous nephrostomy should be
considered
Compromised renal function
Single kidney with ureteral obstruction/bilateral obstructing stones
Management - Acute
Medical:
o Analgesia
Eg, diclofenac or opioids
NSAIDs lower intra-ureteral pressure
o IV fluids if unable to tolerate PO and vomiting
NB IV fluids do not promote passage of stone
o Antiemetic
o Increase fluid intake to increase urine volume to >2L/day
o Alpha blockers increase the spontaneous passage in distal ureteral stones,
or in stones >5mm
o IV Abx for UTI
Interventional

o If the obstruction endangers the patient (eg, sepsis, renal failure, large
stone >5mm)
Ureteric stent (1.5-2.5cm stone)
Percutaneous nephrostomy
Extracorporeal shockwave lithotripsy (<2.5cm stone)
ESWL
US waves shatter the stone
Prevention
Dietary modification
o Increase fluid intake
o Normal Ca intake
o Avoid high dose vitamin C supplements
Medications
o Thiazide diuretics for hypercalciuria
o Allopurinol for hyperuricosuria
o Potassium citrate for hypocitraturia
Differentials
Renal colic with stone
UTI/pyelonephritis
Trauma to kidney
Ruptured AAA
MSK pain
AMI
Acute pancreatitis
Ischaemic bowel
Diverticular disease
Shingles

RETINAL DETACHMENT

OSCE Station (6 minutes)

You are a 7 foot tall Hispanic GP working in a bulk bill clinic in


Adelaide. Its near the end of the day and they squeezed in one last
patient at 5:15pm.

A 22 year old female, comes in complaining of visual disturbances.

Task:
1 Please take a history from her and
2 in the last minute, discuss the differentials and management plan.

Iris Jones
30 year old flautist
Lives with partner

PC: I cant see properly anymore


HPC:
Started about 3-4 hours ago
Only in my right eye
Lots of lights, like someone is taking flash photographs of me
Everything has become slightly cloudy, and now its started getting wavy as well
No pain
No redness, discharge or tearing
Got scared driving here, it became worse, only used my left eye to see things
Im worried Im going blind
(only notice if asked) There are specks of things in my vision, seem to be
floating
(if asked) a shadow over the upper outer quadrant
(if asked) was playing paintball last weekend with friends and was shot in the
eye, but was wearing goggles
No fever
Never happened before
PMHx:
Must wear glasses/contacts and has been told she has very bad vision
Otherwise healthy
No previous surgeries
On the OCP and multivitamins
Nil allergies
SHx:
Smoker, 10 per day for 3 years
Drinks alcohol, weekend stuff
No illicits

FHx:
Dad has glaucoma
Mum has diabetes
Only child

What would you like to do next for this patient in the clinic?

What will be your management for her?

What do you think the diagnosis is?

Marking criteria
Introduces self appropriately
Gains consent for interview and basic patient data
Determines symptoms
Determines timeline
Asks about previous visual state
Illicit history of trauma
Addresses patients fear of blindness
Recognise this as an emergency retinal detachment

RETINAL DETACHMENT
Produces symptoms of:
o Sudden onset
o Floaters
Due to drops of blood from torn vessels bleeding into the vitreous
o Flashing lights
Due to mechanical stimulation of the retinal photoreceptors
o Visual scotoma in the peripheral field corresponding to the site of
detachment
o +/- RAPD
When light is shone into the affected pupil there will be an
unchanged level of constriction followed by dilation to a larger size
If the fovea is involved, there will be decreased visual acuity
Patients with a history of the following have a risk of retinal detachment
o Myopia
o Trauma
o Prior cataract extraction

SCAPHOID FRACTURE
History & Exam

A 45 year old cabinet maker fell on to his outstretched left had 2


weeks ago while playing basketball. He went to Elsewhere General
Hospital and had an x-ray and was diagnosed with a wrist strain and
told to strap it. 2 weeks later he is still getting wrist pain and has
noticed that his grip is weak and that wrist movements are painful.

Please take a quick focused history about the fall and the injury, do a
physical exam, interpret the x-ray and then decide on management.

ANSWERS

SCAPHOID FRACTURES
Scaphoid fractures account for 75% of all carpal fractures, although they are
rare in the elderly and in children.

The combination of forced carpal movement and compression, such as occurs in


a fall onto an outstretched hand, exerts stress on the Scaphoid and makes it
liable to fracture.
Fall onto an outstretched hand is the classic presentation for a scaphoid fracture
This is with extreme dorsiflexion of the wrist and radial deviation of the wrist
The majority of Scaphoid fractures are stable, however in unstable fractures, the
fragments can become displaced.
The Scaphoid is supplied by an end-artery, and its blood supply diminished
proximally so is put at risk of avascular necrosis if union does not occur. This
accounts for non-union or AVN in:
1% of distal 1/3 fractures
20% of middle 1/3 fractures
40% of proximal 1/3 fractures
Clinical Features
The appearance of the wrist can be deceptively normal. Occasionally, there is
fullness in the anatomical snuffbox and tenderness here to palpation. Proximal
pressure along the axis of the thumb may also be painful.
Symptoms
History of a fall onto an outstretched hand
Deep, dull pain in the radial wrist
Pain is worsened by gripping or squeezing to make a fist

Signs
There may be a wrist effusion and bruising
Possibly fullness in the anatomical snuff box
90% sensitivity of anatomical snuffbox tenderness
Tenderness of the scaphoid tubercle
o Extend the patients wrist with one hand and apply pressure to the
tuberosity at the proximal wrist crease
o Less sensitive but more specific
Absence of pain in these areas makes scaphoid fracture unlikely
Radiography
AP, lateral and oblique views are all necessary. Scaphoid view is also necessary
(wrist is ulnarly deviated and extended whilst the film is shot from a dorsalvolar
angle).
Usually, the fracture line is transverse and runs through the waist of the
Scaphoid the narrowest part.
Scaphoid fractures are commonly missed, and in 30% of cases the fracture is
not visible. If no fracture is visible, it is advisable to splint the wrist and re-xray
within 10-14 days, when the fracture can become more visible due to the
healing process.

If the fracture is non-displaced, a cast is placed from the upper elbow to just
short of the MTP joints. The wrist is held dorsiflexed and the thumb forwards in
the glass holding position. The plaster is retained for 8 weeks.
If the fracture is displaced, open fixation is appropriate
Complications
AVN
The proximal segment may die, especially with proximal pole fractures
At 2-3 months it will appear dense on XRAY

Nonunion
By 3 months it will be evident that the fracture will not unite

Osteoarthritis
Can be secondary to AVN or non-union

SHORTNESS OF BREATH 1

Instruction to Students:

This patient has a respiratory reason for shortness of breath, cardiac


causes are excluded. Please take a detailed respiratory system
history.

HISTORY
Profile
Miss Jane Hollow
23 y/o
Arts student
Lives with roommate
BMI of 21
PC
I think I am allergic to my dog
HxPC
Becoming increasingly short of breath and coughing since getting dog around 1
year ago
SOB
o Wasnt feeling SOB or coughing before hand
o Normally quite fit and active, exercises often
o Finding that she is less and less able to complete her normal 5km jogs as
she has started struggling to breath
o Now getting short of breath just walking up a flight of stairs
o Getting worse
o Feels quite anxious when it happens as it feels as though she is unable to
get air into her lungs
o Has recently started waking up coughing early in the morning and slightly
struggling to breath
2-3 times per week
Started around 3 months ago
o Worse after smoking

Associated chest tightness


Coughs sporadically over the day, worsens at night
o Productive of white sputum
Atopy
o Hayfever during the spring/summer
o Eczema as a child but grew out of it
o Asthma as a child but hasnt used her puffer for years
No recent fevers, no chest pain, no yellow sputum, no haemoptysis, no weight
loss
No recent travel or long periods of immobility
MHx
Allergic to peanuts
o Throat swells up
Nil medications
o Has an implanon
Nil medical conditions
FHx
Not sure of anyone in family with asthma
Nil medical conditions in family
SHx
Smoker
o Cigarettes and marijuana
o 5 cigarettes per day, more on weekends
o Marijuana a few times a month, not too often
Drinks alcohol
o Only on weekends
o Bottle of wine
Nil other illicit drugs

ASTHMA

In simple terms, asthma can be defined as chronic airway inflammation with


reversible bronchoconstriction.
It affects approximately 10-12% of adults and 15% of children
Risk Factors
Endogenous Factors
o Atopy
Atopic eczema and hayfever
Allergic rhinitis can be found in over 80% of asthmatics
o Personal history
Ie, childhood asthma
o Family history
o Childhood viral infections
o Obesity (unknown mechanism)
Environmental factors
o Indoor allergens
o Outdoor allergens
Air pollution
o Occupational exposure
Affects up to 10% of young adults
Triggers
Allergens
o Perennial
Chronic low grade symptoms
Present all year round

Animals, cockroaches
o Seasonal
Grass pollens, tree pollens
Thunderstorms (disruption of pollen grains)
Cigarette smoke
o And marijuana smoke
Viral exposures
o Rhinovirus, respiratory syncytial virus and coronavirus
Drugs
o NSAIDs
o Beta-blockers
Exercise
o Typically begins after the exercise has ended and resolves within 30
minutes
o Worse in cold climates
Physical Factors
o Laughter potentially a trigger
o Cold air, hyperventilation
Occupational Factors
Hormonal factors
o Some women show premenstrual worsening of asthma
Stress
GORD (it is thought that there is a link between GORD and asthma)
o Asthma symptoms do not tend to improve with treatment of GORD

Clinical Features
Symptoms
Wheezing
Dyspnoea
Coughing
o Dry cough or productive of small amounts of white/clear sputum
Nocturnal worsening or waking at early hours of the morning with symptoms
Patient may report feelings of difficulty in filling lungs with air
Patient may report prodromal symptoms before onset of SOB, eg
o Itching under the chin
o Discomfort between the scapulae
o Inexplicable fear (impending doom)
Signs
Tachypnoea
Audible wheeze
Hyperinflated chest
Hyper-resonant percussion note
Diminished air entry
Widespread polyphonic wheeze
Important history questions
PRECIPITANTS
o Cold air?
o Exercise?

o Emotion?

o Allergens? House dust, pets,


pollen
o Infection?

o Smoking and passive


smoking?
o Pollution
o NSAIDs
o Beta-blockers

DIURNAL VARIATION
o Variation in symptoms
o Nocturnal or early hours of the morning worsening
EXERCISE
o Quantify exercise tolerance
How much exercise can you do before becoming short of breath?
Is this different to what you were previously able to do?
How long does the SOB take to relieve itself after exercise
DISTURBED SLEEP
o Quantify nights per week of disturbed sleep
o Whether this has changed recently (severity of asthma)
Acid reflux
o 40-60% of those with asthma have acid reflux
o Treating it improves spirometry but not necessarily symptoms
OTHER ATOPIC DISEASES
o Eczema
o Hay fever
o Allergies
o Family history
HOME ENVIRONMENT
o Pets
o Carpet
o Feather pillows or quilt
o Floor cushions
o soft furnishings
JOB
o Symptom remission at weekends or on holidyas
o Paint sprayers, food processors, welders, animal handlers
DAYS OFF FROM WORK/SCHOOL
MEDICATION
o What medication
o How often it is being used
o Has this changed
DIFFERENTIAL DIAGNOSES
ASTHMA
Viral infection
Pulmonary oedema
Airway obstruction
o Foreign body, tumour

INVESTIGATIONS
In ED setting:
Peak expiratory flow
CBE, EUC, CRP, Blood Cultures

SVC obstruction
Pneumothorax
PE
Bronchiectasis

ABG
CXR
Sputum culture

IN GP SETTING
Lung Function
This is important in making an initial diagnosis and as a means of evaluating
response to therapy
Accurate measurements requires:
o Stopping Long Acting Beta Agonists (LABA) for 12 hours prior to testing
o Stopping Short Acting Beta Agnonists 6 hours prior to testing
The results of spirometry can be used to determine the following:
o Whether baseline airflow limitation is present (obstruction)
o Assess the reversibility of the obstruction with bronchodilator
o Characterise the severity of airflow limitation
o For patients with a normal FEV1/FVC, identify a restrictive pattern as an
alternate explanation for airflow restriction
Airflow obstruction is defined as a reduction of FEV1/FVC
o Obstruction is present if airflow is <0.7
o Considered reversible if there is an improvement of at least 12% and
200ml after bronchodilator
Repeat spirometry 10-15 minutes after BD
o False negatives can occur if:
There was inadequate inhalation of the BD
Recent use of quick acting BD or other anti-asthmatic meds
Minimal airflow obstruction at time of testing (peak almost 100%)
Presence of irreversible airway scarring (remodeling)

Peak Expiratory Flow


Can be useful in patients with previously diagnosed asthma in testing their
response to different allergens
Measurements are taken three times
Bronchoprovocation Testing
Typically used in patients with an atypical presentation (normal baseline
spirometry, isolated symptoms of asthma cough esp).
Can use mannitol or metacholine

Exhaled Nitric Oxide


Biomarkers
The presence of sputum eosinophillia provides useful information about the
asthma phenotype

Allergy Testing
Not useful for diagnostic purposes
Used to identiy triggers and therefore, what to avoid

Imaging
CXR is almost always normal in patients with asthma

TREATMENT

Asthma treatment can be divided into:


Acute
Short term reliever
Long term preventer

Intermittent Asthma
Short acting beta agonist alone is the current recommendation
In patients with intermittent asthma, the addition of a preventer has no proven
benefit

Mild/Moderate/Severe Asthma
Most adults with asthma will require preventer therapy as well as a short acting
bronchodilator
Treatment with an inhaled corticosteroid (ICS) should be considered for patients
with any of the following:
o exacerbations of asthma in the last two years
o use of SABA reliever three times a week or more
o asthma symptoms three times a week or more
o waking at least one night per week due to asthma symptoms
o impaired lung function

Table 2. Classification of asthma severity in a patient with treated


asthma
Colour key

Increase treatment and reassess severity within 3 months

If patient's asthma has matched this category for 3 months


and is stable, consider down-titration of medications and
reassess within 3 months. (For information on adjusting the
medication regimen, see Principles of Drug Therapy)

Clinical features
and lung function

Daytime symptoms
occur less than once per
week

Night-time
symptoms occur less than
twice per month

Daily treatment requirement (includes prn


SABA)

No
inha
led
ICS

Exacerbations are

brief

FEV1 between
episodes is at least 80%
predicted and 90%
personal best.

Daytime symptoms
more than once per week
but not every day

Night-time
symptoms more than
twice per month but not
weekly
FEV1 between
episodes is more than
80% predicted and 90%
personal best.

Any of:
Daytime symptoms

daily

Night-time
symptoms at least weekly

Exacerbations
affect sleep/activity

Lo
wdos
e
ICS

Mil
d
per
sist
ent

Inter
mitte
nt

Lowto
mediu
mdose
ICS
and
LABA

Moder
ate
persist
ent

Highdose
ICS +
LABA
other
agent
s

Severe
persist
ent

Severe
persist
ent

Severe
persist
ent

Severe
persist
ent

Severe
persist
ent

Any of:

SABA use daily

Mild
persi
stent

Mod
erate
persi
stent

Mo
der
ate
per
sist
ent

Mo
der
ate
per
sist
ent

FEV1 is 60-80%
predicted and 70-90%
personal best.

Any of:

Daytime symptoms
every day and restrict
physical activity

Night-time
symptoms frequent

Exacerbations are
frequent

Daily treatment requirement (includes prn


SABA)

FEV1 is less than


60% predicted and less
than 70% personal best.

Seve
re
persi
stent

Severe
persist
ent

Severe
persist
ent

Adapted from GINA 20051


Asthma severity in a patient with treated asthma is classified according to
medications, symptoms and spirometry.
Table 3. ICS dose equivalents: what is meant by low, medium and high
daily doses?

D
os
e
le
ve
l

Lo
w

M
ed
iu
m

Sev
ere
per
sist
ent

Hi
gh

CIC*

80-160
mcg

160-320
mcg

320 mcg
and
above

BDPHFA*
*

Daily
ICS
dose

FP**

BUD
**

100200
mcg

100200
mcg

200400
mcg

200400
mcg

200400
mcg

400800
mcg

Over
400
mcg

Over
400
mcg

Over
800
mcg

SHORTNESS OF BREATH 2

Description of task:

Mr John Smith, a 56-year old man has presented to the ED of a busy


city hospital complaining of severe shortness of breath.

Tasks for the student:


Take a history from the patient
Give a likely diagnosis and explain how you would investigate this
How would you manage this patient?

Patient: Mr John Smith


Age: 56 years old
PC: SOB since this afternoon really bad
HPC:
SOB
Started this afternoon as I was walking home
Acute came out of nowhere
Getting worse
Nothing making it better
Pleuritic chest pain sharp, worse on right side and when I cough/take a deep
breath
Cough with haemoptysis
Now starting to feel dizzy feel I might faint
Feeling hot and cold
Pain in L calf for past couple of days still there, swollen and red
Otherwise have been feeling well no other new symptoms as of today
No recent long-distance travel
No recent operations
No recent trauma

Nothing like this has ever happened before


PMH:
- Colon cancer diagnosed last year
Currently receiving chemotherapy
Been told prognosis is not good
- No previous DVT/PE
- Hypertension on B blocker
No other meds
No allergies
Smoking: Smoking 10 cigs per day, have been since age 21
Alcohol: Drinks socially on weekends, trying to cut down because of the cancer

DIAGNOSIS: Pulmonary embolism


INVESTIGATIONS:
Order of investigations is based on Wells criteria
- Scoring system to determine the probability of a PE before preceding to
investigations
- Criteria;
Clinical features of DVT = 3pts
Other diagnosis less likely than PE = 3pts
Heart rate > 100bpm = 1.5 pts
Immobilisation or surgery in past 4 weeks = 1.5pts
Previous DVT or PE = 1.5 pts
Haemoptysis = 1pt
Malignancy = 1pt
Interpretation: probability of PE (simplified)
>4 = low risk D-dimer first
<4 = high risk CTPA first
This patient has an active malignancy, clinical features of DVT and other
diagnoses are less likely than PE CTPA first
Also will require Doppler US of legs
Other investigations;
Bloods CBE, biochem, coags, troponin
ECG may be normal or show;
o Tachycardia
o RBBB
o Right ventricular strain (inverted T in V1 to V4)
o S in 1, Q in III and inverted T in III (classical pattern rare)
CXR usually normal
ABG respiratory alkalosis

MANAGEMENT:
- Supportive;
100% O2
IV morphine with antiemetic if patient is in pain
Compression stockings to prevent further emboli
- Consider immediate thrombolysis with alteplase bolus indicated if;
Massive PE
Evidence of haemodynamic compromise
High risk patients who do not have hypotension and are at low risk for bleeding
- If thrombolysis not given, commence anticoagulation;
Begin with LMW heparin IV
Subsequently commence IV warfarin if BP stable give heparin and warfarin
together for at least 5 days, and then stop heparin when INR >2
- Duration of anticoagulation with warfarin;
If obvious remedial cause, give for 6 weeks
If no obvious cause, give for at least 3-6 months
Longer duration required if recurrent emboli or underlying malignancy
- Non-pharmacological;
Vena cava filter if patient continues to have emboli despite adequate
anticoagulation, or if patient has absolute contraindication to anticoagulation
Embolectomy indicated in patients with massive PE who cannot have
fibrinolysis

FRACTURED NOF

A 75 year old woman who lives alone and is fully independent,


stumbles, feels something crack in her leg, and falls to the ground
unable to get up due to pain in her groin and thigh.
She is brought to hospital, seen and x-rayed by the casualty officer,
and you, the orthopaedic resident, are called to admit her.

You will be asked to inform the examiner of the following:


1) the key features from the history that you should elicit and which
are essential to the management of this patient
2) the physical examination findings you would expect with this
injury
3) the x-ray appearances (x-ray will be provided)

4) the further investigation and treatment of this injury that you as


the resident would be expected to undertake
5) a brief summary of the case that you would give to the
orthopaedic registrar over the phone
6) the information you would provide to the womans family
regarding the womans prognosis following this injury
7) the likely surgical treatment required
8) the after care, both in hospital and after discharge, including
information regarding possible complications
There is no patient or surrogate patient and you will NOT be required
to take a history or perform an examination.

HISTORY
Alice Potter
75 y/o female
Lives alone

Fully independent in mobility


BMI 23

HxPC:
Fell whilst gardening
o Tripped over the hose
Excruciating pain in groin and thigh
Unable to get herself up
Called for help, neighbour heard her cries within a few minutes and went over to
see what was happening
Called an ambulance
No history of falls

MHx
Hypertension
Diabetes

Taking medications for these but cant remember what they are
Takes calcium OTC for bones as she heard on TV that it was good
Nil allergies
Otherwise well
Nil FHx
Has not had a DEXA, doesnt know if osteoporotic

SHx
Lives alone
Supportive daughter who visits every second day and speaks on the phone to
her daily
Prepares own meals
Smoker 5/day last 20 years
Nil alcohol
Last meal around 5 hours ago, at lunch time
o Ham and cheese sandwich
o Had tea with milk around 2 hours ago, right before beginning gardening

KEY HISTORY FEATURES


Mechanism of fall
o Low or high impact
Presence of risk factors
o
o
o
o

ANSWERS

Osteoporosis/osteopaenia
Age over 65
History of previous falls
Female sex

o
o
o
o

low BMI
Alcohol intake
Vision
Low physical activity levels

Ability to weight bear


Pain
Ability to move leg
Medical history
o High vs low risk
o Comorbidities
o Medications (eg, warfarin)
o Smoker vs non-smoker
Last meal

Fasting Guidelines Prior to Surgery (Canadian Anesthesiologists' Society)


8 hours after a meal that includes meat, fried or fatty foods
6 hours after a light meal (such as toast, crackers and dear fluid) or after
ingestion of infant
formula or nonhuman milk
4 hours after ingestion of breast milk or jelly
2 hours after clear fluids (water, black coffee. tea. carbonated beverages, juice
without pulp)
2. Physical Exam Findings
Shortened, externally rotated left hip
Pain on movement

+/- bruising
3. XRAY Appearance
This is a plain XRAY of Ms Alice Potter, taken on an unknown date at an unknown
time, including AP views of the pelvis in a skeletally mature individual
There is a subcapital fracture of the left femoral neck which is complete and
displaced
This is a Garden IV fracture

Garden
Garden
Garden
Garden

stage
stage
stage
stage

I : undisplaced incomplete, including valgus impacted fractures.


II : undisplaced complete
III : complete fracture, incompletely displaced
IV : complete fracture, completely displaced

4. Further Investigations
CBE, EUC, Group and hold
XRAY is enough imaging
In future, consider DEXA

5. Case Summary
6. Information to communicate to family re prognosis
Hip fracture carries approximately a 30% mortality rate within one year
25-75% of community dwelling people may never regain their pre morbid
function
of elderly people who suffer from a hip fracture die within six months of injury.
More than 50% of elderly patients are discharged to a nursing home, and half of
these patients are still in their nursing home a year later
Long rehabilitation period
Difficult
7. The likely surgical treatment required
ORIF
o She is under 80 and reasonably healthy so this should be the
management option
8. Post Operative Care
In hospital
o Rapid ambulation with physiotherapy
o Appropriate analgesia
o VTE prevention
40mg enoxaparin daily
o Rehabilitation referral
On going
o OT assessment
o Rehabilitation
Major complications
o AVN
Incidence ranges from 5-18%
More common in displaced fractures
Can detect on MRI
Referral
o Non-union

ABDOMINAL PAIN

A 25-year old woman presents to her GP with sudden onset of lower


abdominal pain 6 hours ago. She had an episode of pelvic
inflammatory disease at the age of 18 and has been well since. She
is a non-smoker and drinks 6 glasses of wine per week. She has
never been pregnant and is not taking any medication.
Take an additional directed history from the patient and discuss with
the examiner how you would manage this case.

ID:
Fiona Menz
Age 25
Lives with female housemate
Works in advertising
PC: Really bad pain low down in her abdomen
HPC:
- Pain started 6 hours ago;
Low down on right side (RIF), 7/10
Has always been in the same place, not moving
No shoulder tip pain
Getting worse
No relief with Panadol
- Noticed some vaginal bleeding in last few hours
- Has not vomited, not febrile
- Has never had anything like this before
- Sexual history;
Sexually active with regular partner not officially boyfriend/girlfriend, its really
complicated, he doesnt want to commit but he knows Im the one
Sometimes use contraception (condoms) when they can remember
Not taking OCP
Last menstrual period 8 weeks ago, normally very regular
Has never been pregnant

PMH:
- Usually very healthy
- Pelvic inflammatory disease at age 18 got chlamydia as was raging slut
- Nil surgical hx
- Irregular pap smears
- Nil allergies, nil meds
- Non-smoker, drinks 6 glasses of wine per week with her bffs

SUBSEQUENT MANAGEMENT:
- Physical examination;
Vitals tachycardia, orthostatic hypotension
Abdominal pain in RIF, may be adnexal mass
Speculum exam blood in vagina, cervical motion tenderness
- Investigations;
Beta HCG
Transvaginal US
CBE, urinalysis
- Management;

Laparoscopic surgery salpingectomy or salpingotomy


Methotrexate only given to stable patients meeting strict criteria

Ectopic Pregnancy

- Refers to implantation of an embryo outside of the uterine cavity


- Most commonly occurs in the fallopian tube
- Most common cause of maternal death in 1st trimester

Epidemiology:
- Approx. 2% of pregnancies are ectopic
- Incidence increasing, and more common in assisted pregnancies (IVF etc.)
- Despite increasing incidence, mortality is decreasing
- Recurrence risk;
After 1 EP = 10-15% recurrence
After 2 EPs = 25-66% recurrence

Aetiology & Pathophysiology:


- Risk factors;
Older age (>35 years)
Previous ectopic pregnancy
Pelvic inflammatory disease
Tubal surgery
IVF
Smoking
- 50% of cases thought to be due to damage to fallopian tube cilia following PID
- Other possible mechanisms;
Conception late in cycle
Intrinsic abnormality of ferilised ovum
Transmigration of fertilized ovum to contralateral tube
- 85% of ectopic pregnancies occur in the ampulla or isthmus of the fallopian
tube
- Heterotopic pregnancy refers to an ectopic pregnancy plus an intrauterine
pregnancy occurring simultaneously (i.e. twins but one is ectopic)
Uncommon in general population (1 in 4000), but very common in IVF
pregnancies (1 in 100)
- Uncommon locations; (associated with increased morbidity)
Abdomen
Ovary
Cervical
C-section scar

Clinical features:
- RIF pain + vaginal bleeding + positive pregnancy test = ectopic pregnancy
until proven otherwise

- Typical presentation is of acute abdominal pain (usually RLQ/RIF) in a woman


with recent amenorrhoea or confirmed pregnancy with positive home test
Woman may not know she is pregnant
May have referred shoulder tip pain
- Other common features;
Vaginal bleeding
Tachycardia, orthostatic hypotension
Cervical motion tenderness

Investigations:
- Serial beta-HCG range is usually outside that expected for intra-uterine
pregnancy or spontaneous miscarriage but is almost always positive
- Transvaginal ultrasound;
Only definitive if foetal cardiac activity is detected
Specific finding on transvaginal US is a tubal ring
May have non-specific findings;
o Free fluid in pouch of douglas
o Empty uterus
o Non-cystic mass in adnexa or pouch of douglas
- Laparoscopy is both diagnostic and therapeutic

Management:
- Mainstay is surgery (laparoscopic);
Salpingectomy removal of whole fallopian tube
Salpingotomy removal of embryo only, less favourable as there is a risk of
retention of some of the invasive trophoblast cells
- Medical mgmt. methotrexate (67% success rate); reserved for
haemodynamically stable patients who meet strict criteria

Complications:
- Death
- Retention of trophoblast cells
- Infertility
- Recurrence

Prognosis:
- Accounts for 6-9% of maternal deaths during pregnancy (1st trimester)
- 40-60% will be able to become pregnant again following surgery
- Recurrence relatively common

MENOPAUSE

90 second reading time

Mary Smith aged 50 years has had no periods for 18 months and is
severely troubled by hot flushes. Mary takes over the counter
Remifemin (a phytoestrogen) and evening primrose oil but no
prescription medicines. She has had a recent general and pelvic
examination with no abnormalities detected and her recent cervical
smear and mammogram were normal. Her BMI is in the normal
range.
Task
Please:
1. take an appropriate additional history from Mary (4 minutes)
2. then advise her of your diagnosis and plan of management (1
minute)
3. answer any questions she may have (1 minute).
Please note that the person acting as the patient may not have all of
the described physical characteristics

o
o
o
o

HISTORY
Profile
Mary Smith
50 years old
Interior decorator
Lives at home with husband and two teenaged children

PC
o Troubling hot flashes
HxPC
o Hot flashes for past year or so
o Feels like she is suffocating
o Needs to leave what she is doing and go and stand outside/by an open
window
o Very disruptive to her work and life
o Feeling a bit more tired than normal
o Skin a bit more dry
o Not sleeping well
o Having trouble concentrating
o Dyspareunia
o LMP 18 months ago
o Whilst she was in her 20s and 30s, always had a regular 28 day cycle
o Became slightly irregular in her mid 40s
o No bleeding now
o Was on the pill before/between pregnancies
o Does not use contraception now
o Pregnancies all normal
o Easily fell pregnant


o
o
o
o
o
o

Medical History
Appendicectomy at 11
No personal or family history of CVS or thrombotic disease
No history of breast or gynae malignancy
Aunt had breast cancer at 65
Primrose oil and Black Cohosh
Nothing else

o
o
o

Social History
Non smoker
Drinks a few glasses of wine each night
Doesnt get much exercise

ADDITIONAL HISTORY
Menopause symptoms
o
o
o
o
o
o

Hot flushes
mood swings
formication
joint aches and pains
vaginal dryness
GU symptoms including
urinary frequency

Menstrual history
o LMP
o Previous cycle
Length
Regularity
o Recent irregularity (over last few years)
o Recent abnormal vaginal bleeding
Previous pregnancies (?)
Contraceptive history
o OCP use
o Current contraception

o
o
o
o
o

weight gain
diminished libido
hair loss
dry skin
memory errors

Should be offered until 1 year post LMP


Last DEXA
Last PAP smear and Mammogram
Social history
o Smoking
o Alcohol
o Exercise
o Calcium tablets
o Weight
Medical History (incl family history of these)
o Vascular disease
o Thromboembolic disorders
o Recent abnormal vaginal bleeding
o History of breast or gynae carcinoma
HRT contraindications
o History of stroke, DVT, clotting disorder, MI
o Hypertension and BP control (needs to be under control before HRT
initiation)
o Any vaginal bleeding
o Breast/gynae cancers

APPROPRIATE PLAN
o Lifestyle modification
o Diet
o Exercise
o Cut down on alcohol
o Introduce calcium tablets
o Tests
o DEXA
o Vitamin D and calcium levels
o BGL
o HRT and mirena insertion
o Oestrogen and progesterone
o Replacing oestrogen should reduce symptoms

QUESTIONS:
Is HRT Safe? Will I get any side effects?
o Yes, it is generally safe. We give progesterone to prevent endometrial
cancer, which is one of the larger risks
o Side effects:
Breast tenderness
Nausea
Fluid retention
Headache
Irregular bleeding in first 6 months
Will these symptoms ever go away?
o In the majority of women, these symptoms stop within 4-5 years (60%)
o 20% have ongoing symptoms into their 60s and 70s
I havent found the evening primrose very effective but are there any
other complementary therapies that I can try?

o There is no evidence to support that complementary medications are


effective. If you wish to take them, Id like you to discuss it with me prior
to commencement so that we can be sure that they wont interact with
your HRT medication
Ive heard about bio-identical hormones, could they help me?
o No. We advise against the use of bioidentical hormones for a number of
reasons:
There is no evidence to say that they are safe, of an appropriate
quality or are useful
We cannot monitor the levels of hormones that you are receiving
They carry an increased risk of development of endometrial cancer
You can get progesterone creams that claim to counteract this risk,
however the evidence shows that they do not protect the
endometrium at all

MENOPAUSE

Menopause is the final menstrual period but it is a retrospective clinical


diagnosis:
LMP >12 months ago
The average age for a woman to be in menopause is 51 years, with a median
age of 45-55 years. There are factors, however, that can promote early
menopause:
Smoking
Hysterectomy/oophorectomy
Family history of early menopause
Chemotherapy
Perimenopause is the 4-5 years immediately prior to menopause and bleeding
patterns/symptoms can fluctuate widely during this phase.
Menopause occurs as the result of germ cell depletion, and in some women this
can occur prematurely. In these situations, HRT is recommended until the age of
natural menopause:
Early Menopause
o Age <45 years
o 5% incidence
Premature Ovarian Failure
o Age <40
o 1% before 40, 0.1% before 30, 0.01% before 20
o 4-6 months amenorrhoea with elevated FSH >30 on several occasions
Symptoms
Central
o Hot flushes and night sweats
o Insomnia
o Mood changes
o Memory changes
Musculoskeletal

o Joint aches
o Muscle pain
Urogenital
o Dry vagina
o Dyspareunia
o Urinary frequency
o UTIs
o Incontinence
Skin
o Crawling under the skin
Loss of libido
Long term effects on
o CVS
o Bone
o Brain
Management
It is not appropriate to perform FSH or LH testing
Look at lifestyle and preventative health
o Quit smoking
o Increase exercise
o Decrease alcohol intake
o Weight loss
o Calcium tablets
Update pap smear, mammogram, bone density, BP
Blood tests for cholesterol, blood sugar, Vitamin D, iron, thyroid
Bone density
Contraception
o >50 years up to 1 year post LMP
o <50 years up to 2 years post LMP
HRT

o
o
o
o
o

HRT
This is effective for most symptoms and is safe for healthy women to use for 4-5
years, around the time of menopause (contraindicated >10 years after
menopause, as the damage caused by oestrogen depletion is already done, and
putting the body into a hypercoaguable state after this increases risk of vascular
events)
Indications:
Womans choice
Muscle aches/joint pains
Urogenital/vaginal dryness
Loss of libido
Mood symptoms irritability, sleep disturbance

o
o
o
o
o

Contraindications
>60 years, or >10 years post LMP
Risk of DVT or stroke
Poorly controlled BP
Abnormal vaginal bleeding (investigate this before beginning)
Avoid after breast or gynae cancers

TREATMENT OPTIONS
1. Oestrogen only
Menopausal symptoms are oestrogen deficiency symptoms. As such oestrogen
is the main hormone to be replaced. Otherwise known as unopposed oestrogen
therapy, this is normally given to women who do not have a uterus, as
progesterone is only given to control uterine bleeding and to prevent uterine
cancer. Oestrogen replacement therapy only will increase likelihood of
endometrial hyperplasia and endometrial cancer.
2. Oestrogen and progestogen (cyclical)
Apart from oestrogen, a cyclical progestogen is added for women who are near
menopausal and still menstruating. This is so as to give a more predictable
menstrual cycle than they would get on the next option.
3. Oestrogen and progestogen (combined continuous)
A daily dose of oestrogen and progestogen is given as a continuous regimen.
This is preferred in older women (say 2-4 years post menopause) as there is a
good chance (80% after three months) of no vaginal bleeding with this regimen.
Progestorone is important as endometrial hyperplasia and cancer can occur
after as little as six month of oestrogen only therapy [1].
Another option for combined treatment would be insertion of an intra-uterine
device containing progestogen (eg a Mirena device) combined with oral
oestrogen.
Initiating HRT
o Combined continuous oestrogen and progesterone (need to give continuous
progesterone when >12 months since LMP)
o Daily continuous oestrogen in all HRT
o Start at a low dose
o Can give oral/patch/gel/vaginal. This generally depends on patient
preference
o Transdermal is preferable if there are risks for DVT (smoking, obesity, FHx)
o To give progesterone
o Mirena insertion to protect the endometrium and prevent carcinoma
formation

o
o
o
o
o
o

Side Effects
Breast tenderness
Nausea
Fluid retention
Headache
Irregular bleeding in first 6 months
Weight gain is not a side effect, generally something that just occurs at
menopause

Precautions/Risks
o DVT
o Small increased risk with oral HRT
o Transdermal probably safer
o Stroke
o Small increased risk with oral HRT
o Gall bladder disease
o Heart disease

o No increased risk in healthy women ages 50-59 years


o Breast cancer
o 0.1% increased risk compared to normal population in combined HRT
o Oestrogen only gives no increased risk (in 7 years)

o
o
o

Follow Up
Yearly review and trial lower dose every few years
No maximum time
Patient decision

OTHER TREATMENT OPTIONS


Tibolone (a selective tissue androgen estrogenic activity regulator) is as
effective in alleviating menopausal symptoms and preventing bone loss, has a
greater positive effect on sexual dysfunction and is associated with less vaginal
bleeding. There also may be a role for drugs such as escitalopram (a selective
serotonin reuptake inhibitor) in the alleviation of the frequency and severity of
hot flashes
o Good evidence for venlafaxine and desvenlafaxine
o Also Clonidine, gabapentin

o
o
o
o
o

No evidence for alternative therapies:


Phytoestrogens
Promensil
Remifemin (Black Cohosh)
Evening primrose oil
Chinese herbal medicines (in rare cases, can cause liver failure)

PRE-TERM LABOUR

Alexis Perry, a 28 year old woman comes into the ED in a country


hospital with her husband and 2 year old son. She is 31+2 weeks
pregnant and complains of intermittent abdominal pain. This hospital
is 4 hours away from the city by car. Perform the following tasks:

1. Take an appropriate history from this lady


2. Request for examination findings appropriate to her situation
3. Tell the examiner how you would manage this lady

History
Profile
Alexis Perry
28 years old
Bar tender
G2P1

PC
Intermittent abdominal pain

HxPC
Pain started this morning
o Pain started about 2 hours
after she felt a gush of fluid
whilst lying in bed
o Wet the bed

o Pains occurring every 15


minutes or so
o Getting more frequent
o Cramping pains
o Doesnt hurt to touch her
abdomen

Fluid
o Clear
o No mucus
o No blood
o No offensive discharge
No fever
Not otherwise unwell
No urinary symptoms
Pregnancy has otherwise been uneventful, good fetal movements
Has been attending appropriate antenatal care

Previous pregnancy
o Delivered at 37 weeks
LSCS
Fetal distress
Child now well, 2 years old

MHx
No significant medical history

FHx
Nil significant

SHx
Non smoker
Nil alcohol intake during pregnancy
Nil illicit drugs
o Cocaine, marijuana

Physical Exam
Obs stable, afebrile
Uterine contractions palpable on abdominal exam
o 1 in 10
Speculum
o Pooling in vagina
o 1cm cervical dilation
o Digital exam not appropriate
Laboratory testing
o Ferning on slide
o GBS negative
o WCC and CRP normal
CTG
o Normal fetal activity
o Confirms uterine contraction

Diagnosis
Pre term labour with Premature Preterm Rupture of Membranes

Management
Antibiotic prophylaxis
o No evidence of chorioamnionitis:
Benzylpenicillin for 48 hours or until delivery, whichever is shorter
OR erythromycin orally
o Chorioamnionitis
Amoxicillin + gentamicin + metronidazole
Tocolysis
o Nifedipine
Appropriate if PTL occurring before 34 weeks

Corticosteroids (IM betamethasone) for fetal lung maturation if birth is likely to


occur between 23-35 weeks

o If birth does not occur, re-administer IM betamethasone weekly until birth


occurs
Expectant management until 34 weeks if GBS positive
o Daily assessment of woman
o Clinical observations twice daily
Obs, assessment of uterine activity
Active management if:
o In established labour
o Signs of chorioamnionitis are present
o APH is present
o Signs of fetal compromise
Consider caesarean section if birth is not imminent
Transfer to tertiary centre via ambulance (?)

Definition of Preterm Birth


Birth between 20-37 weeks gestation
If birth occurs before 20 weeks, it is still classified as a miscarriage

Risk Factors
Previous TOP
Lower SES
Smoking
Previous PTL
Inter-pregnancy interval
Congenital anomalies
Chorioamnionitis
PRE-PROM
Abruption
Fetal Demise
Placenta Previa
Poor nutritional status

7 important risk factors are:


Abruptio placenta
UTI
BMI <20
Standing at work >2 hours

Approach to Pre Term Labour


HISTORY
Contractions/abdominal pain
o How long ago did they start
o Frequency and time between contractions
o How long do the contractions last for
o What do they feel like
o Tenderness to touch of abdomen

Uterine anomalies
Advanced maternal age
Age <20
Previous cervical surgery
DES
UTI
Bacterial vaginosis
Polyhydramnios
Serious systemic infection
Multiple gestation
Cocaine
Low pre-pregnancy BMI

Previous PTL
Stress score >5
Previous low birth weight

Has there been any PV fluid loss


o Colour
o Smell
o Amount
o How long ago
PV loss of mucus and/or blood
o Smell
o Colour
o Frequency
Ask about current fetal movements
o >10 per hour
Risk Factors
o UTI symptoms
Dysuria
Polyuria
Frequency
Urgency
o Chorioamnionitis
Fever
Offensive discharge
o Placental abruption
Bleeding
o Pre term/early rupture of membranes
Rupture of membranes occurring anywhere from > 1 hour to 2
weeks before onset of contractions
Considered PPROM if occurring <37 weeks GA
Current pregnancy
o How many weeks
o How has it been going up until now
Previous pregnancies
o How did the pregnancy go? Complications?
o Gestational age at delivery
o Method of delivery
o History of PROM?
PHYSICAL EXAM
Vitals
o Temperature
o Heart rate
o Respiratory rate
Abdominal exam
o Palpation for uterine tenderness
o Palpation for uterine contractions
VAGINAL ASSESSMENT CERVIX
Speculum
o Pooling of fluid in vagina
o Dilation of the cervix
o Cord prolapse
Digital examination
o Do not do if suspecting PROM

LABORATORY TESTS/INVESTIGATIONS
Ferning of vaginal secretions on slide
o Crystalisation occurs if there is rupture of membranes
Bloods
o CBE with WCC and differential
o CRP
High vaginal swabs
o GBS and other microorganisms (not sure which ones)
CTG
o Fetal monitoring
o Uterine contraction monitoring
Ultrasound
o Fetal position and monitoring
Two circumstances in which digital vaginal examination are contra-indicated are:
Suspicion of ruptured membranes
o Introduction of vaginal flora in cervix and lower uterine segment risk of
infection
Bleeding PV
o Risk of APH
Diagnosis
Pre term labour: If there has been spontaneous rupture of membranes and
uterine contractions, with contractions occurring within one hour of membrane
rupture
Preterm rupture of membranes: Membrane rupture without uterine contractions
PPROM and PTL: Pre term rupture of membranes with uterine contractions
beginning > one hour after membrane rupture
Management
PPROM without PTL
Assessment (as above)
Antibiotic prophylaxis
o No evidence of chorioamnionitis:
Benzylpenicillin for 48 hours or until delivery, whichever is shorter
OR erythromycin orally
o Chorioamnionitis
Amoxicillin + gentamicin + metronidazole
No need for tocolytics if contractions are not present
Corticosteroids (IM betamethasone) for fetal lung maturation if birth is likely to
occur between 23-35 weeks
o If birth does not occur, re-administer IM betamethasone weekly until birth
occurs
Expectant management until 34 weeks if GBS positive
o Daily assessment of woman
o Clinical observations twice daily
Obs, assessment of uterine activity
Active management if:
o In established labour
o Signs of chorioamnionitis are present

o APH is present
o Signs of fetal compromise
Consider caesarean section if birth is not imminent
If in a rural centre, transfer to a location with a neonatal HDU (tertiary hospital)
for delivery
PPROM with PTL
Assessment (as above)
Antibiotic prophylaxis
o No evidence of chorioamnionitis:
Benzylpenicillin for 48 hours or until delivery, whichever is shorter
OR erythromycin orally
o Chorioamnionitis
Amoxicillin + gentamicin + metronidazole
Tocolysis
o Nifedipine
Appropriate if PTL occurring before 34 weeks
Other, less favourable options include salbutamol, NSAIDS (only
before 30 weeks gestation)
Generally delays birth by around 48 hours
Corticosteroids (IM betamethasone) for fetal lung maturation if birth is likely to
occur between 23-35 weeks
o If birth does not occur, re-administer IM betamethasone weekly until birth
occurs
Expectant management until 34 weeks if GBS positive
o Daily assessment of woman
o Clinical observations twice daily
Obs, assessment of uterine activity
Active management if:
o In established labour
o Signs of chorioamnionitis are present
o APH is present
o Signs of fetal compromise
Consider caesarean section if birth is not imminent
If in a rural centre, transfer to a location with a neonatal HDU (tertiary hospital)
for delivery
DIFFERENTIALS
Braxton Hicks contractions
o Usually occur no more than once or twice per hour and often just a few
times per day
o Irregular and do not increase in frequency with increasing intensity
o Resolve with ambulation or change in activity

PREGNANCY DATING

You are a resident medical officer in the antenatal clinic of a teaching


hospital.
You are seeing Jane Rush for the first time. She is carrying her
pregnancy record that has been completed by her GP. This records
Janes EDD as 15.05.09. She had a routine mid- trimester ultrasound
scan yesterday.
Jane is upset because, yesterday, the ultrasonographer told her that
her estimated date of delivery is wrong.

SCAN INFORMATION
Scan dated: 17/12/2008
Clinical Summary: Routine morphology scan LMP 08.08.08Scan
summary: Normal morphology. Measurements are consistent with a
gestational age of 16 12 weeks, which is less than the duration
based on the LMP.

Description of task:
Take a history from Jane relevant to her concern and interpret her
ultrasound report so that you can then give her your opinion of her
expected date of delivery.

You DO NOT need to examine Jane.

NB: A pregnancy dating wheel is required for this station

ID: Jane Rush


29 year old
Advertising executive, married to husband of 5 years (happy)
P1G0
PC: I am very upset and concerned because the due date given following my US
doesnt match the one in my pregnancy record. I dont know which one is
correct!
Tasks for the history:
- Menstrual history;
How long/how many days is your normal cycle?
o Patient will tell you cycle is 35 days long
Is your cycle regular? MUST ASK (dating wheel cannot be used if periods are
irregular)
Do you ever have spotting/bleeding between periods?
When was the first day of your LMP?
- Contraceptive history;
What methods of contraception did you use prior to falling pregnant?
Were they used infrequently/sometimes/always?
Have you considered contraception following delivery?
- Address patients anger/concerns;
Unfortunately there is some discrepancy between the dates you have been
given
It is likely this error is due to the assumption that you have a 28 day cycle,
which is the cycle length used on the dating wheel. Your 35 day cycle may not
have been adjusted for
US gave a younger gestational age as she ovulated about a week or more later
than the "usual" 14th day, which was not adjusted for on the dating wheel
Calculate patients new EDD;
Use pregnancy wheel but ADJUST FOR 35 DAY CYCLE
Pregnancy wheel assumed menstrual cycle is always 28 days long, therefore
turn the wheel back 7 days from LMP to adjust for 35 day cycle
o I.e. if LMP is 08/08/13, turn wheel back to 01/08
Therefore EDD is 7th May 2014

Instructions for the patient:


- Appear angry/distressed with discrepancy
- Inform student you have a regular 35 day cycle
Only give details if prompted
- If student incorrectly calculates EDD;
My husband was away on business at that time, that cant be correct
- What day did I ovulate then?

Pregnancy Diagnosis & Dating

- Most reliable methods of detection of early pregnancy;


Home pregnancy test
Transvaginal ultrasound

Home pregnancy tests:


- Available over the counter, cheap
- Sensitivity to 25 IU/L
- Produces two lines;
Control line detects free anti-HCG
Diagnostic line detects HCG-anti-HCG complex
- Very sensitive if properly interpreted, however wrong interpretation occurs
25% of the time when handled by lay-people


Human chorionic gonadotrophin:
- AKA HCG
- Secreted mainly by syncitiotrophoblasts
- Components;
Alpha subunit
Beta subunit
Protein (carbohydrate)
- Functions;
Maintains corpus luteum
Regulates growth factors and cytokines involved in cell proliferation, growth and
differentiation
Clinical use;
o Pregnancy testing/confirmation
o Pregnancy surveillance
o Screening for Down Syndrome
o Tumour marker (trophoblast tumours)
- Levels of HCG gradually increase over first weeks of pregnancy and reach a
peak at 9-12 weeks (25000 288000 IU/L)
Therefore a single level of HCG is not clinically useful, better to observe the
trend
- During the first trimester, levels continue to increase by at least 2/3 (66%)
every 48 hours (called HCG doubling)
If this doubling is observed, intra-uterine pregnancy is likely
Useful in early pregnancy when HCG levels are low, but not useful when
ultrasound findings are clear
- Can be used to guide timing of ultrasound intrauterine sac should be visible
on US when beta-HCG levels are;
1500 IU/L transvaginal
6000 IU/L transabdominal

Pregnancy dating:
- Calculated from first day of last menstrual period (LMP)
Adjusted for length of menstrual cycle
- Pregnancy wheels assumes the menstrual cycle is 28 days long, and ovulation
(and therefore inception) occurred on day 14
Adjustments must therefore be made if the womans cycle is longer or shorter
than 28 days
- Dating is most reliably performed by 1st trimester ultrasound with crown-rump
length (CRL)
- Discrepancy between LMP and 1st trimester CRL;
If LMP date is certain and discrepancy is less than 1 week, dates will not be
changed
Otherwise, CRL is used between 8-13 weeks gestation to date pregnancy

Easy method of dating: take date of LMP, add one week, subtract 3 months and
add a year
e.g. if LMP January 3 2013
Add one week Jan 10

Minus 3 months Oct 10 2013

SALPINGITIS

Instructions to student:

Clinical Case:
Mary Smith is a 24 year old nulliparous woman who was discharged
from the Gynaecology Unit of a public hospital one week ago
following a 3-day admission with acute salpingitis (Chlamydia PCR
positive).
She has been treated with appropriate antibiotics and the hospital
has made arrangements for follow up in six weeks.
Although Marys symptoms are settling she is very upset about her
diagnosis and does not want to wait a further 6 weeks for answers to
some questions.

Requirements of this station:


You will be required to take an additional history and to answer
Marys questions

HISTORY
Profile
Mary Smith
24 years old
Event management
Lives with housemate

PC
Im very upset about being in hospital one week ago with salpingitis and didnt
want to wait for 6 weeks to get my questions answered

HxPC
Around 3 weeks ago started having some lower abdominal pain
o Bilateral
o Difficult to localise
o Getting progressively worse
o Pain during sex (never had this before)
o Some spotting between periods
Also began developing a fever
o Chills
Came to hospital when starting to get very sick

o Looked down there and told me that they thought I had something that
was caused by chlamydia
o Didnt even know she had chlamydia, asymptomatic
o Took antibiotics, starting to feel better
o Told her not to have sex with her partner for a week and hasnt done so
o No changes in discharge, all was normal
Sexual History
o Sexually active from age 17
o Sometimes uses condoms, unless she is drunk and forgets
o Has had 8 sexual partners
3 in relationships
5 were people she picked up whilst out
o All male
o Vaginal sex and oral sex
o Monogamous relationship for almost a year
Doesnt use condoms with current partner
Has an implanon
Had it changed last week
Has been using it for a few years
Has used the OCP in the past when she was young but kept
forgetting to take it

MHx
Appendix out when 6 years old
No medications
No allergies
Otherwise well

SHx
Binge drinks on weekend mostly Friday night drinks
Non-smoker
No illicit drugs

FHx
No significant family history

QUESTIONS
I want to know how I got this. Does this mean that my partner has been
unfaithful?
o Explain that it is an STI, got it from having sexual contact with another
person
o Commonly asymptomatic and long-standing
o Not an indication of cheating
How do I know if I got rid of the infection?
o Good cure rates (>95%) if compliant with antibiotics
o Can re-test in 3 months urine test
Is the infection going to prevent me from having children?
o Normal pregnancy is possible
o Risk of infertility chlamydia is the most commonly associated with
infertility
o Risk of ectopic pregnancies in future

How
o
o
o
o

can I prevent getting the infection again


Get partner tested and treated if necessary
Until treated, use condoms with partner
Abstain from sex for a week following treatment
If you were to get a new partner, use condoms to prevent transmission

SALPINGI
TIS
CHLAMYDI
A
Chlamydia
is the most common STI worldwide.
National Australian prevalence for the young has been estimated at 5.6%.
Contributing factors include:
High rates of asymptomatic carriage
o Up to 80% in females
o Up to 50% in males
Inconsistent condom use
Frequent partner change
Risk factors for infection include:
Adolescent and young adult age group
o Age is the strongest predictive factor
Multiple sex partners, or a partner within three months with other partners, or a
recent new sex partner
Inconsistent use of barrier contraceptives
Clinical evidence of mucopurulent cervicitis
Cervical ectopy
Unmarried
History of prior STD

Low SES or education not beyond high school

Clinical Manifestations
While most infection is asymptomatic, chlamydia can present in a range of
ways:
Cervicitis
o In women, cervical infection is the most common syndrome
o 50% of these are asymptomatic
o Symptoms include:
Vaginal discharge
Post coital bleeding
Intermenstrual bleeding
Poorly localised lower abdominal pain
o Physical examination is often unremarkable, but can including findings of:
Mucopurulent cervical discharge
Cervical friability
Cervical oedema
Dysuria-pyuria syndrome due to cervicitis
o Women can complain of typical UTI symptoms
o Urinalysis shows sterile pyuria
Perihepatitis (Fitzhugh-Curtis Syndrome)
o Occasionally there can be the development of perihepatitis, which is an
inflammation of the liver capsule and adjacent peritoneal surfaces
o It is more commonly seen when there is also PID
o Should be suspected in a person with:
RUQ pain or pleuritic pain in the context of a UTI
There are typically no associated liver enzyme abnormalities
Should be treated with NSAIDs
Pelvic Inflammatory Disease
o Approximately 30% of women with chlamydia will develop PID if untreated
o These women may complain of:
Lower abdominal pain
Vaginal discharge
Dysuria
Constitutional symptoms (eg fever, chills)
o PID due to Chlamydia infection tends to have a higher rate of infertility
than other causes
Pregnancy complications
o Untreated chlamydia in pregnancy can increase risk of premature rupture
of membranes and low birth weight
o If the mother in untreated, 20-50% of babies will develop conjunctivitis if
born vaginally
o 10-20% will develop pneumonia

DIAGNOSIS
NAAT
o Nucleic acid amplification
o Done on first pass urine, or self collected vaginal swab, or endocervical
swab

MANAGEMENT

Azythromycin 1g state
Second line: Doxycyclin 100mg BD for 7 days
Avoid sexual activity for 7 days after treatment
Test of cure:
o Can be performed no earlier than 3 weeks after medication finished
o Cure rates are >95% so these are not usually indicated
Reinfection
o 15-30% reinfection rate in women
Typically associated with resumption of sex with previous partner
(not new partner)
o Test after 3 months?
Discuss contact tracing
o Contact trace 6 months with Chlamydia
o it is important that your partner is treated so that you dont become reinfected
o Explain that chlamydia is often asymptomatic and people dont even know
they have it
Important to mention that it can lay dormant for a number of years
so it does not necessarily mean that their partner is cheating on
them
o Contact tracing can be done by the patient themselves or anonymously by
the GP, web pages sending email notification, nurses in certain practices

PID LONG TERM COMPLICATIONS


Recurrent PID
o Women with a history of PID are at increased risk of disease recurrence
Hydrosalpinx
o After PID resolves, the fallopian tube can become blocked, filled with
sterile fluid and become enlarged
o Associated with tubal factor infertility

o In patients undergoing IVF, hydrosalpinx has a negative effect on


implantation, success of pregnancy, early pregnancy loss, rates of
pregnancy, preterm birth, live delivery
Chronic Pelvic Pain
o Menstrual or non-menstrual pain of at least 6 months in duration that
occurs below the umbilicus and is severe enough to cause functional
disability
o As many as 1/3 of women with PID develop pelvic pain
o Recurrent PID is the strongest risk factor for pain development
Infertility
o Both symptomatic and asymptomatic PID can result in permanent injury to
the fallopian tubes
o Changes to the fallopian tubes include:
Loss of ciliary action
Fibrosis
Occlusion
o The prevalence of infertility after acute PID is dependent upon a number
of risk factors, but seems to be increased several fold
o Chlamydia infection carries the highest rate of infertility
Approximately 1 in 4 women presenting to fertility clinic have a
history of chlamydia infection
Ectopic Pregnancy
o Tubal damage caused by PID leads to an increase in the incidence of
ectopic pregnancy
o In one Swedish cohort study, ratio of intrauterine pregnancy vs ectopic
pregnancy were as follows:
1 episode of PID 1:16
2 episodes of PID 1:6
3 episodes of PID 1:3
Ovarian Cancer
o A number of studies have reported a two fold increase in ovarian cancer
incidence in a patient with PID history
Clinical severity of infection is a poor predictor of long term outcome.

PAEDIATRIC RESPIRATORY HISTORY

28 year old mum has presented to the ED in winter at 10pm, with


her 2 year old boy who has experienced 1-2 days of barking cough,
fever and noisy breathing. This has been especially worse at night.

1. Take an appropriate history from this concerned parent


2. Request for the findings from the examiner when ready
3. Tell her what is the most likely diagnosis and differentials, and how
you will now manage this child

HISTORY
Profile
Max Martin
2 years old
PC
2 days of barking cough, fever and noisy breathing. This has been worse at
night.
Hx PC
1-2 days ago began getting a runny nose
o Fever 38 settled with paracetamol
Gradually worsening
o Developed a cough
o Worse at night
o Barking quality
o Improves after shower
Breathing noisily
o Mostly on breathing in
o No wheeze
o He was getting quite distressed and upset
Not complaining of a sore throat

She does not think he has inhaled anything generally does not let him play
with things small enough to fit into his mouth
Not eating and drinking as much as he would normally
No contact with unwell people
Goes to daycare 3 times per week when mum works

MHx
Nil allergies
Nil previous illnesses
Normal pregnancy and childbirth
No evidence of asthma or atopy

FHx
Nil significant

Examination Findings
Vital Signs
o Fever 38 degrees Celsius
o tachypnoea
o Tachycardic
o 97% saturation room air
General inspection
o Mild respiratory distress
Intercostal indrawing
Tracheal tug
No subcostal indrawing
o Child appears distressed
o Stridor
o Barking cough
o Does not appear dehydrated
o Pharynx erythematous but no tonsillar enlargement. ENT otherwise
normal
Chest clear to auscultation with transmitted upper airway sounds
No further investigations needed can do an NPA for clerical reasons

Diagnosis
Croup
Differentials:
o Acute epiglottitis
o Peritonsillar abscess
o Foreign body aspiration
o Whooping cough

Management
Moderate croup (Stridor at rest, recession on breathing)
o Admit to hospital
o Humidified oxygen
o Nebulized dexamethasone
o IV fluid rehydration maintenance fluids
o Panadol to lower fever

o Continue to monitor 02 saturation and temperature to monitor for


deteriorations

The elements of the Westley croup score describe key features of the physical
examination [36]. Each element is assigned a score, as illustrated below:
Level of consciousness: Normal, including sleep = 0; disoriented = 5
Cyanosis: None = 0; with agitation = 4; at rest = 5
Stridor: None = 0; with agitation = 1; at rest = 2
Air entry: Normal = 0; decreased = 1; markedly decreased = 2
Retractions: None = 0; mild = 1; moderate = 2; severe = 3
Mild croup is defined by a Westley croup score of 2. Typically, these children
have a barking cough and hoarse cry, but no stridor at rest. Children with mild
croup may have stridor when upset or crying (ie, agitated) and either no, or only
mild, chest wall/subcostal retractions [1,32].
Moderate croup is defined by a Westley croup score of 3 to 7. Children with
moderate croup have stridor at rest, at least mild retractions, and may have
other symptoms or signs of respiratory distress, but little or no agitation [1,32].
Severe croup is defined by a Westley croup score of 8. Children with severe
croup have significant stridor at rest, although stridor may decrease with
worsening upper airway obstruction and decreased air entry [1,32]. Retractions
are severe (including indrawing of the sternum) and the child may appear
anxious, agitated, or fatigued. Prompt recognition and treatment of children with
severe croup are paramoun

PAEDIATRIC FEBRILE SEIZURE

You are in the emergency department. Mrs Williams and her baby
Jessica have just been bought in by ambulance. Jessica is 9 months
old and has been a normal healthy child.

Jessica had a turn this morning at home. Her condition is stable


enough for you to take a history. Please take a history from Mrs
Williams.
You can then ask the observing examiner for the examination
findings.

In the last minute explain your diagnosis to Mrs Williams and the
management of Jessica over the next 24 hours. This should include
demonstrating to Mrs Williams using the doll what to do if Jessica has
another turn

You will receive a warning when 5 minutes have passed.

ID: Mrs Sally Williams with baby Jessica


9 months old
Lives at home with husband and son (5 years old)
PC: Jessica had a funny turn this morning at home and now Im really worried

HPC:
- Was changing Jessicas nappy on the changing table this morning (about 2
hours ago) when she had a fit
Went stiff and then her arms and legs started to jerk/shake
Was awake during episode
Lasted 1-2 minutes
Was fine afterwards, back to normal, did not vomit
- Normally healthy, but has been a little unwell for the past few days
Runny nose
Sneezing
Not sleeping well, difficult to settle
Feverish took temperature last night, 38.2
- Has never had a similar episode in the past
PMH:
- Has otherwise been well no problems
- Normal vaginal delivery at 37 weeks
- No complications with pregnancy or delivery
- Vaccinations up to date
FAM:
- Lives with husband and son (age 5)
- Remembers her son had something similar, but occurred at day care so she did
not witness it

EXAMINATION:
- Infant looks normal, but is difficult to settle/crying
- Febrile - 38 degrees
- Tachycardic
- Obs otherwise stable
- ENT;
Nasal congestion, rhinorrhea
Red, watery eyes
Red pharynx, no pus
- No stiffness of neck
- Neurological examination normal
- Chest clear

DIAGNOSIS:
- Febrile seizure/convulsion likely secondary to viral URTI
MANAGEMENT:
- Does not require admission to hospital
- Investigations not necessary likely secondary to viral URTI
- Monitoring of body temperature
- Methods to reduce body temperature;
Antipyretics esp. ibuprofen
Tepid sponging
Remove unnecessary clothing

- Antibiotics not required


- Anticonvulsants not required unless seizure is prolonged (>5-10 mins)
- If another seizure occurs;
Protect child from injury ensure there are no dangers around them, lie flat
Do not restrain infant
Reduce body temperature remove clothes (hats, shoes etc.), open window,
turn down heating
Check ABCs
Time episode
Coma position following seizure hold on their side with head tilted back
Call for help
- Indications for Ix (to rule out meningitis/encephalitis)
Hx of vomiting
Child <6 months
Repeated seizures
Has had ABs
Has not recovered promptly following seizure
Seems more ill than what would be expected

Febrile Seizure

- Defined as a seizure/convulsion occulting between 6 months and 6 years of


age with fever but without evidence of intracranial infection or defined cause
Seizures with fever in children who have experienced a previous non-febrile
seizure are excluded
- Classification;
Simple (70-75%)
Complex (20-25%)
Symptomatic (5%)

Epidemiology:
- Most common seizure disorder in childhood
- Approx. 5% of children <5 years of age have one or more febrile seizures
- Uncommon in children >6 years, peaks between 12-24 months
- More common in boys

Aetiology & Pathophysiology:


- Fever and height of body temperature induced by infection are the essential
factors in aetiology
- Also found to have a genetic component 30% have FHx of febrile seizures
- Other risk factors;
Young age (peaks 12-24 months)
CNS immaturity
Environmental factors
- Causative infections;
Viral most common cause (usually URTI, UTI(
Bacterial infection is an infrequent cause
- Infectious agent is not important in increasing risk of seizure or predicting
future recurrences

Classification:
- Simple febrile seizure;
Most common type (70-75%)
Generalized tonic and/or clonic seizure in setting of fever (>38 degrees) with
normal post-ictal exam
Seizure lasts <15 mins, no sequelae
Child is otherwise neurologically healthy
Fever (and seizure) not due to CNS infection
No Hx of afebrile seizures
- Complex febrile seizure;
20-25% of cases
Age, neuro status before illness and fever as above
This seizure is either prolonged (>15 mins), focal or multiple
- Symptomatic febrile seizure;
5% of cases
Age and fever as above
Occurs in a child with a pre-existing neurological abnormality or acute illness

Investigations:
- Not typically needed as it is a clinical diagnosis, although tests may be useful
in identifying source of fever urinalysis, CXR, NPA etc.
- May need to rule out meningitis/encephalitis LP, blood cultures, viral studies
- Indications for Ix (to rule out meningitis/encephalitis)
Hx of vomiting
Child <6 months
Repeated seizures
Has had ABs
Has not recovered promptly following seizure
Seems more ill than what would be expected

- There is no role for EEG or brain imaging

Management:
- During witnessed seizure;
Protect child from injury ensure there are no dangers around them, lie flat
Do not restrain infant
Reduce body temperature remove clothes (hats, shoes etc.), open window,
turn down heating
Check ABCs
Time episode
Coma position following seizure hold on their side with head tilted back
Call for help
- Most resolve spontaneously ad no anticonvulsant is required, however if
seizure lasts >5 mins, urgent IV or rectal diazepam should be given
- First simple febrile seizure;
Most causes are viral, therefore ABs not required
Reduce body temperature;
o Antipyretics especially ibuprofen
o Tepid sponging
o Remove clothing
Anticonvulsant not given (unless prolonged) do not reduce risk of future
epilepsy
- Some children with recurrent or prolonged (i.e. complex) febrile seizures may
be prescribed prophylactic diazepam or emergency rectal diazepam controversial
- Educate parents and reassure re recurrence, risk of epilepsy in later life, rarity
of neuro problems and management of future febrile seizures

Complications: (all uncommon)


- Todds paralysis transient hemiparesis following seizure; requires neuro
consult + EEG, MRI
- Non-febrile seizures, epilepsy 3%

Prognosis:
- 1/3 of children will have recurrent febrile seizures risk increases to 50% if
onset n infancy or there is a FHx
- 3% go on to have epilepsy in this case it is the manifestation of the same
underlying seizure predisposition (i.e. idiopathic epiliepsy)
- Febrile seizures are not associated with increased mortality or intellectual
impairment

PAEDIATRIC ANAPHYLAXIS

Description of task:

5-year-old Isobella has been brought into the Emergency


Department by SAAS following an anaphylactic reaction that
occurred 1.5 hours ago. SAAS was called after Isobellas mother
noticed her daughter develop an unusual rash and was struggling to
breathe and talk.

On handover from the Ambulance Officer, you find that Isobella was
given 0.01 ml/kg of 1:1000 deep IM adrenaline as well as oxygen at
the scene, and is now currently stable.

Isobella is currently sitting in a cubicle with her mother, Shirley, and


as the intern you have been asked to;

- Take a more detailed history from Shirley


- Explain the diagnosis to Shirley and Isobella
- Outline management plan

ID:
Shirley Williams, 28-year-old mother of two and housewife
Isobella Williams, 5-year-old daughter
Live at home with husband and younger child, Harry (2)
PC: Called the ambulance after Shirley noticed her daughter was developing a
widespread rash and was finding it difficult to breathe and swallow
HPC:
- Occurred about 1.5 hours ago whilst Isobella was eating her lunch
Isobella likes Asian food and was given Pad Thai bought from a shop
Thinks it may have had peanuts sprinkled over it, but not sure
Has avoided giving nuts to children as she heard lots of kids are allergic
- Left room, and when she returned noticed Isobella had developed a rash on
her chest and arms
Red, bumpy, looked like hives
Worsened over 2-3 minutes
- After 5 minutes, noticed Isobella was finding it increasingly difficult to talk and
breathe
Tongue and face looked swollen
Making high-pitched noise when she was breathing in
Gradually got worse until Isobella was clearly distressed
Coughing
- SAAS subsequently called
- Remained conscious throughout
- Has never had a previous episode
PMH:
- Previously generally well
- No known allergies to any drugs, foods or insects
- Some reflux as an infant, some mild asthma symptoms very infrequent
- Normal vaginal delivery, at term
- Vaccinations given as per schedule
- Meeting all milestones, normal growth

1.
2.

FAM:
- Family members all well
- Shirley had asthma as a child

DIAGNOSIS:
- Anaphylactic reaction most likely due to allergy to (pea)nuts
- Explanation;
Isobella has had a severe, acute allergic reaction, most likely to peanuts that
were in the Pad Thai she was eating
Affects multiple body systems, that is why she had the skin rash and respiratory
problems
This is obviously a severe condition, and the risk is that this can happen again if
Isobella is exposed to peanuts again
Thus management plan needs to be formulated!
- But Isobella has never eaten peanuts before. How can she be allergic to
something she has never been exposed to?
So you are correct that allergies occur in people only after they have had an
initial exposure to the allergen (a process called sensitization), but in the case of
childhood food allergies, it is assumed that sensitization occurs during early
exposure to food proteins in breast milk or by allergen skin contact

MANAGEMENT:
- Emergency/Acute mgmt. already performed by SAAS, patient now stable
Remove trigger
Administer adrenaline via deep IM injection
o 0.01 ml/kg of 1:1000 adrenaline (max dose 0.5 ml)
3. Establish airway if required and administer high flow oxygen (100%)
o Mechanical ventilation may be required in case of airway obstruction
4. Assess circulation if hypotensive, administer;
o IV adrenaline dose 0.1 ml/kg of 1:10000 (max dose 3ml)
o IV fluids normal saline 10-20ml/kg as bolus
5. Repeat doses of adrenaline can be administered every 5 minutes until clinical
improvement occurs
- Other therapies to consider;
Nebulized salbutamol recommended if patient is in respiratory distress or is
wheezing
Antihistamines for symptomatic relief of pruritis, second generation preferred
Corticosteroids used mainly for bronchospasm, not commonly used

- The child should be observed for at least 4 hours, and should be admitted
under the following circumstances;
Greater than one dose of adrenaline required
A fluid bolus is required
Inadequate response to treatment
The child lives a long distance from medical services

- Anaphylaxis action plan;


AVOID ALLERGEN

Outlines steps that should be taken if Isobella has a similar episode in the future
Will likely require an adrenaline auto-injector (AAI) e.g. Epipen, Anapen, as
Isobella is at ongoing risk if re-exposed to peanuts
http://www.allergy.org.au/health-professionals/anaphylaxis-resources/asciaaction-plan-for-anaphylaxis
Prescription of AAI is via PBS but requires an authority prescription
Dosage of AAI;
o Weight <10kg not recommended for children <10kg
o Weight 10-20kg AAI 150mcg dose
o Weight >20kg AAI 300mcg dose
- Food allergy testing;
Longer term mgmt., used to confirm food allergy
Methods;
o Serology for allergen specific IgE (ASE)
o Skin prick testing

Anaphylaxis

- An acute, severe, life-threatening allergic reaction in pre-sensitised individuals,


leading to a systemic response caused by the release of immune and
inflammatory mediators from basophils and mast cells
- Must involve at least 2 organ systems and must have;
At least one respiratory or cardiovascular feature and
At least one GI or skin feature
- Most frequently due to allergies to medicines, food, immunotherapy or insect
stings
- Exact epidemiology not known

Aetiology & Pathophysiology:


- Common causes of anaphylaxis in children inc;
Foods (most common cause) nuts, cow milk, eggs, soy, shell-fish, fish, wheat
Bites/stings bee, wasp, ants
Medications beta-lactams, monoclonal antibodies, anaesthetics
Others exercise induced, idiopathic, latex, hydatid cyst rupture, biological fluid
transfusion (blood, antivenom), food additives
- High-risk groups for anaphylaxis;
History of anaphylaxis
Multiple allergy to food and drugs
Poorly controlled asthma
Pre-existing lung disease
- Most reactions occur within 30 minutes of exposure to a trigger
- The clinical symptoms are derived from pro-inflammatory and vasoactive
mediators and cytokines released by massive degranulation of basophils and
mast cells
Classically this cascade is initiated by an IgE-mediated hypersensitivity reaction

Clinical features:
- Respiratory (most common in children);
Tongue swelling
Stridor, dysphasia
Hoarse voice, change in character of cry
Persistent cough
Wheeze
Feeling of swelling/tightness in throat
- Cardiovascular;
Pale, floppy infants
Palpitations
Tachycardia/bradycardia
Hypotension
Cardiac arrest
- GI;
Nausea and vomiting
Diarrhea
Abdominal/pelvic pain
- Mucocutaneous;
Generalized pruritis
Urticarial/intense erythema
Conjunctival erythema and tearing
Flushing
Angio-oedema
- Neurological;
Headache (usually throbbing)
Dizziness
Confusion, altered consciousness
Collapse with or without LOC

Investigations: Clinical diagnosis!

PSYCHIATRY HISTORY

Instruction to Students
You have 6 minutes at this station.

This woman has come to see you because of episodes of severe


shortness of breath associated with palpitations and feelings of
extreme anxiety.
She has been tested for asthma and heart disease and all tests have
been normal.
Please take a focused history aimed at establishing a provisional
diagnosis
Report your provisional diagnosis to the examiner at the end of your
history.

History
Profile
Donna Karen
41 yo
Accountant
Lives at home with husband and teenaged children

PC
Having episodes of severe shortness of breath and heart palpitations. Very
concerned

HxPC
Started 2-3 months ago
At home, lying in bed with husband, started to feel very short of breath
o Heart pounding
o Sweating
o Felt like being suffocated and couldnt catch her breath
o Tingling hands and feet
o Dizziness
o Thought she was going to have a heart attack and die
At first thought it was asthma or a heart problem
o Had asthma as a child, thought it may be coming back
o Has been tested for asthma and cardiac issues all clear but she still
thinks something is wrong and is afraid that she is going to die every time
they happen
Still having attacks
o Tend to only happen in bed when she is trying to sleep
Getting worried about going to bed because she is afraid that they
will happen and she will have a heart attack
Staying up later and later
Reading, watching TV, always feeling very tired
o Very self conscious about them
Triggers:
o Has been having issues with her husband lately
He has started gambling heavily
Losing a lot of money
Arguing a lot
o Finds that she thinks about this a lot when she is trying to sleep. Attacks
tend to happen after this
Otherwise medically well
No other symptoms at any other times

Medical Hx
No medications
Had tubes tied 2 years ago as she wanted no more children
No allergies
Had depression as a teenager, managed with anti depressants
o Has been on and off them throughout 20s

o Hasnt needed them for 15 years or so

FHx
Mother was on tablets for years for her mood swings
o Not really sure what this was about
o Maybe depression

SHx
Married for 25 years
Husband always treated her very well
Now that he has started gambling and spending lots of money, they are arguing
all the time
o Relationship is breaking down
o She is very upset about this
Non smoker
Non drinker
No illicit druGS

Panic Attacks
Panic attacks occur in several different Anxiety Disorders. DSM-5 diagnostic
criteria for a panic attack are:
An abrupt surge* of intense fear or intense discomfort that reaches a peak
within minutes, and during which time four or more of the following 13
symptoms occur:
Palpitations, pounding heart, or accelerated heart rate
Sweating
Trembling or shaking
Sensations of shortness of breath or smothering
Feelings of choking
Chest pain or discomfort
Nausea or abdominal distress
Feeling dizzy, unsteady, light-headed, or faint
Chills or heat sensations
Paresthesias (numbness or tingling sensations)
Derealization (feelings of unreality) or depersonalization (being detached from
oneself).
Fear of losing control or "going crazy"
Fear of dying
* The abrupt surge can occur from a calm state or an anxious state.
Note: Culture-specific symptoms (eg, tinnitus, neck soreness, headache,
uncontrollable screaming or crying) may be seen. Such symptoms should not
count as one of the four required symptoms.
PANIC DISORDER
The cardinal feature of panic disorder is the presence of recurrent, unexpected
panic attacks followed by at least one month of persistent concern about having
another panic attack (cognitive sequelae), worry about the possible implications
or consequences of the panic attack, or a significant behaviour change related
to the attacks (eg, interoceptive and behavioural avoidance). Agoraphobia may
be present or absent.
This behaviour is not due to the physiological effects of a substance or general
medical condition, nor is it better accounted for by another medical condition.
There are three types of panic disorder:
Unexpected
Situationally predisposed
Situationally bound (uncommon)

General Epidemiology
Ratio female:male approximately 2:1
Onset in late adolescence to mid 30s
Onset of Agoraphobia is usually within one year of the onset of panic attacks
Outcome:
o 30% well
o 40% improved but symptomatic
o 20-30% have the same symptoms or slightly worse
Common co-morbid conditions:
o Major depressive disorder (60-90%)
o Social Phobia

o OCD
o Specific Phobia
o GAD

A.
B.

C.

D.

Patients often present to physicians with somatic symptoms, such as cardiac,


GIT or neurological symptoms. There is often a history of alcohol or substance
abuse and attempts at self harm.
There is a resultant impairment of relationships, employment and quality of life.
DSM-V Diagnostic Criteria:
Recurrent unexpected panic attacks
At least one of the attacks has been followed by a month or more of one or both
of the following:
1. Persistent concern or worry about additional panic attacks or their
consequences (eg, losing control, having a heart attack, "going crazy").
2. A significant maladaptive change in behavior related to the attacks (eg,
behaviors designed to avoid having panic attacks, such as avoidance of
exercise or unfamiliar situations).
The disturbance is not attributable to the physiological effects of a substance
(eg, medication or illicit drug) or another medical condition (eg,
hyperthyroidism, cardiopulmonary disorders).
The disturbance is not better explained by another mental disorder. As
examples, the panic attacks do not occur only in response to
Feared social situations, as in social anxiety disorder
Circumscribed phobic objects or situations, as in specific phobia;
Obsessions, as in obsessive-compulsive disorder
Reminders of traumatic events, as in posttraumatic stress disorder
Separation from attachment figures, as in separation anxiety disorder
Differentials for Panic Disorder
Somatisation disorder
Stimulant abuse (eg caffeine, cocaine)
General medical conditions
o
o
o
o

Asthma
Pheochromocytoma
Hyperthyroidism
Angina

o
o
o
o

Cardiac arrhythmias
Temporal lobe epilepsy
COPD
Pulmonary embolism

o Management
Non-Pharmacological
o CBT
o Education about the nature of panic disorders and the physiology of panic
attacks
o Breathing retraining
o Muscle relaxation exercises
o Psychodynamic therapies
o Interpersonal therapy
Address the personal difficulties leading to psychological problems
Pharmacological
o SSRI
o SNRI

o MAOI
o Benzodiazepines
o TCAs
o

o PSYCHIATRY HISTORY DIFFERENTIAL


o

o You have 6 minutes at this station.


Jean Richards is a 72-year old woman who has been admitted to the
ward for further investigation.
Her husband died 9 months ago, and over the last few months she
has not been looking after herself, and has been frequently tearful
Her family report behavioural disturbance and say that she has been
confused and forgetful
Take an appropriate history, with the intention of differentiating
depression from dementing illness. Tell the examiner which diagnosis
you think is more likely.
o You are not expected to complete formal testing of cognition.
o
o
o
o
o
o
o
o
o
o
o
o
o
o
o
o
o
o
o
o
o
o
o
o
o
o
o
o
o
o

o
o
o
o
o
o HISTORY
o Profile
Mrs Jean Richards
72 yo female
Retired
Lives home alone
o PC
My family feels that I havent been looking after myself properly, particularly
since the death of my husband, Barry
o HxPC
Patient is oriented to time, place and person
Is aware that she is in hospital because her family is worried about her
Long term recall childhood - good
Husband died 9 months ago
o Died of lung cancer
o She cared for him at home until he died
o Loved him very much
o Married at 19
o Gets very lonely in the home without him
o Children visit but its not the same
o Misses him a lot and gets very upset when thinking about him
Has felt very sad since her husband died
o Kept to herself
o Stopped going to bowls because it was something that she and Barry did
together and she has now been finding it too hard
o Not as happy/friendly as she was before
o Less interest in her family and their activities finds this quite distressing
Since her husband died, she has found it difficult to keep going with her day to
day tasks
o Before her husbands death, she felt she was doing very well at home
Now feels as if she is having some trouble
Isnt cleaning as much as she used to
o Over the past 3 months has had some memory problems:
Sometimes forgets where she puts her keys
Sometimes forgets conversations she has had with her daughters
Feeling distressed about this as she never had memory troubles
previously
Doesnt notice any variation in this at night or day
o Has had trouble maintaining her appearance when previously this was
something she took pride in
Doesnt get her hair permed anymore because she doesnt see the
point
o Cant really be bothered cooking just for one
o Sometimes forgets to eat altogether, hasnt really had an appetite

o Used to do the crossword


Hasnt really been able to concentrate on them lately
o Hasnt been sleeping well
Waking up early in the morning
Difficulty falling asleep
o Cant stop thinking about her husband
Starting to feel like there is no point living
o Her husband was her soulmate and feels that he was everything
o MHx
Never had any psychiatric illnesses
Heart troubles
o Takes some medications but cant remember what they are called
o The chemist puts them in a special pack for her to take them
o Sometimes skips days because she just forgets
Takes something to help her sleep since husband died
o Temazepam
o Doesnt like taking it all the time but finds that it helps a little
No surgery
Otherwise well
o FHx
2 daughters
o Supportive
o Visit her a few times each week
o Very worried about her
5 grandchildren
o All well
No medical conditions in family
o SHx
Used to play bowls with her husband but has stopped because it reminds her of
him too much
Doesnt drink alcohol or smoke
o
o

o Dementia
Family often unaware of distress
Insidious onset
Longstanding, slow course
Few complaints of sx, often not distressed
Unconcerned about mood
Tries hard on tests, performance consistently poor
Loss of recent memory
Some loss of concentration
Word finding problems
o Depression
Family acutely aware of distress
Onset can often be dated
Short duration, rapid progression
Complaints of memory loss, distress evident
Sad, withdrawn, no interest
Little effort on tests, gives up easily, variable performance
Poor concentration
Vocabulary intact
o Major Depressive Episode
There are four symptoms domains:
o Mood
o Psychomotor activity
o Cognitive
o Vegetative
The DSM IV criteria for MDE is:

o At least 5 of the following occurring nearly every day (except recurrent


thoughts of death or suicide) over at least 2 weeks, which are a change
from previous functioning
Depressed mood most of the day
Loss of interest or pleasure most of the day (one of the symptoms
must be the first or second dotpoint)
or appetite or significant change in weight
Insomnia or hypersomnia
Psychomotor agitation or retardation
Loss of energy
concentration or decisiveness
Feelings or worthlessness or excessive/inappropriate guilt
Recurrent thoughts of death or suicide
o Symptoms must cause clinically significant distress or functional
impairment
o Exclucdes bereavement, substance, general medical condition and mixed
episode

o Major Depressive Disorder


This can be diagnosed if a patient has had one or more MDEs

o
o
o
o
o
o

o
o
o

o PSYCHIATRY HISTORY

o STEM
o This interaction occurs in the Emergency Department (ED) of a
General Hospital.
o You are an ED intern and are the first Medical Officer to see the
patient.
This mid-20s law student is in a distressed state.
He/she has been studying for 2nd Year exams, but has become so
agitated at the prospect of failure that he/she needs to talk to a
professional.
o
o TASK
o Please:
o 1) Obtain a history from the patient in the first 5 minutes,
specifically focussing on the reasons for this presentation,
o
o Then
o
o 2) Tell the examiner the likely conditions that need to be
considered and why.
o
o
o
o
o
o
o
o
o
o
o
o
o
o
o
o
o
o
o
o
o
o
o
o
o

o
o
o
o
o
o
o HISTORY
o PROFILE
Miss Ava Stan
Age 22
2nd year exams for law school
o
o PRESENTING COMPLAINT
Feeling very stressed about exams scared she will fail
Has been having lots of episodes of intense fear over the past 1 month
o Lasts 10mins
o Comes on when she tries to study
o Racing heart, sweating, shaking, SOB, cant breath, nausea, dizzy
Scared to study now because she doesnt want more attacks
Convinced she will fail
o
o PMH
Nil
No substance abuse and no caffeine
No medications
o
o FAMILY HISTORY
Mum bipolar (terrified she is developing her mums condition, thats why she
has come to hospital)
Father depression
No siblings
o
o DIAGNOSIS
panic attacks Panic disorder without agoraphobia (DSM-IV)
Generalized anxiety disorder restlessness, fatigue, poor concentration,
irritable, muscle tension, poor sleep
Specific phobia exams
Medical Thryotoxicosis, pheochromocytoma, caffeine
NOT bipolar no depressive symptoms, no mania (elevated mood)
o
o MANAGEMENT
Psychology CBT, relaxation techniques
Biological SSRI or SNRI
6-10 years prognosis with treatment
o 30% well
o 40-50% improve
o 20-30% no change or worse
o
o
o MARKING
o INTERACTION
o A - empathic, develops rapport, appropriate introduction, appears
unhurried, fluent and efficient

o
o HISTORY
o A - Efficient and directed. Comprehensively covers symptoms, habits and
substance use, family history, past history.
o
o DIAGNOSIS
o A - Expressly and without prompting recognises the thought and
behavioural anomalies. Clearly recognises this is NOT xxxxxxx.
o
o MANAGEMENT
o A - Recommends referral to appropriately trained mental health
professional for (insert appropriate treatment options). Allows patient
choice of options for referral if appropriate. No prompting required
o
o COUNSELLING
o A - Clearly indicates without prompting this is not xxxxxxx and that the
patient does not have and is not expected to develop the same mental
health problem as his mother
o
o
o DSM 1V FOR PANIC DISORDER
o A. both (1) and (2)
1) recurrent unexpected panic attacks: a discrete period of intense fear or
discomfort, in which 4 of the following symptoms develop abruptly and reach a
peak within 10 minutes
o palpitations, pounding heart, or accelerated heart rate
o sweating
o trembling or shaking
o sensations of shortness of breath or smothering
o feeling of choking
o chest pain or discomfort
o nausea or abdominal distress
o feeling dizzy, unsteady, lightheaded, or faint
o derealization (feelings of unreality) or depersonalization (being detached
from oneself)
o fear of losing control or going crazy
o fear of dying
o paresthesias (numbness or tingling sensations}, chills or hot flushes
2) at least one of the attacks has been followed by 1 month (or more) of the
following
o persistent concern about having additional attacks
o worry about the implications of the attack or its consequences (e.g. losing
control, having a heart attack, "going crazy")
o a significant change in behavior related to the attacks
o
o B. absence of agoraphobia
o
o C. the panic attacks are not due to the direct physiological effects of a
substance or GMC
o
o D. the panic attacks are not better accounted for by another mental
disorder, such as Social Phobia, Specific Phobia, Obsessive-Compulsive
Disorder, Post-Traumatic Stress Disorder, Separation Anxiety Disorder

o
o
o
o
o
o
o
o
o

o PSYCHIATRY HISTORY

o STEM
o A middle-aged man who has a diagnosis of schizophrenia
presents with chest pain to your local hospital. He has been
compliant with medications and sees his GP every six months
to get his medication prescriptions. He has neither presented to
ED, nor needed hospitalisation or intervention from Mental
Health Services in the last 10 years. You are the on call doctor
at the hospital.
o
o TASKS
o Please:
o 1) Obtain a history from the patient in the first 5 minutes,
specifically focussing on the reasons for this presentation,
o
o Then
o
o 2) Tell the examiner the likely conditions that need to be
considered and your immediate course of action.
o
o
o
o
o
o
o
o
o
o
o
o
o
o
o
o
o
o
o
o
o

o
o
o
o
o
o
o
o
o
o
o HISTORY FROM PATIENT
o PROFILE
Mr Steve Smith
Age 55
Supported by DSP
Lives in group house with other mental health patients
o
o PC
Chest pain central, heavy, severe, radiates to arm/jaw, lasting 40mins
Thinks chest pain could be someone squeezing his chest
SOB past 20 mins
Sweaty + dizzy
Past angina like pain 3 episodes this year
No change in schizophrenia symptoms
o
o PAST MEDICAL HISTORY
Schizophrenia
o Diagnosed age 21 after 4 years of marijuana use
o Paranoid with persecutory demands
o Been on olanzipine for 30 years
o Going well, symptoms under control
CVS risk factors
o Obese
o Hypercholesterolaemia
o Pre-diabetic
Smokes 50pd since age 12
o
o FAMILY HISTORY
Adopted dont know
Adopted family has no contact with
o
o DIAGNOSIS
Acute MI olanzipine, obese, high cholesterol, pre-diabetic
Differentials acute psychotic episode, MSK, pericarditis, pneumonia, PE
o
o MANAGEMENT
Oxygen, IV access, aspirin, clopidogren, GTN, analgesia
ECG, troponins, CBE, electrolytes, BGL, lipids
If ACS PCI or angiogram
o

o
o MARKING
o INTERACTION
o A - Empathic, develops rapport, attends to patients cues, listens to what
the patient is saying in a respectful manner. Appears unhurried, fluent and
efficient. Able to prioritise questions.
o
o HISTORY
o A - Thoroughly explores nature of the chest pain and establishes all
cardiac risk factors. Comprehensively covers both physical and psychiatric
symptoms. Obtains a description of previous psychiatric presentations and
treatment, and/or other background history.
o
o DIAGNOSIS
o A - Achieves the criteria for B standard, and also recognises the
delusional or idiosyncratic interpretation of the physical symptoms.
o
o B - Considers the correct leading provisional diagnosis that must be
Further investigated in this presentation. Can also mention xxxxxxx, but if
so, must recognise this to be of lesser likelihood and lesser urgency in the
current situation.
o
o MANAGEMENT
o A - Achieves B standard with greater sophistication in management plan,
e.g. evaluates xxxx risk factors (yyyy), offers appropriate treatment
pending review of results, appropriate monitoring.
o
o B - Intends to undertake an appropriate examination and investigations for
the correct leading provisional diagnosis (e.g. xxxxxxxx).
o
o
o
o
o
o
o
o

o
o

o
o
o

o
o
o PSYCHIATRY HISTORY
o

o You have 6 minutes for this station.


o
o Spend 5 minutes obtaining a history from the patient about
their presenting problem and any relevant background history.
o
o Describe to the examiner the patients major symptoms and
provide a diagnosis
o
o
o
o
o
o
o
o
o
o
o
o
o
o
o
o
o
o
o
o
o
o
o
o
o

o
o
o
o
o
o
o
o
o
o
o
o
o
o
o
o
o PSYCHOSIS HISTORY

o PATIENT PROFILE
Mr Colin Farell
19 year old male
o Born 23/10/1992
Apprentice electrician
Lives at home with his parents
Only child
Appears dishevelled

o PRESENTING COMPLAINT
Mother brings him to the ED complaining that Colin has been acting
aggressively towards her and has been acting strangely
o HISTORY OF PRESENTING COMPLAINT
Colin says
o Colin believes that he needs to stop the aliens from destroying the world
o He believes that his parents are working to help the aliens and doesnt
trust them anymore.
o He knows this because he can hear someone speaking to him and telling
him to hurt his parents because they are evil. Evil like cats. Cats are kind
of evil. Cats like eating food. Cat food, not human food. I dont like cat
food. I dont trust it. And I dont trust my parents
o He thinks that the aliens are constantly watching him and fears that they
are going to abduct him soon, so that they can destroy the world
o He thinks the aliens have tried to abduct him before because he could
hear them creeping around in his bedroom. He escaped this time because
he heard them and turned on the light, causing them to disappear. He is
afraid that he will not be so lucky next time.
o Not depressed
o Not manic
Colins mother says:
o For the past 8 months or so, Colins behaviour has been changing
gradually

o He used to be very enthusiastic about going to work and would attend


every day, but now he seems apathetic and rarely goes
o He has not been showering daily, washing his clothes or shaving as he
normally would
o He does not seem to be eating as much as usual
o She has noticed him mumbling to himself more recently, perhaps over the
last month or so
No trouble sleeping
No thyroid symptoms
No headaches or changes in vision
No evidence of diabetes or endocrine disorders
o MENTAL STATE EXAM
Appearance: dishevelled, appears not to have shaved for a while
Behaviour: Cooperative, doesnt make much eye contact
Conversation: Speaks disjointedly and often about the aliens. Has trouble
staying on track with conversation
Affect and mood: anxious and fidgety
Perception: auditory hallucinations: hearing the aliens. Has not had visual
hallucinations
Cognition: Poor concentration and talks about paranoid themes. Orientated to
time, place and person
Insight: Believes he is not sick and is becoming frustrated that he is being kept
at the hospital for so long because he cannot protect the world from here
Judgement: Impaired, cannot see that his beliefs are delusional
Rapport: Superficial only
o MEDICAL HISTORY
He has never had any medical problem
He does not take medications
He has smoked marijuana intermittently over the past 5 years or so, and last
smoked 2 cones one month ago
No other illicit drugs
No allergies
o FAMILY HISTORY
His mother was adopted and has never met her birth parents/biological family
so he is unsure about his family medical history
His mother has never had any medical problems
o SOCIAL HISTORY
This time last year he had an active social life and would regularly have friends
over to the house. He has become progressively more withdrawn and now has
no visitors
He broke up with his girlfriend about 3 months ago
He has not been going to work as often as normal but this does not seem to
bother him
He drinks a moderate amount of alcohol, 2-3 beers per night, but rarely to
excess

o MAJOR DIFFERENTIALS
Schizophrenia
Substance induced psychosis
Bipolar disorder with psychotic depression
Unipolar Psychotic depression
Schizoaffective disorder

1.
2.
3.
4.
5.

o
o DIAGNOSIS
o DSM-5 diagnostic criteria for schizophrenia are described in more detail
below [1].
o A. Two or more of the characteristic symptoms below are present for a
significant portion of time during a one-month period (or less if
successfully treated):
Delusions
Hallucinations
Disorganized speech (eg, frequent derailment or incoherence)
Grossly disorganized or catatonic behavior
Negative symptoms, ie, affective flattening, alogia, or avolition
o B. For a significant portion of the time since the onset of the disturbance,
one or more major areas of functioning such as work, interpersonal
relations, or self-care are markedly below the level achieved prior to the
onset. When the onset is in childhood or adolescence: failure to achieve
expected level of interpersonal, academic, or occupational achievement.
o C. Continuous signs of the disturbance persist for at least six months. The
six-month period must include at least one month of symptoms (or less if
successfully treated) that meet Criterion A (ie, active-phase symptoms)
and may include periods of prodromal or residual symptoms. During these
prodromal or residual periods, the signs of the disturbance may be
manifested by only negative symptoms or two or more symptoms listed in
Criterion A that present in an attenuated form (eg, odd beliefs, unusual
perceptual experiences).
o D. Schizoaffective disorder and mood disorder with psychotic features
have been ruled out because either: (1) no major depressive, manic, or
mixed episodes have occurred concurrently with the active-phase
symptoms; or (2) if mood episodes have occurred during active-phase
symptoms, their total duration has been brief relative to the duration of
the active and residual periods.

o E. The disturbance is not due to the direct physiological effects of a


substance (eg, a drug of abuse or medication) or a general medical
condition.
o F. If the patient has a history of autistic disorder or another pervasive
developmental disorder, the additional diagnosis of schizophrenia is made
only if prominent delusions or hallucinations are also present for at least a
month (or less if successfully treated
o Causes of Psychosis in a young person
GMC (head trauma, tumor, MS, DLB, AD, PD)- CT head, MRI, Neurological
Examination
Electrolyte disturbances, metabolic, UTI (CBE, EUC, TFT, LFT
Hypoglycaemia
AIDS, leprosy, malaria, neurosyphillis
Dementia/ delirium
Epilepsy (EEG)
Substance induced psychosis (EtOH, benzodiazepines, barbiturates,
anticholinergics, antidepressants, stimulants, hallucinogens, psychotropics)
urine Toxicology screen
Affective disorders: psychotic depression, schizoaffective disorder, manic
episode with psychotic features
Personality disorders: borderline, paranoid, schizoid
Primary psychotic disorder: schizophrenia, schizophreniform, schizoaffective
Olfactory hallucinations think organic, esp temporal lobe epilepsy?
Visual/Tactile hallucinations: common in delirium and alcohol withdrawal
o
o
o

o
o
o

o
o
o
o
o

o PSYCHIATRY STATION
o
o

o You will be asked to take a history in a 55-year-old married


woman whose presenting complaint is: Since my home was
broken in to I just cant relax or sleep well at night
Please take a history focussing on post-traumatic symptoms in the
first 5 minutes
In the last minute please state the appropriate treatments for this
woman
o
o (The Examiner will inform you when the 5 minutes has
elapsed).
o
o
o
o
o
o
o
o
o
o
o
o
o
o
o
o

o
o
o
o
o
o
o
o
o
o
o
o
o
o
o
o
o
o
o
o
o
o
o
o
o Profile
Sue Hill
55 year old woman
Receptionist
Marries to Husband Greg

o PC
Since my home was broken into I just cant relax or sleep at night

o HxPc
Home was broken into 6 weeks ago
o Middle of the night
o She was sleeping and woke to the sound of someone smashing a window
o She called the police whilst lying in bed and tried to pretend she was
asleep
o Could hear a person walking through the house
o A man opened her bedroom door and walked through her bedroom,
stealing her iPad on her dressing table
o She stayed quiet and he didnt realise she was awake
o Husband was away on business
o Children have moved away home alone
o Police came and arrested him
Has been having very vivid dreams about the event and waking up in a cold
sweat nearly every night. They feel very real.
Has been very scared of sleeping, has been staying up as late as she can and is
often unable to sleep well as she is very anxious when trying to sleep

o Sleeping makes her think of the robbery and makes her afraid that it will
happen again
Always wakes with a start because she is afraid that someone has broken in
whilst she is sleeping
She has spoken to her friends and husband about it but feels that they dont
understand
o They werent there so cant possibly understand how she is feeling and
why she is scared
o She has been isolating herself from them and cutting them out
Cant remember the last time she felt happy, just feels teary and anxious all the
time
Difficulty concentrating
Has been getting scared by the smallest things
o Her cat unexpectedly rubbed against her leg whilst she was sitting at the
table and it made her jump
o House creaking terrifies her now
Feels that her life has changed for the worst after this event
Her husband feels that she is no longer the same person
These symptoms have been occurring for the past 6 weeks, but have been
getting progressively worse
Has had to take some days off work when she feels too anxious
Hates being in her house

o MHx
Nil significant

o FHx
Mother had depression
Nil significant
Adult children

o SHx
Non-smoker
Has been drinking a few bottles of wine per week to help her to relax and sleep
for the last few weeks
o Used to drink 1 glass of wine per night, now 3-4 per night
No illicit drugs
o

o PTSD

o Since the new DSM-V has been introduced, the diagnostic criteria is based
upon the following 8 components:
o 1: Any of
Directly experiencing the traumatic event
Witnessing, in person, the event as it occurred to others
Learning that the traumatic event occurred to a close family member or friend
Experiencing repeated or extreme exposure to aversive details of the traumatic
event(s). This does not apply to exposure through media
HISTORY:
o Tell me about this event

o 2: Persistent re-experiencing of the event in one of the following


ways:
Thoughts or perception
Images
Dreams
Illusions or hallucinations
Dissociative flashback episodes
Intense psychological distress or reactivity to cue that symbolise some aspects
of the event
HISTORY:
o Have you been thinking about the event and reliving it?
o Have you had any bad dreams about the event?
o Have you been seeing images of the event in your mind/head?
o Have you had any flashbacks to the event and felt like it was happening to
you again?
o Have you seen/heard/experienced anything related to the event that other
people may not have been able to see/hear/experience?
o Have you found that any activities or things remind you of the event and
make you feel very distressed?
o 3. Avoidance of stimuli that are associated with the trauma in one
of the following ways:
Avoidance of thoughts, feelings or conversations that are associated with the
event
Avoidance of people, places, or activities that may trigger recollections of the
event
HISTORY
o Have you been avoiding any thoughts, feelings, people, places that
remind you of the event?
o 4. Two or more of the following symptoms of negative alterations
in cognitions and mood associated with the traumatic event
Inability to remember certain important aspects of the event
Persistent and exaggerated negative beliefs about oneself, others or the world
Persistent distorted cognitions about the cause or consequences of the event (s)
Persistent negative emotional state
Markedly diminished interest or participation in significant activities
Feelings of detachment or estrangement from others
Persistent inability to experience positive emotions
HISTORY
o Are you able to remember everything that happened during the event?
o Have you been having any negative thoughts about yourself, others or the
world around you that you were not having before the event? Like What?
o Are you still keeping up with your normal interests?
o Have you been socialising much? Do you still feel that you are able to
relate to the people around you?
o Have you been able to feel happy since the event occurred?
o depression screen
o 5. 2 or more of the following alterations in arousal and activity

Irritable behaviour and angry outbursts


Reckless or self destructive behaviour
Hypervigilance
Exaggerated startle response
Concentration problems
Sleep disturbance
HISTORY
o Have you found that you are more irritable than normal, or become angry
about things that never used to upset you?
o Have you been doing anything to intentionally harm yourself or which put
yourself in danger?
o Are you finding it difficult to relax?
o Do you feel on edge?
o Have you been having difficulty concentrating?
o How has your sleep been?
o The remaining criteria:
Symptoms present for at least 1 month
The disturbance is causing clinically significant distress or impairment of
functioning
The disturbance is not attributable to the physiological effects of a substance or
other medical condition/DMS criteria
o On history, special attention should be paid to the patients sleep hygiene.
o COMPLICATIONS OF PTSD:
Increased risk of:
o Alcoholism
o Impulsive behaviour
o Suicide
o Homocide
o MANAGEMENT
o Treatment is often best achieved with a number of pharmacological and
non-pharmacological interventions.
o Non-Pharmacological
Group therapy
Individual and family therapy
CBT
Art therapy
Anxiety management
Relaxation exercises
Hypnosis
Referral to psychologist/psychiatrist
o Pharmacological
Principle agents of treatments have been antidepressants and beta-blockers:
o SSRI, TCA
Beta-blockers
o Useful in controlling symptoms related to hyperarousal
o PROGNOSIS

Average duration:
o With treatment: 36 months
o Without treatment: 64 months
o DIFFERENTIALS
o Adjustment Disorder
o An Adjustment disorder is a psychological response to an identifiable
stressor that results in clinically significant symptoms. The DSM IV criteria
are:
Clinically significant emotional or behavioural symptoms in response to an
identifiable stressor (s), occurring within 3 months of its onset, and non
persisting beyond 6 months upon the termination of the stressor
Clinical significance is define as either marked distress in excess of what one
would generally expect relative to the stressor, or significant impairment in
social or occupational functioning
Symptoms do not represent bereavement or meet the criteria for another axis I
disorder
o A patient may have a depressed mood following experiencing an
appropriate stressor, but may not exhibit other symptoms of depression,
and therefore cannot be diagnosed with MDD
o Mood Disorders with Psychotic Features
MUST do a depression screen
o Substance induced anxiety
o

o PSYCHIATRY HISTORY
o
o

o 1 minute reading time 6 minute question time


o Instruction to Students
o You have 6 minutes at this station Description of task: Interview
takes place in emergency room of hospital. You have called to
assess a man in his late 50s. He has been brought to the
hospital by his wife who is extremely worried. She is very
concerned he is so down and having very negative thoughts.
o Please interview the patient for five minutes THEN tell your
findings and basic treatment plan to the examiner.
o (The Examiner will inform you when the 5 minutes has
elapsed).
o
o
o
o
o
o
o
o
o
o

o
o
o
o
o
o
o
o
o
o
o
o
o
o
o
o
o
o
o
o
o
o
o
o
o
o
o
o Profile
Ken Smith
59 y/o
Finance Manager
Lives at home with wife and 2 adult children (daughters)
o PC
Wife is very worried about Ken because he just seems really down and is always
speaking and thinking negatively
o HxPC
Ken has been feeling down for the last 6 months or so
o Progressively getting worse
o Depressed mood
o No longer interested in meeting up with his friends at the pub
Social hang gliding club
Doesnt attend meetings either
Started when he began to invest money into some new stocks that were
supposed to skyrocket
o They did not
o He lost a lot of money
Continued to buy shares but just keeps losing
o Wife doesnt know

o They are very in debt and he doesnt know how to get out of it as all of his
attempts keep failing because he keeps choosing the wrong shares
o Has tried gambling which did not work lost more money
o Daughters are relying on him whilst they finish studying (both arts
degrees) as he does not feel they should be working
o Has not been able to pay bills
Feels worthless
Has been waking in early hours of the morning and cant get back to sleep
Decreased interest in food
Unable to concentrate at work
Has been having suicidal thoughts
o Feels hopeless for future and feels like a failure wants relief from these
thoughts
o Loves his wife and daughters but feels they would be better off without
him
o No previous attempts
o Plan in place
No set date he would just do it when it felt right, probably when he
loses the house not far away
Plans to hang himself in the garage
Has researched how to tie a noose
Has access to equipment at home
o Medical History
Problems with anxiety as a teenager
o Received no treatment
Otherwise well
o Social History
Has been drinking more alcohol recently
o Doesnt know how much, but it is too much
o Maybe a few bottles of wine
Non smoker
No illicit drugs
o Family History
Nil significant
o
o MANAGEMENT PLAN
Admit to hospital
o Unsafe to go home
Psych consult
o Discuss treatment options with him
Medication for depression
Psychotherapy
+/- ECT

o
o
o
o
o
o
o
o HISTORY
o Depression History
o Confirm diagnosis of depression:
The DSM IV criteria for MDE is:
o At least 5 of the following occurring nearly every day (except recurrent
thoughts of death or suicide) over at least 2 weeks, which are a change
from previous functioning
Loss of interest or pleasure most of the day (one of the symptoms
must be the first or second dotpoint)
or appetite or significant change in weight
Insomnia or hypersomnia
Psychomotor agitation or retardation
Loss of energy
concentration or decisiveness
Feelings or worthlessness or excessive/inappropriate guilt
Recurrent thoughts of death or suicide
o Symptoms must cause clinically significant distress or functional
impairment
o Excludes bereavement, substance, general medical condition and mixed
episode

NB: the major change in the DSMV is that bereavement no longer


excludes a diagnosis of major depressive disorder
Can be remembered with DIGSPACES mnemonic:
o Depressed mood
o Loss of Interest
o Guilt
o Sleep disturbances
o Psychomotor agitation/depression
o Appetite changes/weight loss or gain
o Concentration difficulties
o Decreased Energy
o Suicidal Ideation

o Major depressive disorder can be diagnosed when there have been 2 or


more MDEs.
o Additional questions to ask on history
Identifiable triggers
o What it was, how long ago, if it has triggered similar episodes in the past
o Need to differentiate from adjustment disorder
Occurs in response to an identifiable trigger that occurred within the
last 3 months
Symptoms are clinically significant in that they exceed what would
be expected following the stressor, or social/occupational
functioning is impaired
Resolves within 6 months of the stressor ending
Does not fit the criteria for any other DSM diagnosis
Unipolar vs bipolar differentiation
o Episodes of mania
Feelings of elation, grandiosity
Decreased need for sleep
Increased impulsivity
Delusions
Schizoaffective features
o Psychotic features
Delusions
Hallucinations
o Presence of psychotic symptoms for at least 2 weeks without mood
symptoms
Other forms of depression
o Atypical
Hypersomnia, hyperphagia, leadened paralysis, mood
improvements in the morning
o Catatonic features
o Psychotic features
Family history
o Suicide attempts
o Psychiatric disease
Personal history/medical history
o History of psychiatric problems
What were they?
Anxiety, depression, psychosis, substance abuse/addiction

Management of these (medication, psychotherapy)


o Alcohol intake
o Smoking
o Drug use
o Other medications
o Other medical problems
Chronic pain/disease
Profile features
o Occupation
o Family
o Children
o Marital status
o Suicide History
o Risk Factors
Psychiatric disorder
o History of previous or current psychiatric disorder is a strong predictor of
suicide
Feelings of hopelessness
o Strongly associated with suicide
History of previous suicide attempts or threats
o The strongest single factor predictive or suicide is the presence of
previous suicide attempts
o Patients with a prior history are 5-6 times more likely of attempting again
o Up to 50% of successful victims have had prior attempts
o This risk is greatest in patients with schizophrenia, unipolar mood
disorders or bipolar disorder
Age, sex and race
o The risk of suicide increases with increasing age
o Young adults attempt suicide more often than older adults
o Females:males attempts are 4:1, but males are 3 times more successful
than females
o Elderly white men (aged 85 and older) have the highest suicide success
rate
Marital status
o Highest among those never married, followed in descending order by:
Widowed
Separated or divorced
Married without children
Married with children
Occupation
o Unemployed may be at greater risk
Health
o Suicide risk increases with physical illness including:
Chronic pain
Recent surgery
Chronic or terminal disease
o BMI and suicide are inversely related in men
Adverse childhood experiences
o Childhood abuse and adverse childhood experiences seem to increase the
risk of suicide in adults

Antidepressants
o Increased risk of suicidality in those aged 18-24 during the first month (or
so) of treatment
Access to weapons
o Fire arms, knives
Other groups with increased risk include:
o Those who live alone
o Those who have recently lost a loved one
o Protective Factors
Social supports
o Friends
o Social groups/clubs
Family supports
o How close is the family?
o Feuds?
o Family discord increases the risk of suicide
Pregnancy
Parenthood
o How are the children?
Religious beliefs
o Participation in religious groups/activities
o Evaluation of Suicide Risk
Are you having any thoughts of ending your life?
o Content
o For how long have these been occurring?
o Have these thoughts changed recently?
Have they become more frequent, more intense
o Do you feel that you are able to control these thoughts?
Have you attempted anything like this before?
o How?
o When?
o Trigger?
o How did you feel when it was not successful?
Are you afraid of death?
o How does thinking about death make you feel?
Scared?
Relieved?
How do you think ending your life will improve your current situation?
o Thoughts on how it will affect friends/family? (not sure about these
questions)
Have you formulated a plan to commit suicide?
o When did you start planning?
o When do you plan for it to happen?
o Triggering event before it happens?
o How?
o Where?
o What do you anticipate the outcome to be?
Do you have readily available access to any equipment that you plan to use? Do
you know how to use the equipment?

Have you started to collect any of the equipment that you will need?
Do you feel hopeful for the future?

o Assess need for detainment


Does the patient have a mental illness?
Is the patient at imminent risk to themselves or others?
Is there no less restrictive means than detainment to ensure the patient
receives appropriate treatment
MANAGEMENT
Management of Depression
Mild to Moderate Depression
Depression is considered mild to moderate when there is no suicidal
ideation, no psychotic features, no aggressiveness.
o Management:
Combination of pharmacotherapy and psychotherapy
Pharmacotherapy
o SSRIs are generally recommended and are the most widely prescribed
Eg, sertraline, escitalopram
o SNRIs are also appropriate
Eg venlafaxine
o TCAs are generally avoided because they can be toxic at high levels and
would provide accessible means of committing suicide.
Psychotherapy
o CBT or interpersonal therapy are frequently prescribed
Addition of relaxation techniques, exercise
o
o
o
o

o Severe or major depression


o Depression is considered severe when there is active suicidal ideation,
psychotic features, poor judgment, grossly impaired functioning.
o In this case, both pharmacotherapy and psychotherapy, and
pharmacotherapy alone are appropriate. ECT is another appropriate
alternative (particularly in classic melancholic depression with
neurovegetative features)
o Treatment:
Pharmacotherapy
o SSRI or SNRI
ECT
o Particularly in patients requiring a fast response (eg suicidal or homicidal
ideation)
o Management of Suicidal Ideation
o Management of the individual should include:
Reduce immediate risk
Manage underlying risk factors
Monitoring and follow up
o Reducing Immediate Risk
Immediate psychiatric services, usually including hospitalisation, is appropriate
for those at imminent risk of suicide/self harm

In patients at high but not necessarily imminent risk (eg, desire to commit
suicide but no specific plan) :
o Hospitalisation may not be required, but rapid intervention is appropriate
o Involve a family member or person close to the patient if that is allowed
o Ask about the availability of lethal means and make them inaccessible to
the patient
o Increase the frequency of contact with the patient and communicate a
commitment to helping them
o Begin aggressive treatment of psychiatric disorder or substance abuse
In severely depressed suicidal patients, the rapid onset of effect of ECT may be
lifesaving
o

o
o
o
o
o PSYCHIATRY HISTORY
o
o Description of task:
o
o Mr David Smythe is 55-year-old gentleman who has been
brought to the ED by SAAS following the ingestion of lots of
pills an hour ago.
o
o The SAAS Officer explains to you (an Intern in the ED), that the
patient was attempting to commit suicide but SAAS were called
by the patients concerned wife.
o
o
Tasks for the student:
Take a focused history from the patient, to determine the method
and causes behind his presentation
Outline your management of this patient
Describe the criteria for detention under the Mental Health Act
o
o
o
o
o
o
o

o
o
o
o
o
o
o
o
o
o
o
o
o
o
o
o
o
o
o
o
o
o ID: David Smythe
55-year-old man
Lives with wife, but having lots of problems
Recently fired from job as accountant
o
o PC: Took lots of pills of whatever I could find, because my wife
was saying she was going to leave me.
o
o History of suicide attempt:
o - Was having argument with wife, threatened she was going to
divorce me fighting about money
o - After she left, I went into the medicine cabinet and pulled out
a packet of paracetamol with 20 tablets in it
Happened 1 hour ago
Took all the tablets
o - Didnt take anything else
o - Called wife afterwards to tell her what he had done, she called
SAAS
o - Feels well/asymptomatic;
No N+V, no abdo pain
No headache
No LOC or confusion
o

o HPC:
o - Has never attempted suicide before but has thought about it
considered driving car into a tree, but couldnt do it
o - No access to firearms
o - No homicidal thoughts
o - Has been feeling depressed for 1 year, since being fired
Depressed mood
Loss of interest in normal activites not seeing friends, stopped
playing golf
Spends all his time at home
Not looking for work no point as he feels worthless/useless
Not sleeping well, cant get to sleep
o - Has increased alcohol consumption in last 6 months only 1
or 2 alcohol free days per week, drinking rum to ease the pain
o - Financial strain since being fired, cause of argument with wife
o - Has never been depressed prior to this
o
o PMHx:
o - Anxiety as a teenager, but received no Tx
o - Nil other significant
o - No meds, no operations, no allergies
o
o Smok/Alc:
o - Does not smoke, has never smoked
o - Alcohol as above
o - No illicit drugs
o
o Social:
o - Lives with wife has been generally supportive but is
becoming increasingly frustrated with him as he is doing
nothing to help his situation
o - 1 daughter, lives out of home, supportive
o
o Fam:
o - Mother had depression
o - Nil other significant
o
o
o DIAGNOSIS:
o - Paracetamol overdose (10g) on the background of depression
o
o
o ACUTE MANAGEMENT:
o - Patient has taken overdose of paracetamol 1 hour ago

Took 20 tablets = 10g of paracetamol


o - Admit patient
o - Acute mgmt:
Gastric lavage
+/- activated charcoal (usually given if presentation within 1 hour)
Supportive Tx fluids
Bloods CBE, biochemistry, LFTs, ABG, coags
o - Wait 4 hours to obtain paracetamol level and plot on
nomogram
If levels fall on or above the line
o N-acetylcytseine (NAC);
Should be given no matter timing of paracetamol
ingestion
Promotes hepatic glutathione synthesis
Continue until there is clinical/lab evidence of
improvement
If levels fall below the line supportive therapy only
o
o
o LONG-TERM MANAGEMENT:
o - Psych consult
o - CBT
o - Antidepressants
o - +/- ECT
o
o MENTAL HEALTH ACT:
o Criteria for detention:
o The Mental Health Act 2009 allows for the detention of
individuals for involuntary treatment when:
o
(a) The person has a mental illness; and
(b) Because of the mental illness, the person requires treatment for the
persons own protection from harm(including harm involved in the
continuation or deterioration of the persons condition) or for the
protection of others from harm; and
(c) There is no less restrictive means than a detention and treatment
order of ensuring appropriate treatment of the persons illness.
o
o Furthermore;
Detention can only be enforced if the facility is appropriate
Patient must be reviewed by psychiatrist within 24 hours to remain
detained
o

o
o

o
o
o PARACETAMOL OVERDOSE

o
o - Most common cause of acute liver failure
o - Toxic dose varies from person to person but general principles;
Toxicity is unlikely to result from a single dose of <150 mg/kg in a child or 7.510g for an adult
Toxicity is likely to occur with single ingestions >250 mg/kg or those greater
than 12g over a 24-hour period
Smaller dose required to produce toxicity in alcoholics/anticonvulsant users (46g)
o
o Aetiology & Pathophysiology:
o - After ingestion of a therapeutic dose, paracetamol undergoes hepatic
sulfation and glucuronidation, and the resulting non-toxic metabolites are
excreted in the urine
o - About 4% of a therapeutic dose is metabolized by cytochromes P450
(mainly CYP2E1) in the liver that produces the potentially toxic
intermediate metabolite (NAPQI)
Normally NAPQI combines with intracellular glutathione to become a non-toxic
metabolite that it renally excreted
o - After ingestion of an overdose however, the production of NAPQI exceeds
the capacity to detoxify it binding of NAPQI to hepatocyte membrane
hepatocyte death
o - If a sufficient dose is taken, hepatocyte death may be massive and
sufficient to cause acute liver failure
o - CYP2E1 is also found in the kidney, and its presence may be a factor in
variable degrees of renal injury occasionally seen in paracetamol overdose
o

o Clinical features:
- First 24 hrs: asymptomatic or nausea and vomiting (usually within 4-12
hours of overdose)
o - Next 2-3 days: hepatic necrosis resulting in increased aminotransferases
(ALT and AST), jaundice and right upper quadrant pain
o - After 3 days: possibly hepatic encephalopathy: confusion, decreased
conscious level, and asterixis (indicates progression to fulminant hepatic
failure), acute renal failure and death
o - NB: there may be a delay in presentation with overdose of sustainedrelease products
o - Should also investigate for history of self-harm
o
o Investigations:
o - Need to determine time course when were pills taken?
o - Paracetamol level;
Check as early as possible, but at the earliest 4 hours after ingestion
The level of serum paracetamol relative to the time of ingestion will determine
need for subsequent treatment (see the normogram below only for a single
acute paracetamol overdose)
o - LFTs;
AST and ALT typically elevated, in severe toxicity (doses >350 mg/kg)
transaminase levels may be >1000 IU/L
o - ABG may show metabolic acidosis
o - Biochemistry may show elevated urea and creatinine
o - Coags coagulopathy
o - Urine drug screen to determine if other substances have been taken
o
o Management:
o - Check paracetamol level after 4 hours since ingestion
If a patient presents within 4 hours of ingestion, the level is drawn when 4 hours
have elapsed since the possible overdose
Once obtained, the level should be plotted on the relevant nomogram against
the time since ingestion
o - If patient presents <1 hour since ingestion;
Gastic lavage/emesis (can be used up to 2 hours since ingestion)
Activated charcoal absorbs paracetamol, hence preventing its absorption in
the GIT
o - Once paracetamol level obtained;
If levels fall on or above the line
o N-acetylcytseine (NAC);
Should be given no matter timing of paracetamol ingestion
Promotes hepatic glutathione synthesis
Continue until there is clinical/lab evidence of improvement
If levels fall below the line supportive therapy only
o - NAC should also be started immediately/empirically when;
Patient presents 8 hours or more after ingestion
Serum paracetamol level is not available within an 8-hour time window
There is uncertainty as to the timing of the overdose
Patients are unconscious or have a suspected overdose
o - All patients should receive supportive Tx

o - Patients presenting with or progressing to fulminant hepatic failure


should be treated with continuous IV NAC and be referred for liver
transplantation
o
o Prognosis:
o - Hepatic regeneration in survivors is normally rapid and complete, with
normalization of LFTs within 1-3 weeks
o - 1-2% die from hepatic failure
o

o
o
o

o
o
o
o
o
o
o
o
o
o
o
o
o
o
o

o
o
o
o
o
o
o
o
o
o
o
o

o PHYSICAL EXAMS
o
o
o
o
o
o
o
o ACUTE ABDO EXAM 1
o

o
o
o
o
o
o
o
o
o
o
o EXAM FINDINGS (FOR SP TO FAKE)

General inspection severe pain, not moving much, sweaty, (note: NOT
jaundiced)
Observations tachycardia
GIT RUQ tenderness, positive Murphys sign, palpable gallbladder
o
o STUDENT TO THEN COUNSEL SP ON LIKELY DIAGNOSIS AND
MANAGEMENT
Likely to have acute cholecystitis
Differentials hepatitis, PUD, gastroenteritis, choledolithiasis etc
Management
o Pain relief
o Nil by mouth
o IV fluids
o USS ABDO to confirm diagnosis
o Cholecystectomy
o
o SUMMARY OF CONDITION: ACUTE CHOLECYSTITIS

o PATHOPHYSIOLOGY
Chemical irritation from concentrated bile
Physical irritation from increased pressure
Sometimes bacterial infection
o
o PRESENTATION
Severe RUQ pain
Fever
Tachycardia
Gall bladder may be palpable
Positive Murphys sign
o
o INVESTIGATIONS
Is usually a clinical diagnosis
Ultrasound can support the diagnosis by showing gall stones and a thickened gall
bladder wall
o
o MANAGEMENT
Pain relief
Antibiotics are not usually necessary as it is not usually infective but they are given
when the case is severe
Restrict oral intake and give IV fluids
Cholecystecomy
o Can be done early (within a few days) or late (>6weeks later)
o Gall bladder appears edematous and inflamed with petechial haemorrhages,
purulent exudates and fibrous adhesions. There will be gall stones present
Empyema of the gall bladder
o The gall bladder is filled with pus
o Sometimes, part of it becomes necrotic perforation subphrenic abscess
generalized peritonitis
o This is rare due to the rich blood supply from the hepatic bed and cystic artery
o Required urgent surgery
o

o
o
o
o

o
o
o
o
o
o MARKING SCHEME

o
o
o

o
o
o ACUTE ABDO EXAM 2
o

o Instructions to student:
o You are an intern working on a surgical unit. This patient has
presented with a 24-hour history of colicky abdominal pain,
which has now shifted to the right iliac fossa and become
constant. He is nauseated and has not eaten anything for 24
hours. He has vomited once this morning. He is in good health
and does not take any medications. His only medical history of
note is a fractured tibia five years ago (a motor bike crash),
which was complicated by a DVT.
o Task
Perform an appropriate physical examination
Give the patient your diagnosis
Counsel the patient on your plan of management
o Please note: you are NOT required to obtain additional history
o
o
o
o
o
o
o
o
o
o
o
o
o
o
o
o
o
o
o
o
o
o
o
o
o
o
o
o
o
o

o
o
o
o
o
o EXAM FINDINGS
General Inspection
o Lying still on bed clutching abdomen
o Looks distressed
o Dry mucous membranes
o Coated tongue
o Lower limbs not erythematous, swollen or tender to palpation
Vital Signs
o Tachycardic
o Tachypnoeic
o Febrile 38.0 degrees Celsius
o SaPO2 99% RA
o BP 130/90, no postural drop
o Urine output poor
Abdomen
o Inspection
No abnormalities
No scars
o Palpation do not start by palpating the RIF. Keep looking at the patient
No rigidity
Guarding to palpation over RIF
No palpable mass
Positive McBurneys sign
McBurneys point located at the lateral third of the distance
between the ASIS and the umbilicus
Positive Rovsings sign
Crossed tenderness
Negative psoas sign
Pain on passive hip flexion when the patient lies on his side
with extended knees, OR active hip flexion
Negative obturator sign
Pain on hip flexion and internal rotation
Negative murphys sign (probably no need to do this)
rebound tenderness over RIF percussion tenderness is a better
choice
o Percussion
Percussion tenderness
o Auscultation
Reduced bowel sounds
o DIAGNOSIS
Acute appendicitis
o Dx (although probably not applicable in this case)

Ectopic pregnancy
UTI

Mesenteric adenitis
Cholecystitis

Diverticulitis

Crohns

MANAGEMENT
In this case, Im not sure if you would just go straight to theatre given the
diagnosis is so clear cut (ie, no USS). If the diagnosis was less clear cut:
Investigations
o CBE
Neutrophilia and leucocytosis
o CRP
Elevated
o (pregnancy test)
o Abdominal Ultrasound reliable for diagnosis confirmation but not
exclusion
Presence of an aperistaltic, non-compressible, blind ended, sausage
shaped structure arising from the base of the caecum
Distinct appendiceal wall layers
An outer diameter greater than 6mm
A target appearance
Faecolith
Periappendiceal fluid collection
Echogenic, prominent pericaecal fat
o Other option of CT scan
Only if diagnosis is uncertain following USS
Surgical Management
o Prompt appendicectomy
o Antibiotics
If there is no rupture:
Metronidazole 500mg/8hr and Cefuroxime or cefazolin
In penicillin and cephalosporin allergy:
o Clindamycin plus one of ciprofloxacin, gentamicin,
levofloxacin
Start 1 hour pre op to minimise wound infection
Rupture
Ceftriaxone plus metronidazole

ASSESSING FITNESS TO DRIVE


STEM
CLINICAL SCENARIO
Mr Talbot has come to you to determine whether he is fit
to continue to drive his car. He is in his 70s, an ex farmer,
and lives on an outlying property with his spouse.

He had a small non-dominant occipito-parietal stroke two


months ago. His family note that he is still bumping in to
things around the house. He walked unaided into the
consulting room and has no obvious weakness.

He was told he would have to wait 6 weeks after the


stroke before he could drive. The following additional
information is available: BP 120/70mmHg sitting visual
acuity 6/6 with his glasses on Mr Talbot has a mini-mental
score of 25/30.

TASK
Examine Mr Talbots vision, and advise Mr Talbot regarding
his suitability to resume driving his car. (Fundoscopy will
not be required)

ANSWER
Visual fields
o No vision on left side on either eye
Ocular movements
o Normal
Diagnosis
o Left sided homonymous hemianopia
o Caused by his stroke
Cannot drive

MARKING SCHEME

BREAST EXAM

Instructions to student:
This woman presents with a painless swelling in her right
breast, which she first noticed about three weeks ago. She
is post-menopausal and has never had any previous
breast problems. She is otherwise in good health.
Task
1. Undertake a formal breast examination
2. Provide the patient with a plan of immediate management based on
the findings

EXAM
Inspection (sitting)
o Asymmetry
o Scars
o Lumps
o Peau dorange
o Nipple changes crusting, bleeding, oozing, inversion
Inspection (sitting with hands on hips and pushing inwards)
o Skin tethering
Palpation (lying)
o Note: patient to place arm behind head on side you are examining
o Repeat inspection
o All quadrants of each breast
o Nipple attempt to express any fluid
o Axillary tail
o Lymph nodes including supraclavicular fossa

Palpation technique
- The vertical strip, or grid, technique

1.
2.
3.
4.
5.

The breast is divided into 8 or 9 vertical strips, each approximately one fingers
width
The examiners 3 middle fingers are held together and slightly bowed to ensure
contact with the skin
The pads, not tips must be used
Using dime sized circles, examine the breasts at each of three different levels
of pressure
o Light
o Medium
o Deep
Describing the lump
The size of the mass in centimetres and its position
The shape of the mass
The delimitation, referring to the borders of the mass. Is it well delimited, as
with a cyst? Are the edges diffuse, as with a carcinoma?
The consistency, describing the hardness of the mass. A carcinoma is often
stony
The mobility of the lesion. Is the lesion movable in the tissue that surrounds it?
Benign tumors and cysts are freely mobile whilst carcinomas are usually fixed to
the skin, underlying muscle or chest wall
Findings on Physical Exam
Solid, 3cm, non-mobile, painless lump found in right upper outer quadrant
No skin tethering
No peau dorange
No erythema
Lump not visible on general inspection of the breasts
Further Management
The gold standard of assessment of breast lumps is triple
assessment:
Clinical examination
Breast imaging
o Diagnostic mammography
Biopsy (where indicated)
o FNA initially

These are to occur on the same day (in some clinics) and have an
accuracy of 99.6% when performed by experienced personnel.
MARKING

Remember: triple assessment for breast lump = clinical exam,


imaging (e.g. diagnostic mammogram or ultrasound) and biopsy
(e.g. FNA or Tru-Cut/core biopsy)

CARDIAC EXAM

Instructions to student:
Clinical scenario
You are an intern in the Department of Surgery in a
metropolitan hospital. You are conducting a surgical preadmission clinic with a medical student. The student asks
you to review a patient who the student feels has an
abnormal heart examination. You are informed that the
patient is booked for a herniorrhaphy and is otherwise
well.

Task
Conduct a cardiovascular examination that focuses on the
blood pressure, neck and praecordium, and inform the
student of your findings.
Assume the examiner is the medical student .

Examination Findings
Blood Pressure

Normal
Checks radial pulse first as an estimate
Inflate to 30-40 mmHg higher than the radial pulse estimate

Neck
Carotids palpation and auscultation
JVP

Praecordium
Displaced apex beat
o 6 ICS, Anterior axillary line
o Pansystolic murmur
o Radiation to axilla
o Valsalva makes murmur softer

Diagnosis is Mitral regurgitation

The Cardiovascular Examination

Position the patient at 45 degrees


ANSWER
General inspection
o Respiratory distress
o Sweaty or cyanosis
o In pain
o Obese/wasted
BP
o Measure with radial pulse first?
Neck
o Carotids
Auscultate for bruit or radiation
Palpate for volume and character
Aortic stenosis, pericardial effusion - small volume +small
upstroke
Aortic regurgitation, anaemia, PDA - collapsing
o JVP
Inspection
Should be less than 3cm from sternal angle

Not palpable, has two flickers (a and v waves), moves on


respiration
Hepatojugular reflex
Firm abdominal pressure for 10sec
Normal no increase or small increase for <3sec
Abnormal sustained increase in JVP of 4cm or more during
abdominal pressure (or a drop of 4cm or more after release of
pressure)
Cause of abnormal result right heart failure (most correlation
to pulmonary artery pressure)

Chest
o Inspection
Scars or pacemaker
Chest deformities
o Palpation
Apex beat
Heaves, thrills
o Auscultation
All 4 quadrants
Radiation
Mitral regurgitation axilla
Aortic stenosis carotids
Dynamic manoeuvres
Aortic regurgitation lean forward, expire and hold breath
Mitral stenosis roll patient onto left side and use bell
HOCM (systolic)
o Only one which will get louder with valsalva
o Squatting will make it quieter
Further murmurs
VSD pansystolic head over the 4th ICS with no radiation
ASD right sided ejection systolic murmur OR wide fixed
splitting of S2 OR signs of RV overload
o Back
Palpate for sacral edema
Percuss for dullness of the bases
Ausculate for pulmonary edema or effusion

NOTES FROM STATION


Blue
o Neck: Massive pulsations, masking the JVP (?not elevated), palpable
carotids that were very strong
o Chest: lots of scars (front and sides)
o ?RV heave, No thrills
o Apex beat palpable ?deviated
o Auscultation: dual, ?AS/AR/PS/radiation to axilla
o There was a lot going on with this guy, so just be firm with your findings
and manage your time well
o Tetralogy of Fallot repair?
Green
o ?AF
o Very soft heart sounds

o Some could hear AS, others couldnt hear anything


Red
o Regular rhythm but very soft heart sounds as well
o Some could hear AS too but others couldnt hear anything
JVP VS CAROTID PULSE
Multiphasic
o The JVP "beats" twice (in quick succession) in the cardiac cycle.
o In other words, there are two waves in the JVP for each contractionrelaxation cycle by the heart.
o The first beat represents that atrial contraction (termed a) and second beat
represents venous filling of the right atrium against a closed tricuspid valve
(termed v) and not the commonly mistaken 'ventricular contraction'.
o These wave forms may be altered by certain medical conditions; therefore,
this is not always an accurate way to differentiate the JVP from the carotid
pulse.
o The carotid artery only has one beat in the cardiac cycle.
Non-palpable
o The JVP cannot be palpated.
o If one feels a pulse in the neck, it is generally the common carotid artery.
Occludable
o The JVP can be stopped by occluding the internal jugular vein by lightly
pressing against the neck.
o It will fill from above.
Varies with head-up-tilt (HUT)
o The JVP varies with the angle of neck.
o If a person is standing, his JVP appears to be lower on the neck (or may not
be seen at all because it is below the sternal angle).
o The carotid pulse's location does not vary with HUT.
Varies with respiration
o The JVP usually decreases with deep inspiration.
o Physiologically, this is a consequence of the FrankStarling mechanism as
inspiration decreases the thoracic pressure and increases blood movement
into the heart (venous return), which a healthy heart moves into the
pulmonary circulation.

CARPAL TUNNEL EXAM


Description of task:

Ms Jenny Roberts, a 30-year-old receptionist, has


presented to you complaining of a dull ache in her right
hand and forearm that has been present for the past few
months, and is getting worse. Her left hand is normal.

She has also been experiencing some pins and needles in


the same hand, that occurs concurrently with the pain

The pain is worst when typing at work, and at night.

Tasks for the students:


Perform an appropriate examination
Outline the likely diagnosis and subsequent management
Answer the patients questions and address her concerns

CARPAL TUNNEL EXAM


- Position patient in chair with hands out, resting on pillow
- Always compare the two sides

General inspection of hands:


- Atrophy of thenar eminence
- Scars previous carpal tunnel release?
- Evidence of underlying condition associated with CTS?
RA ulnar deviation of MCPs, swan necking, boutonnieres, Zdeformity
Hypothyroidism cool peripheries, slow pulse, hair changes
Psoriatic nail changes

Palpation:
- Palpate for tenderness/swelling/crepitus etc.;
Wrist
MCPs
IPJ

Special tests:
- Phalen and reverse Phalen maneuvers
- Tinel test
- Manual carpal compression test apply pressure over the
flexor retinaculum - +ve if paraesthesia occurs within 30
seconds, very sensitive
- Hand elevation test raise hands above head for 1
minute, +ve if symptoms are reproduced

Nerve screening:
- Median OK sign (thumb abduction and opposition)
- Radial make a gun shape
- Ulnar star

Movement and functional ability:

- Active and then passive movement wrists, MCPs,


thumb
- Typical deficits are weakness of thumb abduction and
thumb opposition (solely innervated by median nerve)
- Test power;
Grip strength
Thumb opposition strength
- Functional ability show me how you use a key, show me
how you undo a button

Sensation:
- Pinprick or cotton wool
- Should be tested in all regions of the hand, forearm and
upper arm
Typically sensory deficits involve the median-innervated fingers but
spare the thenar eminence (as this is supplied by the palmar brach
of the median nerve, which separates from the median nerve before
the carpal tunnel so is unaffected)
- Will affect median nerve distribution lateral 3.5 fingers

DIAGNOSIS:
- Carpal tunnel syndrome in right hand, likely due to
repetitive movement from typing at work
Carpal tunnel is the most common nerve entrapment syndrome,
where the median nerve is being compressed as it passes through
the carpal tunnel to reach the hand

INVESTIGATIONS:
- Typically a clinical diagnosis
- If in doubt;
Electromyography (EMG) will show focal slowing of conduction
Nerve conduction studies
TFTs, rheumatological testing for underlying cause

MANAGEMENT:
- A number of management options are available, and the
modality chosen depends largely on the severity of nerve
dysfunction (i.e. mild, moderate or severe)
- Mild- moderate CTS manage with conservative therapy;
Splinting typically worn at night, maintains wrist in neutral position
Oral/injected steroids
Yoga

Nerve-gliding exercises performed with OT


Ultrasound therapy - ultrasound and electrical stimulation have been
used to promote recovery after nerve and tendon injuries
- Severe CTS/non-responsive to conservative measures
require surgical release of carpal tunnel

PATIENT QUESTIONS:
- Why are some fingers not affected?
Only affects the fingers supplied by the median nerve, which are the
lateral 3.5 digits
- Why is it worse at night?
Due to nocturnal fluid redistribution when lying flat and retention
increased pressure in carpal tunnel
- Could it be my neck?
Possible but unlikely as the deficits are only in the median nerve (i.e.
a peripheral nerve) median nerve cant be damaged in the neck
- Why didnt Nurofen (NSAID) help?
Does nothing to reduce the pressure in the carpal tunnel, which
produces ischaemia in the nerve, thus causing pain
Therefore pain is still present, even with nurofen
- Should I have tests done?
If diagnosis clear no
If diagnosis unclear yes
Neurophysiology helps by confirming lesions site and severity, as
well as likelihood of improvement after surgery
If Hx features suggest an underlying cause, further Ix may be
necessary
- Could my hand get weak?
Weakness, if it does occur, would only be present in the movements
supplied by the median nerve (primarily movements of the thumb)
- What do you believe would be the best treatment?
See above, depends on severity

CARPAL TUNNEL SYNDROME

- A collection of symptoms and signs caused by compression of the


median nerve as it passes through the carpal tunnel under the flexor
retinaculum, to enter the hand
Epidemiology:
- Common disorder prevalence = 3.7%
- More common in women
- More common in white people
- Typically occurs in people who perform repetitive wrist movement
e.g. typists
Aetiology & Pathophysiology:

- Risk factors;

Female
Hypothyroidism
Obesity
Connective tissue diseases
Diabetes
Genetic predisposition
RA
- The final common pathway for most of these above risk factors is the
development of raised pressure within the carpal tunnel
This raised pressure ultimately leads to ischaemia, scarring of the nerve and
demyelination
- Ischaemia is likely to be the cause of the typical intermittent sensory
symptoms (i.e. numbness and pain)
- Raised pressure may also occur secondary to inflammation of one or more of
the 9 flexor tendons that pass through the carpal tunnel alongside the median
nerve
- Other possible causes of compression include;
Congenitally small anatomic space
Mass lesions (cysts, neoplasm, persistent median artery)
Oedema, inflammatory conditions e.g. RA

Clinical features:
- Common symptoms;
Dull, aching discomfort in hand, forearm or upper arm
Paresthesia in the hand (numbness and tingling)
Weakness/clumsiness of hand
Occurrence of these symptoms in a median distribution
- These symptoms typically have a precipitant;
Sleep symptoms often worse at night
Sustained hand or arm position
Repetitive actions of the hand or wrist
- Physical examination;
Atrophy of thenar eminence may be present
Sensation should be tested in all regions of the hand, forearm and upper arm
o Typically sensory deficits involve the median-innervated fingers but spare
the thenar eminence (as this is supplied by the palmar branch of the
median nerve that separates before the carpal tunnel)
Muscle weakness typically weakness of thumb abduction and thumb
opposition
Special tests;
o Phalen and reverse Phalen maneuvers
o Tinel test
o Manual carpal compression test apply pressure over the flexor
retinaculum - +ve if paraesthesia occurs within 30 seconds
o Hand elevation test raise hands above head for 1 minute, +ve if
symptoms are reproduced

Investigations:
- Electromyography (EMG) will show focal slowing of conduction
- Nerve conduction studies

Management:
- A number of management options are available, and the modality chosen
depends largely on the severity of nerve dysfunction (i.e. mild, moderate or
severe)
- Mild- moderate CTS manage with conservative therapy;
Splinting typically worn at night, maintains wrist in neutral position
Oral/injected steroids
Yoga
Nerve-gliding exercises performed with OT
Ultrasound therapy - ultrasound and electrical stimulation have been used to
promote recovery after nerve and tendon injuries
- Severe CTS/non-responsive to conservative measures require surgical release
of carpal tunnel

Complications:
- Surgical complications
- Recurrence

CRANIAL NERVE EXAM

ANSWERS
CN V (trigeminal)
o Sensation in a 3 divisions (ophthalmic, maxillary, mandibular)
o Masseter tone
o Temporalis tone
o Jaw jerk
CN VII (facial)
o Upper facial movements
o Lower facial movements
o Taste of anterior 2/3 of tongue
CN VIII (vestibulocochlear)
o Whisper 66,99
o Rinne + Weber test
o Hallpike manoeuvre
CN IX (glossopharyngeal)
o Pharyngeal sensation
o Gag reflex (afferent)
o Taste of posterior 1/3 of tongue
CN X(vagus)
o Uvula movement (note: deviates away from the side of the lesion)
o Gag reflex (efferent)
o Hoarsenss of voice
o Cough (recurrent laryngeal)
o Swallow
CN XI (spinal accessory)
o Shoulder shrug (trapezius)
o Turning head (SCM)
CN XII (hypoglossal)
o Tongue observation (wasting, fasiculations)
o Sticking tongue out (note: deviates to side of lesion if LMNL; unilateral
UMNL does not cause deviation due to duel innervation)

MARKING
SCHEME


DIABETIC
LOWER LIMB EXAM

STEM
This patient is diabetic and has come in for their yearly check up of
their lower limbs.

Task 1: Please perform a focused diabetic examination on the


patients lower limbs. DO NOT examine gait.

Task 2: Report your findings to the examiner (positive and negative)


either as you examine or at the 5 minute mark of this station. (The
examiner will tell you when 5 minutes have elapsed).

Task 3: Tell the examiner the most likely diagnosis.

Task 4 (if there is time)


Discuss other examinations findings in diabetes other than LL
Investigations

GENERAL APPEARANCE
Mental state - alert, drowsy
Body habits - obese
Endocrine facies - Cushings (moon)
Pigmentation - haemachromatosis (bronze)
Kussmaul breathing - DKA
Dehydration - DKA, acute presentation
EXAMINATION
INSPECTION
Skin
o Skin status: color, thickness, dryness, cracking, hair loss, sweating
o Infection check between toes for fungal infection
o Ulceration neuropathic, arterial or venous
o Calluses/blistering: hemorrhage into callus?
o Thighs insulin injecting sites get fat atrophy or hypertrophy
MSK
o Deformity claw toes, prom inent metatarsal heads, Charcot joint
o Muscle wasting quadriceps due to femoral nerve mononeuropathy
Infection
o Cellulitis
o Tinea
Dermatopathology
o Pigmented scars

Late diabetic dermopathy


Small rounded plaques with raised borders lying in a linear fashion
over the shins
o Necrobiosis lipoidica diabeticorum:
Reasonably specific skin manifestation of diabetes mellitus, but is
rare (in fewer than 1% of diabetics).
It is found over the shins, where a central yellow scarred area is
surrounded by a red margin when the condition is active. These
plaques may ulcerate.

VASCULAR
Temperature
Capillary return
Pulses
ABI
NEUROLOGICAL EXAM
Full neurological exam sensation (VIBRATION), motor, reflexes
Sensation exam monofilament (10g), light touch, pain, vibration (128hz),
proprioception
Proximal muscle power femoral nerve mononeuropathy

MARKING SCHEME

IMAGES OF FOOT
DEFORMITIES
A: Claw toe deformity. Note the
buckling phenomenon that
causes increased pressure on
the dorsal hammer digit
deformity, as well as on the
plantar metatarsal head.
B: Bunion and overlapping toes.
This deformity can lead to
pressure ulceration between the
digits, on the dorsal or plantar
surfaces of displaced digits, and
over the medial first
metatarsophalangeal joint.
C: A rocker-bottom deformity
secondary to Charcot
arthropathy can cause excessive
pressure at the plantar midfoot,
increasing risk for ulceration at
that site.

The Monofilament
This involves the use of a 10mg monofilament, and tests for the development of
peripheral sensation loss. When combined with clinical assessment, this is one
of the most sensitive and specific ways of identifying persons at risk of
developing foot ulceration.
Instructions for use:
Test the monofilament on a sensitive area of the patients skin (ie, the inside of
the forearm) and demonstrate what it should feel like
Ask the patient to close their eyes and say yes when they feel the
monofilament touching them

Place the monofilament at 90 degrees to the skin surface and allow it to bend
approximately 1cm. This should take 1-2 seconds. Ensure that you do not jab
the skin with the monofilament
Hold the monofilament in this position for 1-2 seconds and then slowly release
the pressure, over 1-2 seconds, until the monofilament is straight. Now remove
the monofilament from the skin surface
Repeat for all testing sites on both feet (see below)
If there are areas in which the patient does not respond, repeat the test in the
same place twice more, and if there is still no sensation, record a negative
response.

GROIN LUMP HISTORY & EXAM

Tim is a 26 year old footballer who presents to you, his local GP,
after noticing a lump in his groin.


Instructions to the student:
Please talk through a physical exam, telling the examiner what you
are looking for and asking for specific findings
Ask for investigations and their findings
Discuss appropriate management

EXAMINATION FINDINGS
General Inspection
o Exposed from the waist down

o Fit, healthy looking


o Skin lesion on right lower leg (ONLY IF SPECIFICALLY ASKED ABOUT)
Mole
Irregular border
Irregular colouring
2-3cm diameter
Suggestive of melanoma
SCROTUM
o Inspection
Lump in right inguinal region
1-2cm
Not erythematous
Scrotal skin otherwise looks normal
Testes hanging at equal length
o Palpation
Testes feel firm, mobile, no lumps palpable in scrotal sac
Non-tender to examination
Lumps in inguinal region
Stony hard
Non mobile
Non tender
Able to get above it
Doesnt connect with the scrotal sac
Not reducible
DIFFERENTIALS
Lymphadenopathy
o In this case, possibly secondary to melanoma
o Can also be from carcinoma of scrotal skin or local invasion from abdomen
o Infectious cause:
Orchitis
Inflammation of a joint groin mass
Lipoma
Inguina/femoral Hernia
o Very unlikely
Testicular Cancer
o Very unlikely
Aneurysmal swelling
INVESTIGATIONS
Ultrasonography of inguinal region/groin
o Suggestive of malignant lymph node
Beta-hCG
o Not elevated
AFP
o Negative
LDH
o Mildly elevated (elevated in many malignancies)
LFTs
o Normal (looking for liver metastases)
Biopsy of skin lesion

o Malignant melanoma
MANAGEMENT
o Biopsy of inguinal lymphadenopathy
o Positive for malignancy
o Inguinal node clearance
o Resection of skin lesion
o CT scan to look for distant mets and assist with staging
MARKING SCHEME
o MUST do a general inspection
o SHOULD enquire about lower limb lesions
o MUST discuss scrotal examination and SHOULD correctly identify that lesion is
likely to be a lymph node
o MUST order USS
o SHOULD order blood tests to rule out testicular carcinoma

INGUINAL LYMPHADENOPATHY
o The inguinal region drains:
o Lower part of vagina, vulva, penis, rectum, anus, skin of lower extremities,
scrotal skin

MALIGNANT MELANOMA STAGING


Prognosis depends on the disease stage at diagnosis, as follows:
Patents with stage I disease - 5-year survival rate of greater than 90%
Patients with stage II disease - 5-year survival rate ranging from 45-77%
Patients with stage III disease - 5-year survival rate ranging from 27-70%
Patients with metastatic disease have a grim prognosis, with a 5-year survival
rate of less than 20%.

Stage IA

Lesions less than or equal to 1 mm thick with no evidence of ulceration or


metastases (T1aN0M0) are associated with a 5-year survival rate of 95%.

Stage IB

Lesions less than or equal to 1 mm thick with ulceration noted but without
lymph node involvement (T1bN0M0) or lesions 1.01-2 mm thick without
ulceration or lymph node involvement (T2aN0M0) are associated with a 5-year
survival rate of approximately 91%.

Stage IIA

Melanomas greater than 1 mm but less than 2.01 mm in thickness with no


evidence of metastases but with evidence of ulceration (T2bN0M0) or lesions
2.01-4.0 mm without ulceration or lymph node involvement (T3aN0M0) are
associated with an overall 5-year survival rate of 77-79%.

Stage IIB

Melanomas 2.01-4 mm thick with ulceration but no lymph node involvement


(T3bN0M0) or lesions greater than 4 mm without ulceration or lymph node
involvement (T4aN0M0) are associated with a 5-year survival rate of 63-67%.

Stage IIC

Lesions greater than 4 mm with ulceration but no lymph node involvement


(T4bN0M0) are associated with a 5-year survival rate of 45%.

Stage IIIA

Patients with any depth lesion, no ulceration and 1 positive (micrometastatic)


lymph node (T1-4a,N1a,M0) have a 5-year survival rate of 70%. T1-4a,N2a,M0
lesions (any depth lesion, no ulceration but 2-3 nodes positive for
micrometastasis) are associated with a 5-year survival rate of 63%.

Stage IIIB

Patients with any depth lesion, positive ulceration, and 1 lymph node positive
for micrometastasis (T1-4b,N1a,M0) or 2-3 nodes positive for micrometastasis
(T1-4b,N2a,M0) have a 5-year survival rate of 50-53%. Patients with any depth
lesion, no ulceration, and 1 lymph node positive for macrometastasis (T14a,N1b,M0) or 2-3 nodes positive for macrometastasis (T1-4a,N2b,M0) have a 5year survival rate of 46-59%.

Stage IIIC

Patients with any depth lesion, positive ulceration, and 1 lymph node positive
for macrometastasis (T1-4b,N1b,M0) or 2-3 nodes positive for macrometastasis
(T1-4b,N2b,M0) or 4 or more metastatic lymph nodes, matted lymph nodes, or
in transit met(s)/satellite(s) have a 5-year survival rate of 24-29%.

Stage IV

Melanoma metastatic to skin, subcutaneous tissue, or lymph nodes with normal


LDH (M1a) is associated with a 5-year survival rate of 19%. M1b disease
(metastatic disease to lungs with normal LDH) has a 5-year survival rate of 7%.
M1c disease (metastatic disease to all other visceral organs and normal LDH or
any distant disease with elevated LDH) is associated with a 5-year survival rate
of 10%.

SCROTAL EXAM
It is essential to wear gloves for this procedure, for aesthetic and protective
reasons.
Inspect the genitals for evidence of mucosal ulceration. This can occur in a
number of systemic diseases.
If necessary, retract the foreskin to expose the glans penis.
Look for mucosal discharge, and if there is a history of discharge, attempt to
expel it by milking the shaft.
INSPECTION
Inspect the scrotum with the patient standing
Usually the left testis hangs lower than the right
Torsion of the testis may cause the involves testis to appear higher and lie on a
more transverse plane than normal

Inspect for:
Oedema of the skin
Sebaceous cysts
Tinea cruris
Scabies
Scrotal oedema
o Common in severe cardiac failure and may occur with nephrotic syndrome

PALPATION
Palpate each testis using the fingers and thumb of the right hand.
Normally equal in size, smooth and relatively firm

Absence of one or both testis can be due to:


Previous excision
Failure to descend
A maldescended testis has a high chance of developing malignancy

Orchitis is suggested by an exquisitely tender, indurated testis.


This is often due to mumps in post-pubertal patients

An undescended testis may be palpable in the inguinal canal.


Feel posteriorly for the epididymis and the upwards for the vas deferens and
the spermatic cord. It should be possible to differentiate the vas from the
testis.
A varicocele feels like a bag of worms in the scrotum. The testis on the side of
the varicocele often lies horizontally.
Causes of Genital Lesions
ULCERATIVE
o Herpes simplex (tender)
o Syphillis (non-tender)
o Malignancy (SCC) non tender

o Chancroid (Haemophilus ducreyi infection tender)


o Behcets syndrome
NON-ULCERATIVE
o Balanitis (due to Reiters syndrome or poor hygiene)
o Venereal warts
o Primary skin disease (eg. Psoriasis)
Differential Diagnosis of a Scrotal Mass
If there is a palpable mass, it is first important to see if you can get above it.
Have the patient stand up.
If no upper border is palpable, it must be descending down the inguinal canal
from the abdomen and is therefore an inguinal-scrotal hernia
If it is possible to get above the mass:
o Decide whether it is separate from or part of the testes
Check for transillumination
In a darkened room, apply a torch to the posterior part of the swelling

TRANSILLUMINATION
Cystic mass will transilluminate
Hydrocoele
Cyst of the epididymis
NON-TRANSILLUMINATION
Tumour
Possibility of chronic epipdidymitis
o A cystic-style lesion that is separate from the testes
Syphilitic gumma (rare)

INVESTIGATIONS OF SUSPECTED TESTICULAR CANCER

The first investigation is ultrasonography:

Of the scrotal contents


If this confirms a solid mass, further surgical investigation must occur

Preliminary staging investigations are usually performed beforehand:

Chest XRAY or CT chest to look for hilar node involvement and lung secondaries
Blood levels of tumour markers
o Useful for tracking residual or recurrent tumour metastases
o Blood levels closely correlate with tumour bulk

Tumour Markers

Beta-hCG (Human chorionic gonadotropin)


o Secreted by syncytiotrophoblastic cells
o Levels may rise in any tumour type, particularly in poorly differentiated
germ cell tumours
Alpha-fetoprotein (AFP)
o Produced by yolk-sac elements

o About 75% of patients with metastatic teratoma have elevated AFP levels,
but this is not expressed in seminoma
Lactic Dehydrogenase (LDH)
o Elevated in more than half of patients with metastatic seminoma
Tumour markers are repeated at interval during follow-up and are invaluable for
predicting the appearance of new metastases

Surgical Exploration

Orchidectomy is the only appropriate treatment for the primary tumour and is
usually performed as part of the diagnostic process.

There is inguinal incision to avoid the scrotal skin, the spermatic cord is
temporarily clamped to preclude venous spread of the malignant cells and the
testis is brought out for visual examination.

If the testis is obviously malignant, orchidectomy is performed. If there is doubt,


biopsy is taken and examined.

The other testis is usually unaffected and its function can be preserved.

HEARING ASSESSMENT

A patient presents to you complaining of hearing loss.


Perform an appropriate examination limited only to the ears and
hearing. You can describe your examination as you proceed and you
will be asked questions at the conclusion of your examination.
You will receive a warning when 5 minutes have passed.

INSPECTION OF EXTERNAL EAR


- Lightly pull pinna to test for tenderness and inspect pre-auricular
skin
- Scars, cauliflower ear
- Use otoscope to assess external ear for;
Wax, other obstruction
Infection
Evidence of eardrum inflammation bulging, loss of light reflex, red,
obvious exudate
Eardrum perforation

Using an otoscope:
- In adults, retract the pinna outwards and backwards to allow easier
insertion
- Hold otoscope like a pencil, resting your hand gently against the
patients cheek allows anchoring
- Gently adjust otoscope to allow visualization of entire external ear
and eardrum
- Additional tests with an otoscope;
Pneumatic otoscopy special attachment on otoscope which blows
air against eardrum; allows assessment of tympanic membrane
mobility to aid in diagnosing perforations, middle ear fluid or
tympanosclerosis

http://www.youtube.com/watch?v=FE0sot4OoAE

HEARING TEST
- Place one hand next to one ear and move fingers as a distraction
whilst whispering in the other ear

- Whisper numbers (66, 99) into the other ear and ask patient to
repeat what they heard
- Repeat in opposite ear

RINNE & WEBERS TESTS


- Rinne test;
Where does it sound louder?
Place a vibrating 512 Hz tuning fork on the mastoid process behind
the patients ear and ask if they can hear the vibrations
Tell patient to inform examiner when vibrations can no longer be
heard/felt at this point move tuning fork inline with external
auditory meatus
Vibrations should still be heard at EAM i.e. air conduction is
better than bone conduction
Rinne +ve if sensorineural deafness present, both air and bone
conduction are equally reduced, so air conduction is still better
Rinne ve if patient has conductive deafness, bone conduction will
be better than air conduction in affected ear, thus no note will be
audbile at EAM
- Webers test;
Is the buzzing louder on one side?
Place a vibrating 256 Hz tuning fork on the centre of the forehead
Ask patient if vibrations are felt/heard best in the centre of the head
or if there is localization to one ear
Normally the sound is heard in the centre of the forehead
Sensorineural deafness causes lateralization to normal ear,
conductive deafness causes lateralization to affected ear

Interpretation:

SENSORINEURAL HEARING LOSS:

- Results from defects central to the oval window in the cochlea (sensory),
cochlear nerve (neural) or rarely more central pathways
- Causes;

Congenital;
o Cochlear aplasia
o Chromosomal syndromes (rare)
o Congenital cholesteatoma
o Congenital rubella syndrome
Acquired;
o Long term exposure to environmental noise
o Ototoxic drugs streptomycin, vancomycin, aminoglycosides (esp.
gentamicin), chloroquine, hydroxychloroquine
o Postinfective measles, meningitis, mumps, flu, herpes, syphilis
o Cochlear vascular disease sudden hearing loss
o Physical trauma
o Menieres disease
o Tumours acoustic neuroma, meningioma

OBSTRUCTIVE HEARING LOSS:

- Impaired sound transmission via the external canal and middle ear ossicles to
the foot of the stapes
- Causes;
External canal obstruction wax, pus, debris, foreign body, developmental
anomalies
Drum perforation
Problems with ossicular chain otosclerosis, infection, trauma
Inadequate Eustachian tube ventilation of middle ear

KNEE EXAM

Description of task:
Max Johnson is a 65-year old man who has come to see you with
pain in both knees.

The pain is exacerbated by walking and he is having trouble walking


more than about 50 metres at present.

He twisted his right knee a few days ago and the pain in this knee
has been much worse since then.

Perform an appropriate focused examination of Mr Johnsons knees


to assess the severity of the problem.

LOOK:
- Compare the two knees
- With patient standing;
Valgus/varus deformity;
o Valgus = knees point in, knock knees
o Varus = knees point out, bow leg
Assess gait antalgic, stiff knee, unstable, locking
Squatting is it painful? Screening for meniscal tear
- With patient supine;
Swelling, erythema of knees
Scars
Wasting of quadriceps can use measuring tape to assess muscle bulk
Fixed flexion deformity

FEEL:
- Patient supine, begin with good knee (if there is a good knee)
- Warmth
- Palpate anatomy systematically with knee flexed feel for tenderness;
Along ligaments
Along joint line for meniscal or chondral injury
Popliteal fossa Bakers cyst
- Palpate for effusion;
Bulge test for small effusions
o http://www.youtube.com/watch?v=RMp6HOnS7Q0
Patellar tap
Cross fluctuance only for large effusions

MOVE:
- Active movement flexion and extension
- Passive movement flexion and extension, palpate for crepitus
- Painful movement
- Assess power

SPECIAL TESTS:
- ACL tests always comment on the endpoint
Lachman test;
o Most sensitive test
o Quads must be relaxed
o Flex knee and isolate joint with one hand on distal femur and the other on
the proximal tibia sharply push knee backwards and forwards to

determine if there is a strong endpoint


o http://www.youtube.com/watch?v=gfN-p-xZx24
Anterior drawer test sit on patients foot with knee flexed
- PCL posterior drawer test
- Collateral ligaments hold patients leg in armpit with knee slight flexed and
apply varus and valgus stress
- Meniscus;
McMurrays test flex knee with one hand on knee and other hand on patients
heel
o Alter degree of flextion whilst everting and inverting foot
o Positive test is feeling a click
o http://www.youtube.com/watch?v=uKvQ_6C3U_o
- Finish by assessing pulses

IN THIS PATIENT:

- Chronic pain likely due to long-standing OA


- Acute pain following twisting injury;
ACL tear/rupture;
o Acute pain
o May report a pop noise at time of injury
o May be unable to weight bear, knee feels very unstable
o Large effusion, swollen (haemarthrosis)
o Diffuse pain
o Reduced range of motion
o Anterior drawer test +ve
o Lachmans test +ve
Meniscal tear;
o Acute and ongoing severe pain, swelling over hours-days, avoidance of
activity
o +ve McMurrays test
o Pain along joint line, most commonly medially
o Patients may complain of knee locking or giving way
Combination injury (e.g. ACL + meniscus etc.)

PARKINSONS EXAM

Betty Smith is a 85 year old woman with a long-standing history of


Parkinsons disease. She has come to see her GP as she believes her
condition is worsening. Perform a focused examination on this
patient and report your findings.
Describe the management options of Parkinsons disease.

GENERAL INSPECTION:
- Hunched posture
- Limited spontaneous movement
- Facial expression;
Masked facies expressionless
Serpentine stare, reduced blinking
Dribbling
- Speech;
Hypophonia
Slow, monotonous
- Autonomic dysfunction;
Postural hypotension

EYES:
- Test eye movements to exclude Parkinsons plus syndromes;
Progressive supranuclear palsy vertical gaze palsy

Multiple system atrophy cerebellar eye disturbances (e.g. gaze-evoked


nystagmus); will also have autonomic dysfunction
NEUROLOGICAL: (upper and lower limbs)
- Assess for tremor;
Typically in hands;
o Resting pill-rolling tremor
o 4-8 Hz
o Improves with movement
Jaw
Worse when performing serial 7s
- Tone;
Leadpipe rigidity
Leadpipe + tremor cogwheeling
- Coordination;
Slow (but accurate) finger to nose test
- Reflexes;
Glabellar tap (primitive reflex) positive persistent blinking when tapping above
eyes
- Motor assessment;
Difficulty initiating movements (akinesia)
Slow movements (bradykinesia)
o Wave test, finger to finger test
o Progressive reduction in speed and amplitude of repetitive actions
- Power, sensation and reflexes generally normal

GAIT:
- Akinesia
- Bradykinesia difficulty getting out of seat
Timed up and go time to get out of chair and walk 3 metres, normal is <8-11
seconds
- Shuffling, festinating gait
- Retro/anteropulsion can be tested by pulling patient towards you
- Reduced arm swing on one side
- En-bloc turning (multiple steps required to turn)
- Instability
- Difficulty to stop
- Kinesia paradoxica able to perform fast movements but not slow ones;
E.g. patient is able to run down stairs in response to a fire alarm but may not be
able to stop
Not a recommended test but should be mentioned

ADDITIONAL TESTS:
- Writing micrographia, writing progressively gets smaller in the sentence
- MMSE;
Dementia
Depression


Investigations:
- No diagnostic tests for PD exist, therefore typically a clinical diagnosis
- Possible investigations;
MRI most likely normal
Genetic testing if familial PD suspected
Tests for Wilsons disease (serum ceruloplasmin, 24 hour urine copper)

Management:
- Levodopa dopamine precursor for old patients
Give with peripheral decarboxylase inhibitor to prevent peripheral DA
conversion
First-line for idiopathic PD, unless patient is <60 years
Does not alter natural progression of PD
Commencement delayed until patient experiences loss of function, to delay
wearing off phenomenon of L-dopa
Issues with long-term therapy;
o L-dopa induced involuntary dyskinesias
o Resistance wearing off of effects (40% at 5 years, 60% at 10 years of
therapy)
Other adverse effects psychotic features, nausea and vomiting, orthostatic
hypotension, confusion
- Dopamine D2 receptor agonists for young patients
Used as alternative to L-dopa
Less benefit than L-dopa but have longer duration of action
- Enzyme inhibitors;
COMT inhibitors given with L-dopa in late stage disease to delay wearing off
MOA-B inhibitors given when L-dopa is starting to wear off to prolong effects
- Others;
Amantidine NMDA receptor antagonist, enhances DA release and reduced
uptake
Anticholinergics may improve tremor
- Surgical;
Deep brain stimulation (response in 71%)
Pallidotomy
Thalamotomy
- Psychological therapy manage dementia, depression etc.
- Education

PVD EXAM

Description of task:

Mr Brian Watson, a 73-year-old retired builder, has come to you (his


GP) as he has been experiencing pain in both legs when walking. It

develops after walking approximately 50 metres on the flat, and is


relieved completely with rest.

He has a history of coronary artery disease and MI, which occurred


10 years ago.

Tasks:
Perform an appropriate examination of this mans lower limbs
Describe what investigations you would order for this mans
presentation.

PERIPHERAL VASCULAR EXAM

- Performed with patient supine


- Always compare both sides

General inspection:
- Patient able to comfortably move onto bed?
- Body habitus overweight/obese
- Any obvious respiratory distress etc.

Inspection of legs:
- Colour of skin pale/purple/black?
- Trophic changes;
Distal hair loss
Temperature change
Shiny, dry, scaly skin
- Muscle atrophy, scars
- Ulcers, gangrene check whole foot and ankle (inc. between toes, heel, sole)
Arterial punched out, painful lesions over pressure areas/bony prominences
Venous irregular, moderately painful lesions over gaiter area
Neuropathic (diabetic) deeply penetrating, painless lesions over pressure
areas
- Venous changes;
Varicose veins (best seen with patient standing)
Venous ulcers, thrombophlebitis
Haemosiderin deposition, venous eczema

Palpation of legs:
- Temperature change;
Start distally and move up leg
Compare both sides simultaneously
- Capillary refill of nail bed normal <2 seconds
- Palpate aorta and peripheral pulses start distally and move up
- +/- check for radio-femoral delay to exclude coarctation of aorta

Buergers test:
- Test for arterial insufficiency
- Raise leg allowing to 45 degrees, allowing blood to drain from leg
In a leg with normal circulation, the foot and toes will remain pink, even with leg
rasied to 90 degrees
In a leg with arterial insufficiency, the foot/toes will become pale
- Then ask patient to dangle leg over side of the bed
In presence of PVD, foot will become pink-red and painful (reactive hyperaemia)

Auscultation:
- Auscultate for renal and femoral bruits

Other tests:
- Trendelenburgs test to assess level of venous insufficiency in patients with
varicose veins
Patient lies supine and leg is flexed at hip to allow emptying of veins

Tourniquet/hand pressure is then applied at the sapheno-femoral junction (2cm


below and 2cm lateral to pubic tubercle)
Patient then asked to stand rapid filling of varicosities with tourniquet still on
suggests incompetence is below SFJ
Depending on result, tourniquet can be raised or lowered and test repeated to
determine level of incompetence
- Sensory examination

INVESTIGATIONS

- Doppler US of lower limb;


Assess blood flow in lower limb, as well as location and degree of stenosis
Normal pressure is slightly above brachial systolic
Patients with claudication range from 50-120 mmHg
Peak systolic velocity >2 = stenosis >50%
- Ankle brachial index (ABI);
Ratio of BP in lower limbs compared to BP in upper limbs i.e. ABI = systolic BP of
dorsalis pedis (or posterior tibial)/systolic BP of brachial artery
Assessed via Doppler US
Results;
o 0.9-1.2 = normal
o 0.5-0.9 = moderate arterial disease intermittent claudication
o <0.5 = severe arterial disease ischaemic rest pain, gangrene
o >1.2 = abnormal/artificially elevated result due to arterial calcification in
PVD causing artificial patency of vessels
- Toe pressure test;
Performed if ABI normal but patient is clinically abnormal, or if ABI >1.2
If toe pressure is <30mmHg in a non-diabetic with an ulcer, it suggests this will
not heal
- Arteriography (CT or MRI);
Reserved for patients thought to require intervention in the form of angioplasty
or reconstructive surgery
Provides map or arterial system, showing sites and severity of stenosis
Does not measure rate of blood flow
- CBE to exclude polycythaemia or thrmocythaemia (hypercoagulability
disorders)

Instructions to student:
This patient presents for a respiratory examination requested by an insurance
company.

RESPIRATORY EXAM

NOTE: There are no abnormal findings in the head and no palpable


lymph nodes in the neck. Thus, You DO NOT need to examine the
head. DO NOT examine for lymph nodes in the neck."

Task 1: Please examine the patients periphery and chest from a


respiratory viewpoint.

Task 2: Report your findings to the examiner (positive and negative)


as you examine (PREFERRED) or at the 5 min mark of this station

Task 3: Tell the examiner the most likely diagnosis before the 6 min
bell.

Examination Findings
Peripheries
o No clubbing
o Tar staining present
o Cyanotic
Chest
o Inspection
Barrel chest
Scar
Need to look under arm pneumonectomy scar on R side
Chest asymmetry
o Palpation
Tracheal deviation to the right
Decreased chest movements on the right
o Percussion
Dullness to percussion over entire right side
o Auscultation
Reduced/absent breath sounds over right side
Left sided breath sounds soft, coarse early inspiratory crackles
Vocal resonance reduced on right side
Differentials
o Pneumonectomy
Would be thinking lobectomy if trachea was not deviated and if
entire lung field not involved
o Pleural effusion
Stony dull percussion note
decreased vocal resonance
o Large malignancy

Address

Position
Patient initially sitting at 90 degrees
Patient at 45 degrees for anterior chest examination

General inspection
Patient distress/anxiousness
Scarring
Disfigurement

Hands
Peripheral cyanosis
Clubbing
Small muscle wasting
Palmer creases

Eyes
Pupil constriction
Conjunctivae pallor
Singular droopy eyelid

Nose
Deviated septum
Polyps
Enlarged turbinates

Mouth
Central cyanosis
Tonsil enlargement

Tar staining
Pulse
Respiratory rate

Respiratory
Exam
Overview
Introduction
Name
DOB

Redness

Tongue
Colour
Dental problems

Sinuses - Palpation
Trachea
Trachea midpoint placement

Lymph nodes
Lymph node palpation

Posterior Chest
Chest expansion
Percussion-midline, connection,
axillary

Anterior Chest
JVP
Percussion
Auscultation-breath sounds right, mitral valve left

Auscultation-midline, connection,
axillary
Vocal resonance

RESUSCITATION

You are an intern on a cardiology ward, and you have been called to
see one of your patients who has reportedly just collapsed. The
patient has a history of unstable angina.

On arrival in the patients room, you find the patient has indeed
collapsed and the panicking nurse is asking you to quickly assess the
patient.
Demonstrate on the model how you would go about assessing and
managing this collapsed patient.

PHASE 1 BASIC LIFE SUPPORT


DRS ABCD

Danger:
- Common dangers on the ward;
Bodily fluids urine, vomit, blood
Cables, wires tripping hazards
Furniture around bed TV, table, chairs etc.
Sharps
Family members
Response:
- Begin verbally John, can you hear me?
- If no response, attempt painful stimuli;
Press shoulders
Press fingernails
Sternal rub
- Look for signs of life conscious, responsive, breathing spontaneously, moving

This patient is non-responsive

Send for help:


- ASAP, even before assessing danger/response
- Methods on the ward;

Call code blue/MET call


Tell nurse to send for help
#33 on bedside telephone to speak to switch to call code blue
Airway:
- Open airway to look for airway obstruction

No visible airway obstruction in this patient

Breathing:
- Is normal breathing present? look for chest rise and fall, listen, feel chest
movement
- Chin lift/jaw thrust

Patient is not breathing

Compressions:
- Ratio of 30 compressions to 2 breaths
Aim for 100 compressions/minute compress to Staying Alive or Row, row,
row your boat
Compress 1/3 vertical height of chest
Place hands at lower 1/2 of sternum
- During CPR;
Ensure defibrillator is available/is on its way
Oxygen
Waveform capnography
IV/IO access
Plan actions before interrupting compressions
Defibrillator:
- Attach defibrillator leads and follow instructions
- Will indicate whether heart rhythm is shockable or non-shockable;
Shockable VT, VF
Non-shockable asystole, pulseless electrical activity (PEA)
- Transition to advanced life support

Patient is found to be in VF what now?

PHASE 2 ADVANCED LIFE SUPPORT

- After attaching AED, will indicate whether patient is in shockable or nonshockable rhythm
Shockable rhythm: VF, VT
- Administer shock
- Immediately resume compressions for 2 minutes to ensure perfusion
- Re-shock if required
- Adrenaline 1mg of 1:10000;
Give after 2nd shock
Then give after every 2 CPR cycles (i.e. every 2 minutes)
- Amiodarone 300mg give after 3rd shock if persistent shockable rhythm

Non-shockable rhythm: Asystole, PEA


- Give immediate adrenaline 1mg
Then after every 2 CPR cycles
- Immediately recommence compressions for 2 minutes
- Reassess rhythm;
Still non-shockable continue
Shockable see above
If still unresponsive after numerous cycles:
- Consider and correct 4Hs and 4Ts
Hypoxia
Hypovolaemia
Hypo/hyperthermia
Hypo/hyperK+, other metabolic derangements
Tension pneumothorax
Tamponade
Toxins
Thrombosis
Post-resus care:
- Re-evaluate ABCDE
- 12-lead ECG
- Treat precipitating cause
- Re-evaluate oxygenation and ventilate
- Temperature control (cool)

RHEUMATOLOGICAL HAND EXAM

Maureen Rogers is a 45 year old lady who presents to you with a


three month history of painful hands when she wakes up in the
morning.
Your Tasks:
Please examine her hands (4 minutes)
Tell the examiner which investigations you would like to perform and
their findings. From this, discuss possible management options for
this lady (2 minutes)
You will be warned when 4 minutes has passed.

PHYSICAL EXAM FINDINGS


General Inspection
o Ulnar deviation of fingers
o Z-line deformity of thumbs
o MCP and PIP joints red and swollen
o No scars
o No rheumatoid nodule
Effusions of MCP and PIP joints on both hands
o Hot to touch
o Boggy
Reduced grip strength
Wrists not involved
Movements:
o Decreased ability to move fingers painful
o Wrists have near normal ROM
INVESTIGATION FINDINGS
CBE with differential and platelet count
o Normocytic normochromic anaemia
LFTs
o Normal
EUC
o Normal
Serum uric acid
o Normal
RA
o Positive
Anti-CCP
o Positive
ESR and CRP
o Both elevated
ANA
o Negative
XRAY of hands
o Bilateral and symmetric
o Periarticular or diffuse osteoarthritis (diffuse OA is only seen in RA)
o Periarticular soft tissue swelling
o Marginal erosions
Extension of pannus from synovial reflections onto the bone
o Joint space narrowing
MANAGEMENT
Up to Date recommends that all patients be started on a DMARD at diagnosis.
In patients with mildly active RA:
o NSAIDs for rapid inflammatory relief

o DMARD treatment with either Hydroxychlouroquine or Sulfasalazine


In patients with moderately to severely active RA:
o Initiate with either a glucocorticoid or NSAID depending on disease
severity
o DMARD treatment with methotrexate
If unresponsive to DMARD can add TNF-alpha inhibitor or use an alternative
DMARD

DEFINITION OF MODERATELY TO SEVERELY ACTIVE RA Patients with moderately to severely active


RA are at greater risk of developing joint damage and disability than patients with mildly active disease [13].
These patients typically meet the American College of Rheumatology (ACR)/European League Against
Rheumatism (EULAR) criteria for RA and exhibit a combination of the following:

At least five inflamed joints

Elevation in the erythrocyte sedimentation rate (ESR) and/or serum C-reactive protein (CRP) concentration

Positive rheumatoid factor and/or anticyclic citrullinated peptide (CCP) antibodies (present in most patients)

Evidence of inflammation on plain radiography of the hands, wrists, or feet, such as


osteopenia and/or periarticular swelling. In addition, minimal joint space narrowing and small peripheral
erosions may be observed.

RHEUMATOID ARTHRITIS

Rheumatoid arthritis is an inflammatory disease of unknown aetiology, marked


by a symmetric, peripheral polyarthritis. It is 3-5 times more common in
women than in men

CLINICAL FEATURES

The incidence of RA increases between the ages of 25-55 years. After this point,
it plateaus until 75 years, after which time it begins to decrease.
Early Symptoms:
Morning stiffness of > 1 hour and easing with physical activity
First joints to be involved are typically the small joints of the hands and feet
The initial pattern of joint involvement is usually symmetrical
o Can be mono, oligo or polyarthritis

Once the disease has been established, symptoms tend to progress:


The wrists, MCP and PIP joints are the most frequently involved joints
o The hips and shoulders are rarely affected
DIP involvement may occur, but this is normally a manifestation of co-existing
osteoarthritis
Flexor tendon tenosynovitis
o Frequent manifestation
o Results in decreased ROM, reduced grip strength and triggerfingers
Ulnar deviation of the fingers
o Swelling of the MCP joints causes the fingers to become displaced and
deviate towards the ulnar side of the body
Swan-neck deformity
o Hyper-extended PIP joint and a flexed DIP joint
o Chronic inflammation of the PIP joint causes laxity of its supporting
ligaments, allowing for hyper-extension to occur. This causes the flexor

tendon to become unbalanced flexion of the DIP to balance out the


forces (?)

Boutonnieres deformity
o This shows PIP hyperflexion and DIP hyperextension (ie, essentially the
opposite of swan-neck deformity)

Z-line deformity of the thumb


o This shows hyper-extension of the interphalangeal joint of the thumb
Subluxation of the distal ulna
Involvement of the feet:
o MTP joint involvement is an early feature of disease
o Ankle and midtarsal involvement usually occurs later and often
predisposes to pes planus
Atlanto-axial involvement
o This has the potential to cause compressive myelopathy and neurological
dysfunction
o The prevalence of this is now declining and occurs in <10% of patients
o RA is therefore important to consider when consenting someone for an
anaesthetic procedure
It does not affect the thoracic or lumbar spine

SHOULDER EXAM 1

Description of task:

Mr John Brown, a 51-year-old mechanic has come to your GP practice


complaining of pain in his right shoulder that has been gradually
worsening over the past 6 months.

On further history, Mr Brown explains the pain is worse at night and


limits his ability to work as he experiences pain when lifting and
working overhead. He also explains his right shoulder feels stiff. The
left shoulder is normal.

Tasks;
Perform appropriate examination of Mr Brown
Explain likely diagnosis
Outline investigations and management

Shoulder Exam

Look: (Inspect from the back and front)


Asymmetry of shoulders
Scars, swelling, erythema
Wasting of pectoral muscles
Normal contour of deltoid
Normal clavicle shape
Feel:
Palpate anatomy
Sternoclavicular joint clavicle Acromioclavicular joint (osteoarthrosis is
common here) palpate down to the Glenohumoral joint subacromial space
coracoid process supraspinatus body infraspinatus body spine of
scapula borders of scapula
Heat, swelling
Move: (Begin with active movement then assess passive movement)
Assess for limitation, crepitus, pain
Abduction;
o Compare the two sides
o Assess for presence of painful arc (impingement syndrome) pain on
abduction between 60-120 degrees
Forward flexion
Internal/external rotation;
o Arms abducted with elbows flexed at 90 degrees (external rotation)
o Arm behind back (internal rotation)
Special tests: (Test power of rotator cuff muscles)
Supraspinatus;
o Empty can test
o Neer test forcefully elevate internally rotated arm in scapular pain,
causing supraspinatus tendon to impinge against anterior inferior
acromion - http://www.youtube.com/watch?v=U8-yLHQ_JaM
o Hawkins-Kennedy test forcefully internally rotate a 90 degree forwardly
flexed arm, causing supraspinatus tendon to impinge against
coracoacromial arch http://www.youtube.com/watch?v=OYK5qL2om-c
Infraspinatus/teres minor power of external rotation
Subscapularis lift off test (internal rotation)
Serratus Anterior (and long thoracic nerve);
o Ask patient to lean and press against wall
o Look for lifting off of the scapula
Shoulder instability apprehension test
In this patient:

Normal appearance of both shoulders, no asymmetry


Stiffness in all movements of R shoulder
Painful arc pain on abduction of R shoulder between 60-120 degrees
Pain on special tests

DIAGNOSIS:
- Impingement syndrome (AKA chronic tendinitis)
Impingement of rotator cuff by structures of the shoulder
- Investigations;
X-ray may show calcification just above greater tuberosity
Ultrasound and MRI may show changes in the cuff
- Management;
Begin with short course of NSAIDs
If this fails, local injection of corticosteroid (methylprednisolone) and lignocaine
is tried
If symptoms continue, surgical decompression may be necessary

Impingement syndrome

- AKA chronic tendinitis


- Refers to the encroachment of the rotator cuff by the structures of the
shoulder resulting in shoulder pain due to;
Supraspinatus tendinitis and oedema (secondary to tears, rupture, over-use)
most common cause
Osteophytes on under-surface of acromioclavicular joint
Impingement against coracoacromial arch during abduction

Epidemiology:
- Usually occurs in patients aged 40-60
- Incidence increases with age

Aetiology & Pathophysiology:


- There are 3 stages in the spectrum of rotator cuff impingement;
Stage 1 commonly affects patients <25 years, impingement due to acute
inflammation, oedema and haemorrhage in rotator cuff
Stage 2 affects patients aged 25-40 years, rotator cuff tendon progresses to
fibrosis and tendonitis
Stage 3 affects patients >40 years, progression of the condition may lead to
mechanical disruption of the rotator cuff tendon and to changes in the
coracoacromial arch with osteophytosis along the anterior acromion

Clinical features:
- Shoulder pain;
Often felt over deltoid muscle
Worse at night
Limits functional ability (e.g. pain when lifting overhead, putting on a jacket etc.)
- Physical examination;
Normal appearance
Stiffness in shoulder movement

Painful arc pain on abduction between 60-120 degrees. Above 120 degrees,
there is involvement of the scapula in abduction, thus no pain is elicited
Special tests from impingement;
o Neer test forcefully elevate an internally rotated arm in the scapular
plane, causing supraspinatus tendon to impinge against anterior inferior
acromion
o Hawkins-Kennedy test forcefully internally rotate a 90 degree forwardly
flexed arm, causing supraspinatus tendon to impinge against
coracoacromial arch
o Supraspinatus isolation test/empty can test rotate arm so thumbs point
to the floor and apply resistance; weakness in affected arm suggests a
positive test
Investigations:
- X-ray may show calcification just above greater tuberosity
- Ultrasound and MRI may show changes in the cuff

Management:
- Begin with short course of NSAIDs
If this fails, local injection of corticosteroid (methylprednisolone) and lignocaine
is tried
- If symptoms continue, surgical decompression may be necessary;
The coracoacromial ligament and anterior-inferior part of acromion are excised
Osteophytes are removed if present and small tears are repaired

Complications:
- Shoulder instability
- Osteoarthritis

SHOULDER EXAM 2

truction to Students
You have 6 minutes at this station
Description of task:
This patient has left shoulder pain.
Please examine the shoulder and explain your actions and
findings to the examiner as you perform them.
Confine your examination to the shoulder girdle
Answer the examiners questions based on X-ray provided

XRAY interpretation
Plain Xray of left shoulder joint in AP and lateral views taken of Mr X X on DATE
Displaced proximal humerus fracture
Soft tissue swelling
IGNORE GROWTH PLATE

SHOULDER EXAMINATION
LOOK: Skin, soft tissue, bone
FEEL: Skin, soft tissue, bone
MOVE: active, passive, special tests

THYROID/NECK EXAM

A 33-year-old otherwise well female presents with an incidental


ultrasound finding of a 2 cm lump in the right lobe of the thyroid.
Please perform a suitable examination.

The Basic Procedures;


1. Inspection
2. Palpation
3. Percussion
4. Auscultation

Thyroid Exam

1. Inspection
- Normal thyroid may just be visible in a thin young person below the carotid
cartilage usually on the isthmus is visible as a diffuse central swelling
- Inspect for; (from front and sides)
Asymmetry goiter, nodule (multiple or solitary)
Swelling localized or general
Scars thyroidectomy (ring around base of neck)
Enlarged lymph nodes around thyroid
- Ask patient to swallow small sips of water and watch the swelling carefully
Only a goitre or thyroglossal cyst will rise during swallowing (due to attachment
to larynx)
2. Palpation
- Begin palpation of thyroid gland from behind
- Palpate both lobes of the gland and its isthmus
- Assess;
Size only an approximation is possible; carefully feel for a lower border
Shape note whether gland is uniformly enlarged and whether the isthmus is
affected; if a distinct nodule is felt, determine location, size, consistency,
tenderness and mobility
Consistency may vary; soft is normal, firm in simple goiter, rubbery hard in
Hashimotos thyroiditis, stony and hard suggests carcinoma, calcification in a
cyst or fibrosis
Tenderness a feature of thyroiditis
Mobility carcinoma may tether the gland
Location can you get below it? (i.e. retrosternal)
- Repeat assessment while patient swallows
- Palpate for thrills over the gland
- Palpate the cervical lymph nodes
- Palpate carotid arteries absence of pulse indicated malignant infiltration by
thyroid carcinoma
- Palpate the trachea to assess its position
3. Percussion
Percuss upper part of manubrium from one side to the other change in
resonant to dull suggests goitre unreliable sign
4. Auscultation
Listen over each lobe for a bruit
o Bruit is a sign of increased blood supply which may occur in
hyperthyroidism
May be a carotid bruit
5. Additional examination
- Thyroid state explain to examiner you would also want to assess;
Hands tremor, palms, nails
Pulse rate and regularity
Eyes lid lag, exophthalmos
Deep tendon reflexes
- Pembertons sign;

Test for thoracic inlet obstruction due to a retrosternal goitre or any retrosternal
mass
Ask patient to lift both arms above head
Wait a few moments and assess face for signs of congestion (plethora) and
cyanosis
Assess neck for distension from venous congestion

TRAUMA

This is a young patient who has been involved in a high speed MVA.
Please complete a primary survey of this patient and discuss
immediate management as you examine.

THE PRIMARY SURVEY

First, carry out the ABCDE survey:


Airway

Breathing

Circulation

Disability

Exposure.

The ABCDE survey is sometimes referred to as the primary survey. Its primary
function is to diagnose and treat life threatening injuries which, if left
undiagnosed and untreated, could lead to death:
Airway obstruction

Chest injuries with breathing


difficulties
Severe external or internal
haemorrhage
Abdominal injuries.

When more than one life threatening state exists, simultaneous treatment of
injuries is essential and requires effective teamwork.
Airway

Assess the airway. Can the patient talk and breathe freely? If obstructed,
consider the following steps.
Chin lift/jaw thrust (tongue is
attached to the jaw)
Suction (if available)
Guedel
airway/nasopharyngeal
airway
Intubation; keep the neck
immobilized in neutral
position.

Breathing

Assess airway patency and breathing adequacy by clinical observation.


Palpate trachea
Listen for tension pneumothorax
If inadequate, consider:
Artificial ventilation
Decompression and drainage
of tension
pneumothorax/haemothorax

Closure of open chest injury.


Reassess the ABCs if the patient is unstable.
Circulation

Assess the patients circulation as you recheck the oxygen supply, airway
patency and breathing adequacy. If inadequate, you may need to:
Stop external haemorrhage
Establish 2 large-bore IV
lines (14 or 16 G) if possible
Administer fluids, if
available.
Give oxygen, if available.
Disability
Make a rapid neurological assessment (is the patient awake, vocally responsive
to pain or unconscious?) There is no time to do the Glasgow Coma Scale (page
PCTM 23) so use the following clear, quick system at this stage:
A Awake
V Verbal response
P Painful response
U Unresponsive
Exposure
Undress the patient and look for injury. If you suspect a neck or spinal injury, inline immobilization is important.
Take care when moving the patient, especially if he or she is unconscious as the
C-spine has not yet been cleared.

Airw
ays

Brea
thin
g
Circ
ulati
on

ANSWERS
Basically, this involves completing a primary survey
Clear airway
Jaw thrust
Oropharyngeal airway
C spine hard colour
Inspect accessory muscles, recession, RR
Check position of trachea
Percuss and auscultate tension pneumothorax, flail chest, sucking chest wounds
Give high flow oxygen
Signs of external haemorrhage
Peripheral perfusion skin colour, capillary refill, temperature
PR, BP, neck veins
O2 saturations
Insert 2 large bore IV

COUNSELLING

SPIKES

Setting up the interview


o Arrange for a private encounter
o Involve significant others
o Sit down
o Maintain eye contact
Assess the patients perceptions
o Assess how the patient perceives their current situation
o What have you been told about your condition so far?
Obtain the patients invitation
o Find out at what level the patient wishes to have the
information disclosed details vs big picture
Pass along knowledge
o Warning shot eg I have some bad news to tell you
o Avoid jargon
o Check patients understanding of the information where
possible
Address the patients emotions
o Offer support by identifying the patients emotions and the
reasons for them
Strategy and summary
o Outline a plan of next steps for the patient, including another
planned encounter
o Having a plan provides some security
o Written information
o Planned second encounter to further discuss their concerns

STARTING AN ACE INHIBITOR

This is Mrs Stevens, she is a 60 year old woman who has recently
been diagnosed as Hypertensive. Despite increasing her exercise
habits, she has been unable to normalise her blood pressure.
Her latest BP reading was 160/90.
She is generally well, but has suffered from Type 2 diabetes Mellitus
for the last 2 years. She is diet controlled and manages this well.
You wish to start Mrs Stevens on an ACE inhibitor. Please consider
whether this is an appropriate drug to start and then provide drug
counselling to Mrs Stevens.

STARTING AN ACE INHIBITOR


1. Check that there are no contraindications/cautions
Renal failure (serum creatinine >200; though not an absolute CI)

Hyperkalaemia (>5.5)

Hyponatraemia (caution if <130)

Hypovolaemia or hypotension (systolic BP <90)

Aortic stenosis or LV outflow obstruction

Pregnancy or lactation

Severe COPD or cor pulmonale (not an absolute CI)

Renal artery stenosis (suspect if arteriopathic eg CVAs, IHD, PVD)

2. Warn about possible side effects

Dry cough (1:10)

Hypotension, especially with 1st dose if CCF


o Light headed, dizzy, headache, fatigue, nausea, stomach upset

Taste disturbance

Hyperkalaemia

Renal impairment

Urticaria and angioedema (<1:1,000)


o Have you ever been diagnosed with angioedema?

Rare: proteinuria, leucopaenia, fatigue

3. How to start taking ACE-i

Delay starting if intercurrent illness (especially if dehydrated)

Stop potassium supplements and K-sparing diuretics, stop other diuretics for
24h
The first time you take this drug it should be at night, and you may notice that
you have to get up at night to go to the toilet
o Make sure there is someone there to help you get out of bed
You may feel dizzy when you first start taking this medicine. You should get up
gradually from sitting or lying to minimise this effect. Sit or lie down if you
become dizzy or light-headed.
Will need some blood tests to look at your kidney function and blood
electrolytes
o Before you start
o Again after 1-2 weeks at which time well review how youre going
Dont start if pregnant / using inadequate contraception
o Breastfeeding OK
Careful with taking other medications like anti-inflammatory drugs and fluid
tablets
4. General knowledge

Mechanism of Action
o Inhibit a normal mechanism in the body that causes blood pressure to
rise.
o reduce the effects of AII vasoconstriction, sodium retention and
aldosterone release.
o reduce the effect of angiotensin on sympathetic nervous activity and as a
growth factor.
Expect a rise in creatinine (<20%, GFR decreased by <15%) if more, there
may be underlying renal artery stenosis
Angioedema
O Onset may not be for several years

ANGRY PATIENT

You are medical student undertaking an attachment in a GP surgery


in a rural practice near Adelaide, which has 2 doctors, Dr Farley and
Dr Wilson. Dr Wilson is your mentor.

Mr Jerry Suzuki, a 40 year-old businessman saw Dr Farley earlier this


week for a blood test.

He has made an appointment with Dr Wilson, to receive his bloodtest results. Dr Wilson asks you to go through the results with him.
The results are normal.

Mr Suzuki shows you a badly bruised arm, which is the result of the
venepuncture with Dr Farley. He says: Ive never had a bruise from
a blood test. He wants to discuss this issue with you further.

Task
Please address this patients concerns and manage the problem as
appropriate.

When you go in the station, the patient shows you the bruise in his cubital fossa. He then tells
you that Dr Ys venepuncture seemed all over the place and his breath smelt of alcohol. It
wasnt entirely clear what they wanted from this station, but Im sure it was vital to maintain
professionalism at all times. Try not to side strongly with either the patient or the doctor

Guide to the station:


- Introduction;
Introduce yourself and explain role
Develop rapport, attends and listens to what the patient is saying in a respectful
manner
Allow patient to fully disclose concerns
Should appear unhurried, respond to patient cues and facilitate the disclosure
- History;

Elicit history of the venipuncture experience and patients feeling about this
interaction ask about;
o Patients feelings at the time
o Events that upset the patient
o The circumstances that led to the bruise
Actively seek information about circumstances of the bruise
Enable patient to disclose suspicion of Dr Farleys alcohol intake I smelt
alcohol on the doctors breath
Facilitate patient concerns
- Counselling;
Offer reassurance about the bruise some bruising is common after taking
blood, and this usually resolves after a few days
Seek patients opinion about how to proceed with this complaint
Recognize own limitations
Explain you will inform Dr Wilson about the issue and this conversation
Display professionalism and maturity, leaving the patient feeling reassured

Specific areas to cover/address:


- Bruise on arm;
Quickly inspect the bruise/arm, assess ROM and state that it will improve over
the coming days
o Can suggest regular simple pain relief if painful
State that bruising is a common side-effect of venipuncture
- Alcohol on Dr Williams breath;
This is a very serious accusation, but is one that obviously needs to be followed
up
Further Hx of patients interaction with the doctor;
o Length of time he has been seeing the GP
o How the doctor has been in the past
o Is this behavior out of character?
You will include patient in further discussion, but maintain confidentiality
Explain you will address the issue with Dr Williams and will record and document
the interview
Explain if the patient wishes, the accusation can be followed up with higher
Medical Board

Instructions for SP:


- To present angry about the scenario and predicament
- If unappeased, threaten to walk out and tell press about Dr Williams drinking
problems FAIL
- If medical student tries to shaft off responsibilities FAIL

BREAKING BAD NEWS COLORECTAL CANCER

Follow-up appointment with Mrs. Baxter, who had presented to you


with rectal bleeding and weight loss.
Colonoscopy has shown a tumour in the sigmoid colon, which on
biopsy has been shown to be a adenocarcinoma. Staging CT scans
show 2 small (<1cm) liver lesions.
Your interview should communicate this information to Mrs. Baxter
and advise her on prognosis and further management.

This is a Dukes D carincoma. 5 year survival is around 5%. There are no surgical
curative options but there is potentially an avenue for palliative surgery.
Palliative chemotherapy and radiotherapy for symptoms control can also be
employed.
Specifics about the interview
SPIKES
Lifetime risk of ?1:20
Risk factors
o Increased risk
Increasing age
Diet (high animal fat, low fibre)
FHx
Genetics (FAP, HNPCC)
Alcohol
o Protective
COCP
HRT (combined)
Refer to surgeon
Refer to oncologist

Staging and Treatment


Dukes stages:
o A: Invades mucosa (90% 5 year survival)
o B: Into/through muscularis propria (not serosa) (75% 5 year survival)
o C: lymph node involvement (50% 5 year survival)
o D: Distant metastasis (<10% 5 year survival)
Treatment options:
o Surgical excision
o Chemotherapy (Stage C and D)
o Radiotherapy (only in palliative really)
Follow-up:
o Colonoscopy at diagnosis/within 1 year of then:
At 3 years post-diagnosis
At 5 years post-diagnosis
Then 5 yearly thereafter (if normal)
o Yearly CEA, FBE, Faecal Hb
o Genetic testing can be done (MSH, MLH-1, APC)

Family member screening:

Category 1 at or slightly above average risk (3-6x risk)


o 1 first degree relative >55 or two relatives on different sides of the family >55
o Screening
FOBT every 2 years from age 50
Consider sigmoidoscopy every 5 years
Category 2 moderate risk
o 1 fist degree relative <55; two first degree or 1 first and 1 second degree on the
same side of the family
o Screening
Colonoscopy at 50 OR 10 years before earliest cancer every 5 years
FOBT in intervening years

Category 3 - high risk


o Three or more 1st degree relatives, bowel cancer before age 50, FAP, Lynch
syndrome (uterine/ovarian + colonic), Turcots (CNS hamartoma + colonic
polyps).
FAP: sigmoidoscopy yearly from age 12-15 until polyps then prophylactic
surgery
HNPCC: colonoscopy every 1-2 years from age 25 or 5 years before
diagnosis in family member
FOBT in intervening years

Setting up the interview


o Arrange for a private encounter
o Involve significant others
o Sit down
o Maintain eye contact
Assess the patients perceptions
o Assess how the patient perceives their current situation
o What have you been told about your condition so far?
Obtain the patients invitation
o Find out at what level the patient wishes to have the information disclosed
details vs big picture
Pass along knowledge
o Warning shot eg I have some bad news to tell you
o Avoid jargon
o Check patients understanding of the information where possible
Address the patients emotions
o Offer support by identifying the patients emotions and the reasons for
them
Strategy and summary
o Outline a plan of next steps for the patient, including another planned
encounter
o Having a plan provides some security
o Written information
o Planned second encounter to further discuss their concerns

Im
por
tan
t
poi
nts
Em
pat
hy /
rap
port
Paci
ng
inte
rvie
w

Comments

Seating patient comfortably / use of silence, pause, avoiding


premature reassurance / appropriate use of body language

Checks whether patient wants someone else

Preliminary-established goal of appointment

Givi
ng
info
rma
tion
For
war
d
pla
nni
ng

Warning shot signalling of bad news


Check OK to go on
Giving patient opportunity to ask questions
Explains test findings and implications including appropriate
interpretation of CT findings

Communicating news to others

Referring to surgeon
Referring to oncologist
Dukes D woman with 2 liver mets
Break bad news key was to give her time
Offer tissues
Offer to help tell family

CHILDHOOD OBESITY

STEM
You are in general practice
Mrs Thompson has come to see you today because she is worried
about her grandson Eric. Eric is 5 years old. He is a healthy child.
Plot Erics growth from the measurements provided on the growth
chart
Age 5: weight 27kg, height 112cm; age 4: 22kg, 103cm; age 3:
17kg, 92cm; age 2: 15kg, 85cm
Describe Erics growth pattern to his (grand)mother and answer her
questions.

ANSWER
PLOTTING

QUESTIONS
Charts show Eric is an obese child
Why is Eric obese?
o This is most likely due to decreased activity and increased food/calorie
intake
o Medications glucocorticoids, valproate, antipsychotics, insulin
What risks does Eric have being obese?
o Psychosocial implications
o Orthopaedic

Slipped capital femoral epiphyses


Blount disease tibia vara (bowing of the tibia)
Genu valgum
Flat wide feet
Poor soft tissue injury recovery
o Hepatobiliary
NAFLD (raised AST)
Cholesterol cholelithiasis
GERD
o Neurological
Idiopathic raised ICP
o Respiratory
Asthma
Poor exercise tolerance
Obstructive sleep apnea
o Reproductive
Polycystic ovary syndrome
Menstrual abnormalities
o CVS
Hypertension
Lipid profile changes
o Endocrine
Hyperinsulinaemia DMT2
Acanthosis nigracans
Family history?
o Everyone in the family is obese
Possible other causes?
o Medical cause is unlikely due to the pattern of the growth chart
o Endocrine hypothyroid, hypercortisolaemia, GH deficiency
o CNS hypothalamus or pituitary damage e.g. trauma, radiation
o Syndromes
Investigations?
o Exam early puberty, thyroid, dysmorphic features
o BGL
o Cholesterol
o Endocrine hypothyroid, hypercortisolaemia, GH deficiency
Management?
o Increase activity
o Decrease calorie intake

MARKING SCHEME

GENETIC COUNSELLING

Description of task:
Mr Alex Skarsgrdhas come to see you (his GP) with the results of
some blood tests that he is concerned about. He is otherwise feeling
well and has nil medical issues to report.
The results are as follows;

CBE:
Hb: 150 g/L

(135-175)

MCV: 93 fL

(80-98)

MCH: 30 pg

(27-33)

Serum Iron studies:


Total serum iron: 36 mmol/L

(11-27)

Transferrin: 27 mmol/L

(20-40)

Transferrin saturation: 70%

(15-55)

Serum ferritin: 990 ug/L

(30-500)

Liver function tests:


ALT: 30 U/L

(0-45)

AST: 26 U/L

(0-45)

GGT: 40 U/L

(0-59)

ALP: 75 U/L

(30-110)

Albumin: 40 g/L

(34-48)

Bilirubin: 15umol/L (2-24)

Tasks for the student:


Interpret the above blood results and explain the diagnosis to the
patient
Answer the following questions:
o How did he develop this condition?
o Will the patient need any further tests?
o How is the disease treated and monitored?
o What is the prognosis?
o Will his son develop the condition?

BLOOD TEST INTERPRETATION:

- CBE: normal

- LFTs: normal

- Iron studies;

Elevated serum iron, transferrin saturation and serum ferritin

Low transferrin

These results are consistent with iron overload, most likely cause being
(hereditary) haemochromatosis

EXPLANATION OF DIAGNOSIS:

- The blood tests you have given show that you have excessive iron in your
body, and the most likely cause of this is hereditary haemochromatosis

QUESTIONS:

How did he develop this condition?

- HH is an inherited disorder of iron metabolism, in which excessive iron is


deposited in multiple organs, in particular the liver

Autosomal-recessive

Variable penetrance

- I.e. this means the condition has been passed down in your family

- More common in people of Scandinavian descent (this patient likely to be of


Scandinavian descent)

Will the patient need further tests?

- Genetic testing for C282Y and H63D mutations


- Liver biopsy; (probably not required in this patient)
Special iron stain (Perls) quantifies iron loading
Indications;
o Raised LFTs with clinical evidence of HH
o Serum ferritin >1000 mcg/L
o Patients with high probability of cirrhosis
- Imaging; (probably not required in this patient)
Liver MRI used to estimate hepatic iron loading
Joint x-rays may show chondrocalcinosis
CXR cardiomegaly, evidence of CCF (do echo and ECG if suspected)

How is the disease treated and monitored?

- Lifestyle modification;
Reduce dietary iron
Reduce vitamin C supplementation
Limit alcohol consumption
Hep. A and B vaccinations
- Patients with known genetic mutations but no clinical disease require
monitoring only (regular iron studies, LFTs etc.)
- Patients with clinical disease;

Phlebotomy/venesection;
o Simple, cheap
o Approx. 1 unit/week taken until mildly iron deficient
o Maintenance venesection needed for life
Iron chelation when phlebotomy contraindicated (bleeding risk)
- Other;
Diabetic monitoring and management
Management of other endocrine problems (hypogonadism)
Testing of AFP for HCC in those with cirrhosis
Monitoring/managing cardiomyopathy

What is the prognosis?

- Mean survival is 21 years


- Patients have increased risk of dying from;
Cancer (11%) esp. HCC (7%)
Cirrhosis (6%)
DM (2%)
Cardiomyopathy (2%)
MI (2%)

Will his son develop the condition?

- Not necessarily, also depends on whether mother has the gene

- If mother does not have the gene, patient will not develop the condition but
may still inherit the faulty gene, and so becomes a carrier

- If mother also has the gene, patients son has a 25% chance of developing the
condition

Haemochromatosis

- AKA hereditary haemochromatosis (HH)


- An inherited disorder of iron metabolism in which increased iron is deposited in
multiple organs (liver, joints, heart, pancreas, pituitary, adrenals, skin)
- Excessive iron due to;
Increased dietary iron absorption
Increased iron release from erythrophagocytosis

Epidemiology:
- Common genetic disorder but with variable penetrance
- More common in Scandinavian people
- Equal gender prevalence but clinical disease is more common in men;
Typically occurs in middle-age in men
Men more severely affected also
- Women present approx. 10 years later, as menstrual blood loss is protective

Aetiology & Genetics:


- HH is an autosomal-recessive disorder
- The gene responsible for most cases is HFE, found on short arm of
chromosome 6
- 2 major mutations of HFE have been implicated; (totally dont need to know
this!)
C282Y 60-90% of cases
H63D 6-9% of cases
- Penetrance is unknown but is <100%

Pathophysiology:
- Mutation of HFE gene disrupts its normal function;

C282Y mutation reduces hepatic hepcidin synthesis increased duodenal iron


absorption and increased iron release from macrophages associated with
erythrophagocytosis
- This excessive iron is then deposited in various organs (a process called
siderosis) dysfunction and failure
- Liver;
Iron accumulates in peripheral hepatocytes and can progress to cirrhosis
Damage is due to increased oxidative injury/stress
- Heart;
Siderosis thickening of myocardium
Heart may enlarge cardiomegaly, heart failure
- Pancreas siderosis DM
- Anterior pituitary siderosis hypogonadism
- Skin siderosis hyperpigmentation
- Different systems/organs are involved to varying amounts, producing varied
range of symptoms

Clinical features:
- Asymptomatic in early stages, but progresses to fatigue, arthralgia and
impotence
- Typical abnormalities on presentation;
LFT abnormalities 75%
Arthralgia (MCPs and large joints)
44%
Weakness, lethargy 74%
Impotence (males), secondary to
Skin hyperpigmentation 70%
hypogonadism 45%
Diabetes mellitus 48%
ECG abnormalities 31%
- Liver disease;
Hepatomegaly, signs of chronic liver disease
Abnormal LFTs
Eventual development of cirrhosis, and may also develop HCC
- Skin hyperpigmentation;
Often begins as bronzing of skin but typically progresses to slate-grey
discolouration
Changes are usually generalised

Investigations:
- Blood tests;
LFTs transamines typically increased (usually not greater than 2x normal)
Iron studies;
o Elevated serum ferritin
o Elevated serum iron
o Transferrin saturation (first test to become abnormal) screening
threshold for diagnosis = >45%
o Total iron-binding capacity (TIBC) reduced
Blood glucose, HbA1c, OGTT assess development of DM
- Genetic testing for C282Y and H63D mutations
- Liver biopsy;
Special iron stain (Perls) quantifies iron loading
Indications;
o Raised LFTs with clinical evidence of HH

o Serum ferritin >1000 mcg/L


o Patients with high probability of cirrhosis
- Imaging;
Liver MRI used to estimate hepatic iron loading
Joint x-rays may show chondrocalcinosis
CXR cardiomegaly, evidence of CCF (do echo and ECG if suspected)

CARDIOVASCULAR RISK

OVERVIEW
Introduction
o Introduce self
o Confirm reason for visit
Explain result
o Cholesterol is high
o High cholesterol has been associated with a higher risk of heart disease,
stroke and peripheral vascular disease
o Is this is first high reading?
Further risk assessment
o The risk is even higher with other CVS risk factors hypertension, obesity,
smoking (which he has)
o Diabetes (he does not have)
o Enquire for any other relevant history (past MI, arrhythmias, PVD, ATSI
etc)
o Lack of physical activity
o Poor diet
o Family history age of disease, number of relatives
o Explain age, gender and family history also raises his risk
o Overall, he is at high risk (American Heart Association calculation high
risk)
Management plan
o Quantify other risks
Measure waist circumference
Remeasure BP to confirm hypertension
Do fasting BGL
o Conservative
Exercise 30min pd
Diet low calorie, low cholesterol (lower animal foods intake)
Stop smoking counselling, quit line, patches
o Remeasure at 6 weeks
o If still high statin
Prior to commencement LFTs (and every 6mo once starting
treatment)
s/e myalgia, myositis, LFT rise
Eligibility see table (if he is diabetic, HDL <1mmol/L, family
history, triglyceride >8)
Follow up
o Dietician referral if wanted
o See in 6 weeks for repeat blood test and lfts
INFORMATION ON STATIN THERPAY
For PBS funding, any of the below patients can start a statin.

Post 6 weeks of dietary management these people can start a statin.

MARKING SCHEME

PRE-OPERATIVE - THYROID

Sally Smith is a 39-year-old woman who has recently been


diagnosed with papillary carcinoma of the thyroid after noticing a
rapidly enlarging mass in her neck.

She has been advised she is going to need to have a total


thyroidectomy but has had no further information.

She has come to you (her GP) for more information about the
procedure and long-term management.

Desribe/explain to the patient;


The nature of the procedure
Potential complications
Additional treatment she may need and long-term follow-up

Nature of the procedure:


-Total thyroidectomy means that the entire thyroid gland (both lobes
and isthmus) will be surgically removed, in the hope of removing all
cancerous cells.
- It is indicated as the patient either has a high risk of recurrence,
has LN mets, or has bilateral lobe disease.
- Will require general anaesthetic and will have to spend the night in
HDU to ensure there are no complications.
- As the whole thyroid gland is being removed, they will require
lifelong thyroid hormone replacement with thyroxine.

Complications:
Those in Bold are the ones I think are the most important to discuss
- General (as for all surgery);
Pain
Infection

Bleeding acute and chronic haematoma


Scar visible scar on neck, may be concerning for some
patients
- During surgery;
Haemorrhage
Unilateral or bilateral recurrent laryngeal nerve damage
o Bilateral nerve damage presents as laryngeal obstruction after
tracheal extubation and necessitates immediate tracheostomy
o Unilateral nerve damage presents as hoarseness, weak voice,
impaired coughing
Damage to other structures;
o Trachea, oesophagus
o Parathyroid glands need to assess Ca2+ and PTH levels
- Early (within 12 hours post-op);
Major haemorrhage presents as rapid neck swelling or large
blood loss via wound drain
Mediastinal haemorrhage
Laryngeal oedema
Thyrotoxic crisis
Tracheomalacia
- Late;
Hypoparathyroidism due to inadvertent damage/removal of
parathyroid glands
o Presents with muscle cramps, paraesthesiae, tetany within 36
hours
Voice changes;
o Unilateral recurrent laryngeal hoarseness, impaired cough
o External laryngeal nerve changes quality of voice
- Long-term;
Hypothyroidism therefore must take lifelong thyroxine
Recurrence insufficient gland removed

Additional treatment & Long-term management


- Following surgery, patient will require;
Radioactive iodine to destroy any remnant thyroid tissue
Lifelong thyroxine serves to replace thyroid hormone and suppress
TSH to prevent growth of remnant thyroid tissue
Thyroglobulin measurement tumour marker for recurrence (in
papillary ca)
- Prognosis is very good;
10 year survival >90%
40 year survival 94%

SKIN LESION

Mr Jones is a 45 year old builder who has come to see you as he is


concerned about a mole on his back.
Counsel the patient about the likely diagnosis and the likely cause.
Discuss with him the required investigations and subsequent
management, answering any questions the patient may have.

Additional information:
Pathology report:
The excised area was 44mm in diameter. The lesion was 30mmx30mm and was
confirmed to be a malignant melanoma.

The lesion had a Breslow thickness of 0.68mm and was found to be a Clark level
II. The margins were clear.

Explanation:
- Explains that melanoma is most likely
- Assess the patients understanding of what that means
Malignant skin cancer, change in skin cells likely due to UV exposure
- Risk factors;
80% occur in white-skinned individuals
Short periods of blistering sun exposure increases risk more than a consistent
high level of exposure

Family history of melanoma


Freckles, moles esp. on upper back
Red or blonde hair
Blue, green or grey eyes
Presence of solar keratoses
Why do you think its a melanoma?
A: Asymmetry
B: Border (irregular)
C: Colour (non-uniform)
D: Diameter >7mm
E: Elevation
Other- evolution, rapid growth, itchy or painful
What else could it be?
- Benign
Naevi
Seborrhoeic Keratosis
Solar lentigo
- Malignant
Pigmented BCC

Management:
- Excisional biopsy for histology is the initial procedure of choice wide excision
should not be performed until histological diagnosis has been made
- Definitive surgical treatment for primary cutaneous melanoma is wide local
excision down to the deep fascia +/- sentinel node biopsy

What do you look for with histology?


Clark level- anatomical level of invasion
Breslow depth of invasion in mm

What if it has invaded? What is the next step?


Referral to oncology
Staging investigations CT scan, bone scan, chest XR,
Adjuvant therapy;
o Radiotherapy
o Immunotherapy
Breslows depth:
- Determines excision margins required for lesion
Lesion depth
Excision margin
<1mm
1cm
1.1-4mm
1-2cm
>4mm
At least 2cm
As per RACGP guidelines may be other guidelines elsewhere
Any patient with depth of invasion >1mm should have sentinel node biopsy to
assess presence of LN mets.

Clarks Level:
The Clarks level is commonly misunderstood as the melanoma stage diagnosis.
The Clarks level only refers to how deep the tumor has penetrated into the skin.

Research has shown that the Clarks level diagnosis is not a great predictor of
outcome.
Clark's Level I - Confined to epidermis also called in situ melanoma
Clark's Level II - Invasion of the papillary dermis (upper)
Clark's Level III - Filling of the papillary dermis (lower)
Clark's Level IV - Extending into the reticular dermis
Clark's Level V - Invasion of the subcutaneous tissue

Prognosis:

Follow-up:
- Follow up is dependent on the size of the lesion:
Thin lesion (<1mm thickness)
o Follow up annually
Thick lesion (>4mm)
o Follow up at least every 4-6 months for the first 2-3 years
o Less frequently thereafter
Prevention
- Avoidance of sun exposure
- Use of sunscreen and protective gear
- Self checks of moles and return if there are any suspicious lesions

DEMENTIA COUNSELLING

Counselling daughter of patient with Alzheimers


History
o Mothers symptoms
Affecting 2 spheres of cognition, one of which must be memory
Aphasia (language disturbances)
Agnosia (object and face recognition)
Apraxia (motor task problems)
Abstraction
Acalculia
Personality
o Duration of illness
When diagnosed
Who diagnosed
How treated
o Current level of functioning
o Current supports in place
Counselling
o What do you know about dementia/Alzheimers?
Non-reversible disease process
Constant cognitive decline
Treatment delays decline by about 6 months
o How are you coping at the moment?
o Carer stress is a big problem, make sure to use community help (mentioned
later) as a way to get away as this can be a stressful and difficult time for both
parties.
o Advanced directives need to be put in place ASAP if not already done
Enduring power of attorney
Financial/legal
Enduring power of guardianship
Medical/housing/everything else
o ACAT etc. to be involved when mother can no longer live at home
o Daughter does not need to take everything on herself, other services available
Other family members
Dom Care
Community aged care package
Meals on wheels
Extended Aged Care Healthplan Dementia (EACH-D)
Community support groups (Alzheimers Australia)
Respite care
Financial help (Centrelink)
Good geriatrician for mothers medical needs

OESOPHAGEAL CANCER

A male patient comes back to you after a diagnosis of adenocarcinoma of the


oesophagus with a few questions. Answer them

Was this related to my 20 years of heartburn?


o Yes, with transition to Barretts oesophagus
Should I have been treated better?
o Depends how you were treated
Could this have been prevented?
o You could have reduced your risk
o If it was purely GORD contributing, stopping GORD would have decreased the
chances of it. If you continued to smoke/drink alcohol then this would also have
contributed to this disease process.
Prognosis
o Overall 15% 5 year survival
How will they treat it?
o Stage I-III
Surgery is mainstay, however stage III results are poor (add chemo)
o Stave IV
Systemic chemotherapy
Palliation (stenting, radiotherapy, brachytherapy)
Will my kids get it?
o No strong genetic link
IS MY FAMILY AT RISK?
Well, the way that cancer can develop is:
Just by chance
Families share environmental and lifestyle factors
Some have an inherited genetic predisposition, however this is relatively uncommon

BREAST CANCER

SPIKES counselling

Mammography

Epidemiology: Breast cancer is one of the most common causes of cancer deaths in
women and increases in frequency with age. One in 11 South Australian women will be
diagnosed with breast cancer before the age of 75 years.

Breast Screening: Screening is primarily recommended for all women aged 50 to 69


without breast symptoms. It is estimated that for individual women in this age group,
having a screening mammogram every two years reduces the chance of dying from
breast cancer by about 40%.

Research is less clear about the benefits of screening mammograms for women aged
40 to 49 and over 70. However, women in these age groups are also eligible for
screening and are very welcome to phone for an appointment if they wish to attend.
Women from the age of 40 with a strong family history of breast cancer are eligible for
a screening mammogram every year.
Ultrasound: u/s is recommended for women with breast symptoms <35years of age,
as mammograms cannot see much due to the dense breast tissue.

Tamoxifen
Stops oestrogen in breast cells
Increases oestrogen in uterine -> increased risk of uterine cancer
Can cause heavy bleeding, clots in pre-menopausal women
Increased risk of coagulopathy espec. DVT
Side effects: hot flushes, stomach cramps
Protective for bones
Arimidex
Blocks production of oestrogen (aromatase inhibitor)
Increases risk of osteoporosis = have bone density scan
Give calcium and vitamin D
May experience some joint pain which usually resolves after 1 year
Menopausal side effects

Breast Cancer Screening


Category 1
o 1 first degree >50; one second degree at any age
o Screening
Mammograms every 2 years from age 40
Discuss self-check and modifiable risk factors
Category 2
o 1 first degree <50, two 1st degree on same side
o Screening
Mammograms yearly from 40
Category 3
o Everything higher risk
o Screening
Consider risk reduction surgery or medications (tamoxifen)
Annual breast imaging
Consider ovarian cancer

CIN/HPV & PAP SMEARS


Abnormal Pap Smear
Low grade lesions (CINI, II): repeat smear at 12 months, colposcopy if still present
High grade (CINIII): immediate colposcopy
Invasive: immediate gynaecology referral
Colposcopy
Microscope to view the cervix
Acetic acid, then iodine
Neoplastic cells: acetowhite biopsy for histological confirmation

Colposcopy during pregnancy requires skill and experience due to progressive edema,
increased eversion, increased vascularity and the risk of significant bleeding with
biopsy.
HPV infection
Common, most infections are transient and resolve over 12-18 months.
Increased cancer risk: 16, 18 mainly
Vaccine available
Associated with different cancers
However, persistent infection with an oncogenic serotype
Prevention
o Condom usage
o Vaccination
Treatment
Cryotherapy
o Healing over 6 weeks with watery discharge
o No specimen for histology, lesion may be missed
Large Loop Excision of Transformation Zone (LLETZ, or LEEP)
o Risk of severe post-op bleeding
o Provides histo specimen
Cold knife conisation
o For significant lesions in endocervical canal
Avoid tampons, strenuous exercise, douching and intercourse until discharge has
settled
o Usually 2-4 weeks
Treatment failure in 5-10%
Treatment cannot cure HPV
o Test for HPV at 12 months

HPV
70-75% of cervical cancers are associated with HPV genotypes 16 (60%) and 18 (15%).
HPV integrates its double stranded DNA into the hosts (humans) DNA. Proteins from
HPV change the hosts DNA. E5 -> activation of growth receptors. E7 -> sequesters pRb
(promotes cell division) E6 -> activation of p53 causing the cervical cells continue to
cycle despite DNA damage. These changes combined with other mutations and
damage accumulated over many years results in cervical cancer.
Following HPV infection most women are able to clear the infection. However those
who are unlucky will go from CIN I III -> carcinoma over 10-20 years.

Pap smears
In Australia organised cervical screening was initiated in 1991 for sexually active
women aged between 18-69years with a recommended 2 year interval.
(Now two years after sexually active or age 20, whichever occurs last?)
Cervical cancer incidence and mortality has dropped dramatically since the introduction
of Pap smears.
CIN is being replaced by LSIL and HSIL
LSIL = low grade squamous intraepithelial lesion
Low grade dysplasia
CIN I
Koilocytosis on Pap smear
Typically resolves
HSIL = high grade squamous intraepithelial lesion
High grade dysplasia

CIN I and II
More likely to progress to cancer

What to do about Pap results


Scenario:
Results come back unsatisfactory
a Explain to patient results were unsatisfactory
b Repeat pap smear in 6-12 weeks
2 Result comes back normal
a Reassure patient that results was normal
b Recommend they come back in 2 years for Pap smear
3 Result comes back as LSIL in patient <30years
a Reassure they dont have cancer
b Explain they have signs of a virus on their cervix
c Ask them to come back in 12 months for Pap smear to see if the virus has gone
4 Result comes back as LSIL in patient 30+years
a Reassure they dont have cancer
b Explain they have signs on a virus on their cervix
c Offer a colposcopy immediately or a pap smear within 6 months
5 Early pap smear at 12months following LSIL is again showing LSIL
a Reassure they dont have cancer
b Explain that the virus is still present on their cervix
c Refer for colposcopic assessment
6 Early pap smear at 12 months following LSIL is normal
a Reassure they dont have cancer
b Explain that the virus on their cervix has gone
c They should come back at 12 months for pap smear
7 Have had 2 pap smears 12 months apart both normal following a LSIL
a Reassure they dont have cancer
b Virus has been cleared
c Can return to 2 yearly pap smears
8 Patient has had two LSIL report on pap smear in a 3 year timeframe
a Reassure they dont have cancer
b Refer for colposcopic assessment
9 Any pap smear results is HSIL
a Reassure they dont have cancer
b Explain they have a pre-cancerous lesion on their cervix
c Refer for colposcopic assessment
10 Pap smear results report SCC
a Refer to gynae oncologist for urgent evaluation within 2 weeks

At colposcopic assessment after LSIL smear


1 High grade lesion is seen or suspected
a Target biopsy
2 Colposcopic assessment if normal
a Patient referred back for annual pap smears
b When two normal smears are obtained can go back to 2 yearly smears
3 A Low grade lesion is seen or suspected
a Target biopsy
4 LSIL is histologically confirmed
a no treatment necessary
b repeat pap smears at 12 and 24months
c if two smears are normal return to 2 yearly smears
d continue annual smears if LSIL remains

At colposcopic assessment after HSIL smear

HSIL needs histological confirmation from targeted biopsy before definitive treatment is
undertaken.
Ablative therapy (laser ablation or cryotherapy)
May be provided as long as:
the cervix has been assessed by an experienced colposcopist
a targeted biopsy has confirmed the diagnosis
there is no evidence of an invasive cancer on cytology, colposcopic assessment or
biopsy
the entire cervical transformation zone has been visualised
there is no evidence of a glandular lesion on cytology or biopsy
Cone biopsy (cold knife conisation CKC and loop electoral excision procedure LEEP)
May be necessary for women with HSIL where:
there has been a failure to visualise the upper limit of the cervical transformation zone
there is a suspicion of an early invasive cancer on cytology, biopsy or colposcopic
assessment
the suspected presence of an additional significant glandular abnormality on cytology
or biopsy
After treatment for HSIL, a patient should have a repeat colposcopy and cervical
cytology within 4-6months. Cervical cytology and HPV typing should then be carried out
at 12months after treatment and annually thereafter until the woman has tested
negative by both tests on two consecutive occasions. Then the woman can return to
two yearly pap smears.

CERVICAL CANCER

Recall: Perform a pap smear on the patient (dummy) then discuss the results which
will be provided.

APPROACH
SPIKES
Refer
o Gynaecological oncologist
o Radiation oncologists
o Pathologist

Information
Risk factors
o HPV
o Smoking
o Immune suppression
Prognosis
o Depends on how far it has spread
o The sooner cervical cancer is diagnosed, the better the prognosis. Most women
with early cervical cancer will be cured.
o Many women with more advanced cancer are still cured. For others, treatment
can keep the disease under control for long periods of time.
o 5 year survival
CIN: 95%
1: 75%
(cervix)
2: 55%
(beyond uterus)
3: 35%
(pelvic wall, lower 1/3 of vagina)

4: 7% (metastases)
Treatment
o Surgery (radical hysterectomy)
o Radiotherapy
o Palliative
Chemotherapy

DATA STATIONS

ARTERIAL BLOOD GAS

Ted Mosby is a 75 year old man with pneumonia. These are the
results of his ABG:
pO2
88 mmHg
(90-110)
pCO2
28 mmHg
(35-45)
pH
7.6
(7.35-7.45)
HCO3 25 mmol/L
(22-31)

1. Please interpret this ABG.


2. Please give 5 differentials for respiratory alkalosis (does
not have to relate to current presentation)
3. By what mechanism is his pneumonia causing this
presentation?

You then take a random BGL and find that it is 14 mmol/L.

4. Why is his BGL elevated?

ANSWERS
1. This is an acute respiratory alkalosis. In a chronic picture, there would be a
corresponding drop in bicarbonate.

2.

Central nervous system causes are as follows:


o
o
o
o
o
o

Pain
Hyperventilation syndrome
Anxiety
Panic disorders
Psychosis
Fever

o
o
o
o
o

Cerebrovascular accident
Meningitis
Encephalitis
Tumor
Trauma

Hypoxia-related causes are as follows:


o High altitude
o Right-to-left shunts
Drug-related causes are as follows:
o Progesterone
o Methylxanthine toxicity
o Salicylate toxicity
o Catecholamines
o Nicotine
Endocrine-related causes are as follows:
o Pregnancy
o Hyperthyroidism
Pulmonary causes are as follows:
o
o
o
o
o

o
o
o
o

Interstitial lung disease


Asthma
Emphysema
Chronic bronchitis

Miscellaneous causes are as follows:


o
o
o
o

Pneumothorax/hemothorax
Pneumonia
Pulmonary edema
Pulmonary embolism
Aspiration

Sepsis
Severe anemia
Hepatic failure
Mechanical ventilation

o Heat exhaustion
o Recovery phase of
metabolic acidosis
o Congestive heart failure

o 3. In pneumonia, the hypoxic drive is more influential than the hypocarbic


drive in terms of respiration. Therefore, the patient is tachypnoeic to
increase oxygen intake, but as a result exhales excessive CO2
o 4. The disease stress of pneumonia has lead to increased levels of
circulating cortisol hyperglycaemia.
o MARKING SCHEME
Correctly identifies ACUTE respiratory alkalosis
o Half marks if states only respiratory alkalosis
MUST identify 5 correct differentials
Correctly identify that the pneumonia is the cause of alkalosis
o Should give correct mechanism
o Can still pass station without
MUST identify cause of hyperglycaemia

o DATA STATION
o

o Mr George is an 80 year old man who presents to you in the


RAH ED following referral by his GP.
o He has with him a note from the GP stating that he has had
recent onset nausea and vomiting, along with some deranged
biochemistry results:
o
o Sodium
o 120
o (135-145)

o Potassiu
m
o Phosphat
e
o Calcium
o Creatinin
e
o Urea
o
o The patient is also
Digoxin
Gentamicin
ACEI
Diclofenac
Frusemide

o 5.6

o (3.5-5)

o elevated

o
o Elevated

o
o (

o Elevated

on the following medications:

o Please complete the following:


Interpret the biochemistry results
Give differentials
Provide immediate management options
Describe the mechanism by which the patients medications have
contributed to their current situation
o
o
o
o
o
o
o
o
o
o
o
o
o
o
o
o
o
o
o ANSWERS

o Biochemistry
Acute renal failure

o Differentials
Pre renal
Renal

Post renal
Most likely in this case is the combination of the drugs being taken

o Management
Volume management
o Hypervolaemia
o Treat with diuretic therapy if severe
Electrolyte and acid base management
o Acidosis is not treated until <7.20 pH. Can be treated with oral or IV
sodium bicarbonate
o Hypocalcaemia does not usually require treatment
o Hyperphosphataemia can be treated with phosphate binders
Discontinue renally toxic medication
Dialysis if:
o Medical management has failed to control volume overload,
hyperkalaemia, acidosis
o Gentamicin: concentrating in distal convoluted tubule
and directly nephrotoxic
o ACE inhibitor: angiotensin II usually constricts efferent
arteriole / inhibition causes decreased GFR
o NSAID: decreases prostaglandin-mediated constriction
of efferent arteriole

o
o
o
o
o
o ACU
TE

RENAL FAILURE

o DEFINITION
RAPID deterioration in renal function sufficient to result in accumulation of
nitrogenous wastes
o Around 1% of patients have ARF on admission
o 2-5% of all hospitalised patients develop ARF (20% in ICU)
o 20-60% of cases require dialysis
o Increases the likelihood of fatality by 8 times
o Acute Tubular Necrosis occurs in around 75% of cases of ARF
There is diffuse tubular cell damage
o Ischaemic (surgery, trauma) 40-50%
o Medical (drugs, sepsis) 40-50%
o Pregnancy related 5-10%
Prognosis depends on the underlying conditions
o Acute Renal Failure can be Pre-renal, renal or post renal:
Pre-Renal
o Inadequate renal perfusion
Renal
o Glomerular/tubular/interstitial
Post-renal
o Obstruction
o Pathophysiology
o The cause of ARF is a combination of tubular and vascular events:
Tubular obstruction by casts and debris
Backleak of glomerular filtrate across damaged tubular epithelium

Vascular events
o Decreased perfusion pressure and reduced GFR
o Loss of ability to autoregulate glomerular blood flow and perfusion
o Clinical Features Presenting Signs and Symptoms
Recognition of raised/rising urea and creatinine in an ill patient
Decreased urinary output (common but not necessary)
Other clinical manifestations:
o Fluid overload or dehydration
o Pericarditis
o Abnormal mental status
o GIT symptoms
o Lab Findings:
Hyperkalaemia
o Investigate on ECG
o Treat if >6.0 +/- ECG changes
CaCl2, 10ml 10% IV over 5 mins
NaHCO3, 100ml 8.4% IV over 30 mins
o If there is hypercalcaemia, DO NOT give bicarbonate (calcium plus bicarb
= chalk)
Give insulin pluse 50ml 50% dextrose IV over 5 minutes
Consider dialysis
Metabolic Acidosis (high anion gap)
Hyperphosphataemia
Hypocalacaemia
Anaemia (variable)
Abnormal urinalysis and urine microscopy
o PRERENAL CAUSES OF ARF
Hypovolaemia
o Decreased renal perfusion accounts for 40-805 of ARF and is readily
reversible if treated appropriately. Aetiologies include:
Fluid Loss
GIT haemorrhage, vomiting, diarrhoea, burns, diuretics
Haemorrhage
Fluid Sequestration
Pancreatitis
Peritonitis
Burns
Rhabdomyolysis
Decreased effective arterial volume
Cardiogenic shock
Sepsis
Cardiovascular Failure
o Impaired cardiac output
Cardiac failure
MI
Tamponade
o Vascular Pooling
Anaphylaxis

Sepsis
Drugs

o RENAL CAUSES OF ARF


Established ATN (any cause):
o Prolonged renal hypoperfusion
Toxins
o Pigment
Rhabdomyolysis (trauma, muscle disease, coma, heat stroke, severe
exercise, K+ or PO4- depletion)
Haemolysis (transfusion reaction, malaria)
o Antibiotics
Eg aminoglycosides, penicillins, sulphonamides, amphotericin
o Contrast
o Other
NSAIDs
Blocks prostaglandin production vasoconstriction of the
afferent arteriole decreased renal perfusion
ACE Inhibitors
Decreased efferent arteriolar tone
Anaesthetic agents, heavy metals, organic solvents
o Pregnancy related
Septic abortion, uterine haemorrhage, eclampsia
o Primary Renal Diseases
Vascular disease
Vasculitis, occlusion, atheroembolism
Acute (rapidly progressive) glomerulonephritis
Acute interstitial nephritis
o POST RENAL CAUSES OF ARF
Outflow obstruction
o Prostatic hypertrophy
o Bladder, pelvic, retroperitoneal tumours
o Retroperitoneal fibrosis
o Stones
Bladder trauma
Crystals (eg uric acid)
ABSOLUTE ANURIA USUALLY HAS A MECHANICAL CAUSE
o DIAGNOSIS OF ARF
Aims
o Identify whether ATN is present
o Diagnose incipient ATN (potentially reversible)
History
o Features of pre-existing renal disease
o Drug history
Examination
o State of hydration
o Signs of obstruction (eg palpable bladder)
o Urine volume and urine sediment

o It is important to determine whether this is ARF alone, or if the patient has


a background of chronic renal failure.
o Suspect chronic renal failure if:
History of comorbidity
o Eg DM, hypertension, long duration of symptoms, previously abnormal
blood tests (from GP etc)
small kidneys on US (<9cm) with increased echogenicity
Uraemic appearance
Neuropathy or osteodystrophy
o INVESTIGATIONS
ALWAYS
o Creatinine/urea/electorlytes
o CBE
o Urinalysis
o Renal imaging (ultrasound)
SOMTIMES
o Urinary electrolytes (in absence of diuretics)
o Renal biopsy
o Retrograde or anterograde pyelography
o Renal perfusion scan
o Further biochemistry (eg CK, myoglobin)
o Clinical Phases of ARF
INCIPIENT PHASE
o Follows precipitating event, potentially reversible
o Oliguria develops and there is retention of Na+
Urine Na <10mmol/L
OLIGURIC PHASE
o Established ATN
o Ability to retain Na+ lost
Urine Na >20 mmol/L
o Average duration 10-14 days but this is variable
DIURETIC PHASE
o Increased urine output precedes recovery of GRF
o Tubular function still impaired limited capacity to reabsorb sodium and
water
o Prevention
Identify at risk patients
o Eg the elderly and those with pre-existing renal impairment
Hydrate during fasting and avoid dehydration
Avoid or minimise contrast in high risk patients
o Renal impairment, myeloma, diabetes
Maintain high urine output during chemotherapy
Take care with nephrotoxic drugs
o Management of ARF
INCIPIENT
o Rule out obstruction (ultrasound, IDC)
o Correct fluid status

Usually IV saline and consider central venous monitoring


o Diuretic if no response within several hours
High dose loop diuretic (eg frusemide 80-250mg IV)
o No response = established ATN
ESTABLISHED ATN
o Fluid replacement regime, linked to urine fluid output
Insensible losses (eg 500ml) plus output per day
o Daily monitoring of biochemistry
Creatinine, urea
Potassium
Acidosis, Ca2+, PO4o Treat intercurrent or underlying problems
DIURETIC PHASE
o Urine output may reach very high levels, commonly wasting Na+, K+,
Ca2+
o Strict fluid balance charting vitals, weight daily
o Daily biochemistry
Watch Na+, K+, Ca++
o Fluid replacement regime
Urine output plus insensible losses, often adjusted every few hours
Usually use saline plus KCl
INDICATIONS FOR ACUTE DIALYSIS
o Severe hyperkalaemia
o Significant and resistant fluid overload
o Established renal impairment with uraemic symptoms/signs
o Dialysable toxins
o Volume need eg parenteral nutrition
o
o

o
o ASSESSMENT OF FETAL GROWTH
o

o
o

o
o QUESTIONS

o QUESTION 1
o What are the key features that you would expect to be available from the
antenatal record and todays examination that will help you manage the
condition identified in the above symphysio-fundal height chart?
Accuracy of dates
Trend of growth
Pre-existing medical condition
o Hypertension
o Cardiac disorders
o Asthma
o Diabetes
o Renal disease
o Severe anaemia
o CT disorders
Fetal causes
o Congenital abnormality
o Placental/uterine fibroids, uterine anomaly, abnormal placentation
Maternal causes
o Nutrition + maternal weight
o Smoking/alcohol/substance abuse/medications
o Pre-eclampsia/thrombophilia
o
o QUESTION 2
o What investigations are required based on the above symphysio-fundal
height chart??
US for fetal growth
o This confirms or refutes small for gestation age OR growth restriction
o Measurements FAC, EFW, biparietal diameter, head circumference,
femur length
o Umbilical artery Doppler
o Amniotic fluid index
Bloods
o
o
o
o

Maternal serology CMV, Toxoplasmosis, Syphilis


CBE haemoconcentration, decreased platelet count
Thrombophilia screen LAC, aCL, aPC resistance, homocysteine, MTHFR 677,
CT/1298 AC
CVS/amniocentesis

o
o QUESTION 3
o What is the interpretation of the ultrasound fetal growth chart below?
FAC is 5th centile
Interpretation of FAC
o First reading
Is FAC is >10th centile suspicion of SGA is refuted reassure
mother and go back to routine care
If FAC <10th centile confirms suspicion of SGA/IUGR now need
to differentiate between SGA and IUGR repeat US in 2 weeks
o Follow up 2 weeks later
FAC follows same centile + normal Doppler and AFI SGA baby
is fine just small
FAC growth drops off and sits below 10th centile +/- abnormal
Doppler +/- abnormal AFI IUGR

o
o
o
o QUESTION 3 US FETAL GROWTH

o
o
o

o
o QUESTION 4
o How should this case be managed?
Admission or regular review
Umbilical artery Doppler done twice a week (perfusion to fetus)
Amniotic fluid index done twice a week (renal perfusion + output)
USS trend fetal growth done fortnightly
Delivery
o Absent or reverse flow on Doppler LSCS
o Severe IUGR <32 weeks balance between prematurity and IUGR; can do
IOL or LSCS
o
o
o MARKING SCHEME
o

o
o

o ASSESSING GROWTH IN EARLY CHILDHOOD


o

o
o
o
o
o
o
o
o

o
o
o
o
o
o
o
o
o PLOTTING AGE 3MO
My notes say only adjust if over 8 weeks prem (but plot correctly as it asks for
adjustment)
Chart normally after 2 years
o

o
o
o
o
o
o
o
o
o PLOTTING AGE 2.5YO
No need to adjust

o
o
o

o MARKING SCHEME

o
o
o

o ASTHMA MANAGEMENT PLAN

o
o
o STEM
o Michael Smith is a 10 year old boy who has had 4 episodes of
acute wheezing with an upper respiratory tract infection in the
past 6 months. He also develops a cough and chest tightness
with exercise, and often coughs at night even when he is well.
Ventolin temporarily relieves his symptoms.
o
o QUESTIONS
o Please complete this asthma management plan for Michael to
treat his asthma.
o
o Please place your name label in the space provided and
complete BOTH sides of the form
o
o
o
o
o
o
o
o
o
o
o
o
o
o
o
o
o
o
o
o
o
o
o
o
o
o
o

o
o
o
o
o
o
o
o
o
o
o ANSWER
Type of asthma moderate persistent asthma?
o
o
o
o
o
o
o
o
o
o
o
No wheeze, cough or chest tightness
o
Can exercise without symptoms and non at night
o
Reliever needed less than 3x per week (not including before
exervise)
o
o
Preventer: ICS 200mcg beclomethasone dipropionate (BDP-HFA) daily
o
Reliever (if needed): salbutamol up to 12 puffs as required
o
o
o
o
o
o
o
o
Cold runny nose, productive cough
o
Starting to get wheeze or chest tightness, waking at night with
symptoms
o
o
o
Preventer: ICS 200mcg BDP-HFA daily
o
Reliever: Salbutamol puffer with spacer 8-12 puffs every 1-4hrs
o
o
o
o
o
o
o
o
Needing reliever more than every 3hrs for wheeze or chest tightness
o
Waking up at night with symptoms

o
o
o
o

Reliever: Salbutamol puffer with spacer 8-12 puffs every 15-30mins


Start oral steroid: 30mg daily prednisolone
o
o
o
o
o
o
o
o
o
o MARKING SCHEME

o
o

o
o
o
o
o
o
o
o

o PRESCRIBING ASTHMA
o
o Tom Anderson is a university student whose date of birth is 16
August 1986. He has a known history of asthma, which he takes
a salbutamol puffer whenever he exercises. He is usually
otherwise well without any problems. Recently, he has been
getting asthma symptoms at night around 3-4 times a week for
the past month and this time it woke him up from his sleep.
This has made him present to the emergency department.
o On further history, Tom has been well overall and has no past
medical history and is not on any medications. He smokes 5
cigarettes a day. He also mentions that if he takes penicillin, he
develops a rash all over his body. His GP (or might have been
ED) performed a pulmonary function test [showed normal FEV1,
normal FVC, FEV1/FVC 72% (normal >71%)]. The reversibility
post-bronchodilators is 14%.
Prescribe the appropriate medications from the Australian Medicines
Handbook provided to you
Write down any appropriate advice you would give to him
o
o
o
o
o
o

1.
2.
3.

4.

o
o
o
o
o
o
o
o
o
o
o
o
o
o
o
o
o
o
o
o
o
o
o
o
o
o
o
o Asthma Prescribing
o It is appropriate to add a preventer if a patient is using their SABA >3
times per week. Inhaled corticosteroids are an appropriate choice. Always
begin an inhaled corticosteroid at the lowest possible dose.
o The AMH states that they recommend ALWAYS prescribing a LABA (also
considered a symptom controller) when prescribing inhaled
corticosteroids.
o I am not sure if you would give prednisolone in this case (maybe do so to
be on the safe side?) but the dose is 30mg PO
o The regime recommended by oxford handbook is as follows:
o Start at the step most appropriate for severity, moving up if needed or
down if good control is achieved for >3 months. Rescue courses of
prednisolone may be used at any time.
Occasional short acting beta-2 agonist as required for symptomatic relief. If used
more than once daily, or night time symptoms, go to step 2
Add standard-dose inhaled steroid (eg beclamethasone 100-400 micrograms/12
hours) or start at a dose appropriate for disease severity and titrate as required
Add a long acting beta-2 agonist (eg salmeterol). If there is benefit but
inadequate control, continue and increase the dose of beclamethasone to 400
mcg/12 hours. If there is no effect from the LABA then discontinue. Review
diagnosis, consider theophylline or Leukotrine receptor antagonist
Consider trials of:
a. Beclamethasone 1000mcg/12 hours
b. Modified release oral theophylline
c. Modified release oral beta-2 agonist

d. Oral leukotriene receptor antagonist


e. Oral theophylline
5. Add regular prednisolone (1 dose daily at the lowest possible dose) and refer to
asthma clinic

o When writing a prescription:


Name
DOB
UR (if applicable)
Allergies
o On the regular medications section:
Inhaled beclomethasone
o 50-200 mcg BD (id probably write 50 mcg BD as his symptoms are not too
severe)
o Inhaled
Inhaled salmeterol
o DPI 50 micrograms
o BD
o Inhaled
o On PRN section
Salbutamol
Metred dose inhaler
1-2 inhalations (100-200 micrograms) as required
Max 4 times daily
o Counselling points
Corticosteroids
o Rinse mouth out after using
o Use every day, even if you are feeling better
o Do not use this medication in an acute exacerbation
LABA
o Salmetrol should not be used in an acute exacerbation, use the
salbutamol
o Use this every day even when you are feeling better
SABA
o Use in acute exacerbations
o Tell your doctor if using this more than normal
o Clean the mouthpiece regularly to prevent blockages
o Take 1-2 puffs 5-15 minutes before exercising
General
o Aim to quite smoking

o
o
o
o
o
o
o
o PRESCRIBING - HYPERLIPIDAEMIA
o
o Description of task:
o
o Mr David Smith is a 54-year old man who is presenting to your
clinic for follow-up of his recent cholesterol tests. The results
are as follows:
o
o
Total cholesterol: 7.8mmol/L (N < 5.5mmol/L)
o
LDL cholesterol: 6.5mmol/L (N < 4.5mmol/L)
o
HDL cholesterol: 1.3mmol/L (N >1mmol/L)
o
Triglyceride: 1.2mmol/L (N <1.5mmol/L)
o
o Tasks for student:
Take a brief history
Counsel him about non-pharmacological measures to lower his
cholesterol
Prescribe an appropriate medication and counsel him about the
medication you have prescribed
o

o
o
o
o
o
o
o
o
o
o
o
o
o
o
o
o
o
o
o
o
o

o
o
o
o
o
o HISTORY:
o Mr David Smith
54-year old man
Accountant
Lives with wife
o
o Tasks in Hx;
Ascertain what the patient understands about his results
Determine possible causes of hyperlipidaemia;
o Diet
o Exercise
o Obesity
o Fhx (familial hypercholesterolaemia)
Assess for complications related to hyperlipidaemia;
o Chest pain, IHD
o Cerebrovascular disease
o PVD etc.
o Fhx of above disease
Determine whether patient has any relative/absolute contraindications to lipid
lowering agents;
o Drug allergies
o Renal impairment

o
o
o
o
o

Pre-existing myopathy
Liver disease

LIFESTYLE MODIFICATION:
- Should be trialed for 3-6 months prior to commencing drug therapy
(must be at least 6 weeks)
o - Diet modification;
Reduced saturated and trans fats
Replace these fats with monounsaturated and polyunsaturated fats
Increase soluble fibre, fresh fruit and vegetables
Change to cholesterol lowering margarines etc.
o - Other;
Increase physical activity at least 20-30 mins of moderate intensity exercise
most days of the week
Weight loss
Limit alcohol intake
o
o PRESCRIBE APPROPRIATE MEDICATION:
o - Patient has elevated total cholesterol and LDL cholesterol, thus a statin
would be most appropriate first line agent
E.g. Simvastatin, Rosuvastatin, Atorvastatin
o
o - MOA: HMG-CoA reductase inhibitor, which reduces hepatic cholesterol
production
o - Drug choice:
Rosuvastatin and atorvastatin most commonly prescribed fewest interactions
and can be taken at any time of day
o - Dosage:
Start at lower end of dose range to reduce potential side-effects (unless post
stroke or MI, start at high dose)
Increase dose if necessary at 4 week intervals
E.g. if prescribing Atorvastatin, prescribe 10-20mg once daily (safe range is 1080mg once daily)
o - Counseling;
Once daily oral medication
Take in evenings if possible as morning doses can be less effective (not with
rosuvastatin or atorvastatin, these can be taken at any time)
Seek prompt medical advice if urine turns dark or if patient experiences any
muscle pain, tenderness or weakness
Need to check baseline LFTs and CK prior to commencing Tx, and reassess these
1-2 months after initiating therapy
o - Side-effects;
Common myalgia, mild transient GI upset, headache, insomnia, dizziness,
raised transaminases
Rare myopathy, rhabdomyolysis, renal failure, liver failure, peripheral
neuropathy, pancreatitis
o
o Statin PBS Criteria:
o - Restricted drugs, so can only be prescribed if criteria are met
o - High-risk patients can be commenced immediately;

o
o If patients do not meet the above criteria; (i.e. low-moderate risk patients)
1. Prescribe lifestyle and dietary modification as above, must be continued for at
least 6 weeks
2. Recheck fasting lipids after at least 6 weeks
3. If still elevated, assess patient against qualifying criteria (see below)
o
o
o
o
o
o
o
o
o
o
o
o
o
o
o
o
o
o
o
o
o
o
o
o
o
o

o
o
o
o
o
o
o

OTHER LIPID-LOWERING AGENTS:


Fibrates:
- E.g. fenofibrate, gemfibrozil
- Indicated for hypertriglyceridaemia
- MOA activate peroxisome proliferator-activated nuclear receptors,
increasing production of lipoprotein lipase, thus reducing triglycerides
o - Adverse effects GI upset, raised creatinine
Rare cholestatic jaundice, gallstones, anaemia, myopathy, rhabdomyloysis
o
o Bile acid-binding resins:
o - E.g. cholestyramine, colestipol
o - Indicated for hypercholesterolaemia and mixed hyperlipidaemia
o - MOA bind bile acids in intestinal lumen preventing their reabsorption
and increasing their excretion. Thus more cholesterol is used to replace
lost bile acid reduced cholesterol
o - Adverse effects constipation, abdominal pain, N+V, flatulence
o
o Ezetimibe:
- MOA reduces absorption of dietary cholesterol by inhibiting its
transport across the intestinal wall
o - Indicated for hypercholesterolaemia, useful in those intolerant to statins
o - Adverse effects headache, diarrhea, myalgia
o
o
o

o
o
o

o PRESCRIBING - DIABETES
o

o You are a General Practitioner seeing Mr. George Turnbull, who


is a 46 year old man with a recent diagnosis of Type II diabetes
mellitus. You have received his blood test results, which
indicate that his haemoglobin A1C is unacceptably high despite
his best efforts at dieting. He is 179 cm tall and his weight is
currently 106 kg. He is otherwise well, with no history of
cardiac, respiratory or renal problems, apart from mild
hypertension. He is taking atenolol 50 mg daily. On the
prescription form provided, write a prescription for the most
appropriate drug that you should prescribe for Mr. Turnbull to
treat his diabetes. Indicate in the space provided how you will
counsel Mr. Turnbull about his new medication, including when
and how you will follow him up.
o
o
o
o
o
o
o
o
o
o
o
o
o
o
o
o
o
o
o
o
o
o
o
o
o
o
o
o
o
o
o
o
o
o

o
o
o
o
o
o
o ANSWERS
o

o Metformin
o
o
o

Biguanide
See also Type 2 diabetes
For drug interactions see Metformin

o Mode of action
o

Reduces hepatic glucose production; increases peripheral utilisation of glucose.

o Indications
o

Type 2 diabetes in adults, including combinations with


glibenclamide, rosiglitazone, sitagliptinand vildagliptin

Type 2 diabetes in children >10 years

o Accepted
o

Anovulatory infertility due to polycystic ovary syndrome (under specialist supervision):

with clomiphene (if unresponsive to clomiphene alone) and BMI >30

as monotherapy, when clomiphene unsuitable (eg not tolerated or lack of access to appropriate
monitoring) and BMI <30

o Precautions
o

Ketoacidosiscontraindicated.

Respiratory failurecontraindicated.

Severe infection or traumacontraindicated.

Dehydrationcontraindicated.

Alcohol misusecontraindicated.

Moderate-to-severe heart failuremay increase risk of lactic acidosis.

o Contrast media
o

Intravascular iodinated contrast media (ICM) may cause renal failure and therefore
increase the risk of lactic acidosis, especially in those with pre-existing renal impairment.

Follow local protocols where they exist. Current Australian guidelines for adults
recommend:

patients with eGFR >60 mL/minute/1.73m2 who are to receive <100 mL ICM do not need to stop
metformin or have their renal function reassessed before continuing metformin after ICM
administration

patients with renal impairment or any patient receiving a large volume (>100 mL) of ICM should
stop metformin on the day of the procedure; restart metformin after at least 48 hours if renal
function is normal.

o Renal
o

Renal impairment increases risk of lactic acidosis; reduce maximum dose when CrCl is
<90 mL/minute; do not use when <30 mL/minute.

o Hepatic
o

Avoid use in severe hepatic impairment; risk of lactic acidosis.

o Surgery
o

Stop metformin before surgery; monitor blood glucose concentrations; replace with
insulin as required. Restart metformin when patient is no longer fasting and renal
function has recovered.

o Elderly
o

Use cautiously; monitor renal function and for adverse effects; reduce dose (or stop
treatment) if necessary.

Avoid combination with glibenclamide (the fixed-dose combination is particularly


unsuitable as dose titration is difficult).

o Pregnancy
o

Usually replaced with insulin; there is some clinical experience and metformin appears
safe, seek specialist advice; Australian category C.

o Breastfeeding
o

Safe to use.

o Adverse effects
o Common
o

malabsorption of vitamin B12, nausea, vomiting, anorexia, diarrhoea

o Infrequent
o

rash

o Rare
o

lactic acidosis (below), acute hepatitis

o Lactic acidosis
o

Rare, but often fatal; may be associated with metformin accumulation


when precautions or high-risk situations are overlooked. Early symptoms include
anorexia, nausea, vomiting, abdominal pain, cramps, malaise and weight loss.

o Dosage Metformin
o Type 2 diabetes

Adult

Conventional tablet, initially 500 mg 13 times daily; may be increased up to 850 mg 2


or 3 times daily according to response. Maximum daily dose 3 g.

Controlled release tablet, initially 500 mg once daily with the evening meal; may be
increased up to 2 g once daily. When changing from conventional tablets, start with the
patient's usual daily dose. (If >2 g daily is required, use conventional tablets.)

Child >10 years

Conventional tablet, initially 500850 mg once daily; maximum daily dose 2 g in 2 or


3 doses.

o Polycystic ovary syndrome


o

500 mg 2 or 3 times daily as tolerated has been used, may be increased up to 2 g daily.

o Renal impairment
o

A reduced maximum adult dose is suggested based on CrCl:

6090 mL/minute, 2 g daily.

3060 mL/minute, 1 g daily.

o Combination with glibenclamide


o

For additional information see Glibenclamide

Adult, initially 1 tablet of 500 mg metformin with 2.5 mg glibenclamide daily with
breakfast. Increase by 1 tablet (of this strength) every 2 weeks or longer according to
response.

Maximum dose, 1 tablet of 500 mg metformin with 5 mg glibenclamide 3 times a day.

Elderly, the manufacturer suggests an initial dose of 250 mg metformin with 1.25 mg
glibenclamide daily. Give with breakfast and increase dose according to response as
above. SeeElderly.

o Counselling
o

Take with or after food to reduce stomach upset.

Tell your doctor immediately if you have loss of appetite, nausea, vomiting, abdominal
pain, cramps, fatigue, diarrhoea or weight loss.

Drinking alcohol can affect control of your diabetes. It can also increase the risk of
serious side effects. Limit your alcohol intake, avoid binge drinking and have something
to eat when you drink alcohol.

o Combination with glibenclamide


o

Make sure that you, and your friends and family, know how to recognise and treat
hypoglycaemia (low blood glucose); ask your doctor or diabetes educator if you are
unsure.

o Practice points

slow onset of effect; control may take up to 2 weeks to establish

monitor renal function before starting treatment and every 46 months

increase dosage slowly to limit GI adverse effects; reduce or stop treatment if symptoms persist

consider temporarily stopping metformin if illness occurs that may alter renal function (eg
dehydration, shock, sepsis) or increase risk of tissue hypoxia and acidosis (eg MI, pulmonary
embolism)

pre-treatment with metformin may reduce the risk of ovarian hyperstimulation syndrome in
women with polycystic ovary syndrome undergoing IVF

o Combination with glibenclamide

monitor blood glucose when switching from standard tablets to fixed-dose combination because
tablets are not bioequivalent

do not use the previous doses of metformin and glibenclamide to start fixed-dose combination
therapy

o
o

o
o
o
o
o PRESCRIBING POST-OP ANALGESIA
o
o You have 6 minutes at this station
o Description of task:
o
o You are an intern working on the Surgical Unit. You have been
asked by the nursing staff to see Ms Elizabeth Burridge, who
requires a prescription for post-operative analgesia.
o
o She is a 72-year-old woman who had a resection of a rectal
carcinoma yesterday. She was initially started on Patient
Controlled Analgesia, but had difficulty managing it and has not
used it for over 4 hours. She no longer wishes to use this
method. She has been given intravenous Paracetamol 2 hours
ago, but is still in severe pain.
o
o Her past history includes moderate smoking-related COPD, but
she was otherwise well prior to her operation yesterday. She is
making a slow recovery and still has post- operative ileus.
o
o On the prescription form provided, write an appropriate
prescription for analgesia for Ms Burridge.
o
o Indicate in the space provided what requests you would make
of the nursing staff in relation to monitoring of Ms Burridges
condition.

o
o 1. Write a prescription for Ms Burridge on the form
below, using information from the AMH provided if you
wish. You must choose the most appropriate drug, dose

and route for the management of pain IN THIS PATIENT

o
o 2. Advice to nursing staff about monitoring this
patients condition.

o
o
o
o ANSWERS
o
o 1. Write a prescription for Ms Burridge on the form
below, using information from the AMH provided if you
wish. You must choose the most appropriate drug, dose
and route for the management of pain IN THIS PATIENT
o
o Rationale:

o Medication
o OXYCODONE

o
D
o
1
o

o
R
o
S

Dose

PRN

2.5 mg
o

Hourly frequency
1 hourly

o Max
dose
/24
hour
s

o Seda
tion
scor
e <2

o Pharmac
y

o Indication
o Post-operative pain

o Signature
o ldgs

o Print name
o LACHLAN STRANKS

o Contact
o

Patient experiencing severe pain, therefore SC route most appropriate to deliver


rapid analgesia on the ward
o IV PRN analgesia rarely given on ward, usually only in ED
Morphine or oxycodone would be appropriate oxycodone chosen in this
scenario as it is more potent, so more useful for severe pain
If patient had renal impairment, Fentanyl would be most appropriate
Dose: 7085 years, initially 2.55 mg every 2 hours
o Start at lower dose and titrate up to effect
o Hospitalized patients can be given dose every hour (with close
monitoring)

o
o 2. Advice to nursing staff about monitoring this
patients condition.

o - Monitor analgesic response and titrate dose up if necessary (to a


maximum dose of 5mg/hour with close monitoring)
o - Monitor sedation score and ensure it remains <2;
o 0 = wide awake
o 1 = easy to rouse
o 2 = constantly drowsy, easy to rouse but unable to stay awake early
respiratory depression
o 3 = severe; somnolent, difficult to rouse severe respiratory
depression
o - Monitor for other opioid side-effects; (non life-threatening
Nausea and vomiting
Constipation
Urinary retention
Delirium
Itch
Orthostatic hypotension
o
o

o
o
o

o
o
o
o

o PRESCRIBING URINARY INCONTINENCE


o STEM
o Please read the following clinical scenario, and then write a
prescription and advice to the patient as described below.
o
o You may use the Australian Medicines Handbook provided if you
wish.
o
o CLINICAL SCENARIO
o You are seeing Mrs Mary Smith, date of birth 21/12/1950, of 112
Merivale Lane, Adelaide 5000. She has presented with
troublesome urinary urgency, frequency and nocturia. This has
been a problem for a few years and has become worse since
her menopause 5 years ago. She has been under the care of a
physiotherapist, has been diligent with pelvic floor exercises
and bladder drill, and has been referred to you by the
physiotherapist for consideration of prescribed medical
treatment for her urinary symptoms. Mrs Smith has no children
and has no significant medical or family history. Her Pap smears
and mammograms are up to date and normal. She is allergic to
penicillin. This allergy is severe, causing breathing difficulties
and marked facial swelling. Cardiovascular, respiratory,
abdominal, breast and pelvic examination are normal. On
vaginal examination, there is a minor degree of prolapse of the
anterior vaginal compartment only (Stage 1). The vaginal
epithelium appears pale and thin. Recent urine analysis and
culture is normal.
o
o TASK
o 1. Write the appropriate prescription on the National Inpatient
Medication Chart provided for the two medications that should
be considered for this condition.
o
o 2. Write in the space provided the common and/or important
adverse effects that you would advise the patient to be aware
of.
o
o
o
o
o
o
o
o
o
o
o

o
o
o
o
o
o
o
o
o
o SIDE NOTE
o Comment: this marking template is provided unedited as an indicator of
the level of detail expected in relation to drug doses, clinical clerking, and
advice that may be required. In relation to advice in this exemplar, some
information provided by manufacturers regarding topical oestrogen
therapy is erroneous or controversial (it is often a copy and paste of
information that applies only to systemic regimens). This is getting in to
the postgraduate arena, so this question in its present form is
inappropriate for year 5. However, the principles illustrated apply to any
prescribing question, for example:
the subtleties of dosing regimens for oxybutynin hydrochloride (see A
criterion) do not appear in the common handbooks such as Mims or AHM.
Students who write out a standard dose for patients in Mrs Smiths age group,
as recommended in these handbooks, will receive a B, whereas students who
have attended clinics, undertaken additional reading on the subject, or
otherwise have learned that starting at a lower than recommended dose is
preferable in order to maximise compliance, will score above expected
standard for that criterion.
the criteria completeness of the prescription and ADR/allergy data as listed
are standard for a Year 5 prescribing template
o
o MARKING SCHEME
o CHOICE OF DRUG
Oestriol
o 0.5mg pessary nocte daily for 2 weeks, then 0.5mg nocte 2x per week
Oxybutynin
o 2.5mg tab BD, increase as tolerated to max dose 5mg TDS
o A- oestriol or oestradiol intravaginal cream, ovules or pessaries AND
oxybutynin hydrochloride tablets
o B 2nd line anticholinergics e.g. oxybutynin patchy or solifenacin tablet
o C 3rd line anticholinergics e.g. TCA
o D oral or transdernal estrogen +/- progesterone
o E antibiotic
o
o CHOICE OF STARTING DOSE
o Mg, route, time
o A intravaginal therapy (initially nocte for 14 nights, then twice weekly);
oxybutynin tabs (initially 2.5mg BD increasing as required and tolerated to
max dose of 5mg tds)
o B - Intravaginal therapy initially nocte for 14 nights then twiceweekly;
Oxybutynin tabs - 5mg bd.
o C - Intravaginal therapy initially nocte for 14 nights then twice weekly;
Oxybutynin tabs - 5mg tds.
o

o
o
o
o
o
o
o
o

COMPLETENESS OF PRESCRIPTION
A - Patient details, prescriber ID, dose, route and frequency
ADR/ALLERGY DATA
A - Drug (penicillin), reaction (anaphylaxis), prescriber ID and date

ADVICE TO PATIENT
A Intravaginal therapy: increase in physiological vaginal discharge,
possible local irritation, if significant systemic absorption may experience
headache, breast discomfort which should be reported.
o Oral oxybutynin: a wide range of anticholinergic side effects are reported,
including drowsiness, dry mouth, reflux, constipation, urinary retention,
visual disturbances, cardiac arrhythmia, and especially in elderly
dizziness, hyperthermia related to reduced sweating
o
o Advises what to do if side effects encountered (reduce dose or cease
medication, depending on severity)

o DIPSTICK BLOOD TEST


o

o
o

1.
2.
3.
4.
5.
6.
7.
8.

o
o
o
o
o
o
o
o
o
o
o
o
o
o
o
o
o
o
o
o
o Task 1
o Demonstrate and describe to the examiner how you would take this
sample from the patient using the display hand provided.
o
o Task 2
o Undertake the reagent strip testing. Describe to the examiner what you
are doing.
o GLUCOSE AND KETONES CAN BE MEASURED ON THE SAME DEVICE, BUT
REQUIRE 2 DIFFERENT TESTING STRIPS, thus test needs to be performed
twice
only machine that can test both is the Optium Xceed meter
o
wash hands and put on gloves
Calibrate reagentstrip and glucometer
Check they are not expired
Place reagentstrip into glucometer
Note: alcohol swab of site can be omitted
Prick lateral side of finger (less painful) and place
pricker into sharps
Collect blood onto reagentstrip and place pressure onto
puncture site
Read the glucometer reading
o
o http://www.youtube.com/watch?v=rMMpeLLgdgY
o
o Task 3
o Inform the examiner of the result of the test (=
your differential diagnoses).
Interpret results
Hi this means BGL is >27.8mmol/L (according to one
manual I read)
Ketones +++

Differentials DKA, alcoholic ketoacidosis (no hyperglycaemia)


o
o Task 4
o Outline your plan of management (further testing, care and treatment) for
this patient to the examiner.
MLTU answer
o Urgent fluid replacement
o Lowering of blood ketones with IV therapy
o Monitoring of ABG, glucose and potassium
o Identify cause
o Note: IV bicarbonate gives you an unsatisfactory grade as it is
controversial
My answer
o Clinical features
Hyperventilation - Kussmaul respiration (deep)
Nausea and vomiting
Confusion, stupor, Coma
Ketone breath
polyuria
o Causes
Didnt take insulin
New onset diabetes type 1
Currently unwell infection (UTI, pneumonia common), pancreatitis,
MI
New medication glucocorticoids, high dose thiazide,
sympathomimetic (dobutamine), 2nd generation antipsychotics
o Further investigations
Serum BGL level and ketones to confirm hyperglycaemia
Electrolytes hyponatremia, hypochloremia, low bicarbonate,
hyperkalaemia
Renal function raised BUN and creatinine (hypovolaemic)
Urine ketones/glucose (not necessary as blood has diagnosed
DKA), UTI
Plasma osmolality
ABG metabolic acidosis
CBE raised WBCC for infection as cause
ECG hyperkalaemia
If indicated by history
CXR infection
Amylase/lipase if pancreatitis suspected
Urine MCS
o Management
ABC, mental status
IV saline at 10-15mL/kg of lean body weight/hr (max of 50mL/kg in
first 4hrs)
Insulin infusion + potassium (once <5.3)
Once BGLs <12mmol/L give 5% dextrose with insulin until normal
levels
Monitoring hourly BGL, MBA20/osmolality/pH every 2hrs
o Monitor for complications
Hypotension renal hypoperfusion

Coma - must place a nasogastric tube to prevent aspiration


Cerebral edema - usually caused by excessive rehydration and use
of hypertonic fluids such as bicarbonate
Hypothermia body, temp drops below 33
Late complications - stasis pneumonia, DVT

o
o DKA
DKA is characterized by the triad of hyperglycemia, anion gap metabolic
acidosis, and ketonemia
Average fluid loss in DKA is 3-6L
o
o
o
o
o
o
o
o
o
o
o
o
o
o
o
o
o
o
o
o MARKING SCHEME

o
o
o
o

o ECG INTERPRETATION

o
o A 78-year-old woman presents to you (her GP) complaining of
palpiations. She is not able to tell you how long she has been
experiencing these palpitations for, but she thinks it might have
been a while. She says she is sometimes also feeling
lightheaded and fatigues easily.
o She has a long-standing history of diabetes and was identified
as having hypertension 10 years. Her BP today was 155/100.
She does not have heart failure and has not had a stroke or TIA.
She is on no medication other than Metformin and Lasix.
o
o You decide to perform an ECG, which reveals the following.

o
o
o
o
o
o
o
o

1. Describe the ECG. What are the abnormalities?


2. What is the diagnosis?
3. What are the causes of this ECG abnormality?

4. She will require pharmacological management for her


condition. Determine which drugs you would prescribe
and your reasons for prescribing them.
o What management option(s) is/are not appropriate in
this woman?
o

o
o
o
o
o
o
o
o
o
o
o
o
o
o
o
o
o
o
o
o
o
o
o
o
o
o
o
o

ANSWERS
1. Describe the ECG. What are the abnormalities?
- Rate: 152 bpm
- Rhythm: irregularly irregular
- Axis normal
- Absent P waves, tachycardia
2. What is the diagnosis?
Atrial fibrillation
3. What are the causes of this ECG abnormality?
- IHD, MI
- Valvular disease
- HT
- SA disease
- Cardiomyopathy
- Congenital heart disease
- Thyrotoxicosis
- Sepsis, infection
- Alcohol
- PE
- Idiopathic

4. She will require pharmacological management for her


condition. Determine which drugs you would prescribe and your
reasons for prescribing them.
o What management option is not appropriate in this woman?
o Rate controlling agent:
Cardioselective B-blocker metoprolol, atenolol, bisoprolol OR
Diltiazem
Verapamil
2nd line = digoxin
o
o Anticoagulation based on CHADS2 score
>75 = 1 point
Diabetic = 1 point
Hypertension = 1 point
o 3 points patient is at high risk of thromboembolic event so should
receive warfarin as anticoagulation
o
o What management is not appropriate?
Electrical cardioversion not haemodynamically unstable
Rhythm control patient has had AF for a while, not recommended in older
patients, does not have CCF

o
o
o

o
o
o
o
o

o ECG INTERPRETATION
o
o You have 6 minutes to complete this task. 1 minute reading
time
o 1. Place ECG Leads in the correct position on the
dummys chest
o 2. Here is the ECG obtained. Please complete the
following tasks
a) Describe this ECG and its characteristics (rate, rhythm etc)
b) State the diagnosis
c) Give 3 other signs/symptoms that this patient may be
experiencing and acute management options
o

o
o
o
o
o
o
o
o
o
o
o
o
o
o

o
o
o
o
o
o
o
o
o
o
Lead Placement
o MUST correctly place all leads

o
o Lead Placement
V1: 4th ICS, Right sternal margin
V2: 4th ICS, Left sternal margin
V3: Between V2 and V4
V4: 5th ICS, left MCL
V5: 6th ICS, anterior axillary line
V6: 6th ICS, Mid axillary line
o
o ECG INTERPRETATION
SHOULD
o Correctly describe rate, rhythm, axis, QRS duration and morphology, ST
and T wave morphology

o Identify
Prolonged PR interval
Broad, bizarre QRS
complexes
Peaked T waves
o Other features include:

o
SHOULD
o Correctly diagnose
hyperkalaemia
o The easiest way to
determine the rate of an
ECG (in a 10s ECG) is to count the number of QRS complexes and multiply
this number by 6. It can also be done by counting the number of little
squares between QRS complexes and dividing 300 by this number.
o A quick way calculate axis is by looking at leads I and aVF

o
Lead
o
Lead
o
Lead
o
Lead
o

If the axis is in the left quadrant, take a


look at lead II
IF:
I is negative and aVF is negative
Axis is deviated to the northwest territory
I is positive and aVF is positive
The axis is normal
I is negative and aVF is positive
The axis has deviated to the right
I is positive and aVF is negative
The axis has deviated to the left, NOW:
Lead II positive = normal axis
Lead II negative = Left axis deviation

o NORMAL QRS: 3 squares


o SHOULD: Identify diagnosis: Hypokalaemia
If unable to do so, diagnosis can be given
o MUST:
List at least 3 signs/symptoms.
o Fast, irregular pulse
o Chest pain
o Weakness
o Palpitations

o Light-headedness
o Paraesthesia
o Fatigue
Be able to provide acute management in this patient (N: 3.5-5)
o Monitor ECG
o 10mL calcium gluconate IV
Does not change serum potassium but is cardio-protective
o Insulin+ glucose
Moves K+ into cells
o Nebulized salbutamol also causes K+ to enter cells
o Last resort: Dialysis
o

o
o

o ECG INTERPRETATION
o

o
o

1) Location of chest leads

V1: right 4th intercostal space


V2: left 4th intercostal space
V3: halfway between V2 and V4
V4: left 5th intercostal space, midclavicular line
V5: horizontal to V4, anterior axillary
line
V6: horizontal to V5, mid-axillary line

o
o
o

2) Here is the ECG obtained. Please answer the following questions about this
ECG:
o a) Describe this ECG and its characteristics (e.g. rate, etc).
Rate - 150
Rhythm irregular
Axis normal

QRS duration and morphology rapid ventricular response


St segment normal
T wave morphology normal
o
o b) What is the ECG diagnosis?
Diagnosis AF
Ventricular response rapid an irregularly irregular
o
o c) What four (4) additional points of further history would you need to ask?
MUST Chest pain
MUST SOB
Palpitations
Syncope/dizziness
Relevant PMH
medications

o
o
o
o
o
o
o

o
o
o

o
o
o
o
o
o
o

o
o
o PATHOLOGY
o
o You have 6 minutes at this station Description of task:
o A 70 year old man of Egyptian descent, who migrated to
Australia in the 1970s, presented with haematemesis and
collapse and was found to have tachycardia, a BP of 90/60
mmHg, sweatiness, pallor, gynaecomastia and spider naevi.
o You are provided with:
1.

A pathology pot containing a liver

2.

A photomicrograph of a normal portal tract

3.

A serology report
o Please examine the material provided and answer the
questions on the answer sheet provided.
o
o
o
o
o
o
o
o
o

o
o
o
o

o Normal portal tract:

o
o Patients serology report:
o
o LFTs:
ALT: 110 U/L
(0-45)
AST: 80 U/L
(0-45)
GGT: 73 U/L
(0-59)
ALP: 120 U/L
(30-110)
Bilirubin: 36 mol/L (2-24)
Albumin: 21 g/L
(34-48)

o
o Coagulation studies:
INR/PT: Prolonged
o
o Hepatitis serology:
Hep B surface antibody (HBsAb): Positive
Hep B surface antigen (HBsAg): Negative
Hep C antibody: Positive
Hep C RNA detected

o
o
o
o Questions
o
o 1. The pathology pot contains a liver. What abnormality does it
show?
o
o
o
o
o 2. What complication related to the liver pathology is the most
likely cause of the patients acute clinical presentation AND
explain its pathogenesis in this disease.
o
o
o
o
o
o
o
o
o 3. Examine the photomicrograph, which shows a normal portal
tract with surrounding hepatocytes. Could this photomicrograph
have been taken from this patients liver? GIVE reasons for your
answer.
o
o
o
o
o
o
o
o
o 4. State the function of the features labeled A and B in the
micrograph.
o A:
o
o B:
o

o
o
o 5. Examine the serology report for this patient. What is the
underlying likely cause of the patients disease?
o
o
o
o
o 6. Briefly discuss the risk factors for acquiring the underlying
cause of the liver disease and which is likely in this patient.

o
o
o
o
o
o
o Answers
o

o 1. The pathology pot contains a liver. What abnormality does it


show?
o The pathology pot shows an enlarged liver with a nodular appearance.
This is consistent with cirrhosis. There is also a pale, necrotic, irregular
mass in the right lobe hepatocellular carcinoma.
o
o 2. What complication related to the liver pathology is the most
likely cause of the patients acute clinical presentation AND
explain its pathogenesis in this disease.
o The patient is most likely to have presented following rupture/bleeding
from oesophageal varices secondary to portal hypertension, occurring as
a result of cirrhosis.
o Cirrhosis disturbed liver structure increased resistance to portal
blood flow through hepatic sinusoids backpressure into portal/systemic
anastomoses dilation/engorgement of lower oesophageal vessels
(azygos vein/left gastric vein) oesophageal varices prone to bleeding
clinical presentation (haematemesis, hypovolaemic shock etc.)
o
o 3. Examine the photomicrograph, which shows a normal portal
tract with surrounding hepatocytes. Could this photomicrograph
have been taken from this patients liver? GIVE reasons for your
answer.
o NO, it is very unlikely to have been taken from this patients liver. Whilst
cirrhosis is histopathologically defined as fibrous band completely
surrounding regenerating nodules of hepatocytes, diffusely involving the
entire liver, one would expect histopathological abnormalities related to
the underlying cause (in this case chronic Hepatitis C infection most
likely).
o Expected histopathological abnormalities:
Proliferating hepatocytes
Related to chronic HCV infection;
o Large lymphoid aggregates in portal tracts

o
o
o
o
o

Steatosis
Necroinflammation
Hepatocyte apoptosis (interface hepatitis)

4. State the function of the features labeled A and B in the


micrograph.
o A: Bile ductule bile-carrying channel that drains bile produced by
hepatocytes into larger bile ducts
o B: Sinusoid large, expanded capillary spaces between hepatocyte plates
(separated from hepatocytes by space of Disse). Receive blood from
terminal branches of the hepatic artery and portal vein at the periphery of
lobules and deliver it into central veins.
o
o 5. Examine the serology report for this patient. What is the
underlying likely cause of the patients disease?
o Chronic hepatitis C infection.
o 6. Briefly discuss the risk factors for acquiring the underlying
cause of the liver disease and which is likely in this patient.
o Transmission source is from blood and body fluids.
o General risk factors for HCV infection:
IVDU
Tattoos
Sharing /reuse of contaminated
Piercings
needles
Unprotected sexual intercourse
Blood transfusions
(rare route of transmission)

In this patient, reuse of poorly or unsterilized medical equipment is the most


likely route of transmission, particularly considering the patients Egyptian
heritage. HCV infection is very common in Egypt due to the reuse of
contaminated needles during campaigns of parenteral anti-schistosomiasis
treatment in rural areas in the 1960s70s.

GOOD
LUCK!!!

Você também pode gostar